You are on page 1of 73

https://t.

me/Forum_SFG5
PTS 2024 | Test Code: 121405 - Solutions |

Q.1)
Ans) c
Exp) विकल्प c सही उत्तर है ।
➢ अमर नायक प्रणाली एक महत्वपूणण प्रशासननक और सैन्य संरचना थी, विसे दविण भारत में विियनगर साम्राज्य के
शासनकाल (1336-1646 CE) के दौरान लागू वकया गया था, खासकर 15वीं और 16वीं शताब्दी के दौरान। शब्द
"अमर नायक" का मोटे तौर पर अनुवाद "सामंत सरदार" या "कुलीन स्वामी" होता है । इस प्रणाली में सत्ता का
नवकेंद्रीकरण शानमल था, जहां प्रां तीय गवनणरों या प्रमुखों, नजन्हें अमर नायक के नाम से जाना जाता था, को अपने संबंनित
क्षेत्ों पर शासन करने में काफी स्वायत्तता दी गई थी।
➢ अमरनायक मूलतः विियनगर के राया (रािा) द्वारा वनयुक्त सैन्य कमाांडर थे ;
• उनकी प्राथनमक नजम्मेदारी अपने नननदण ष्ट क्षेत् के भीतर एक मजबूत सैन्य दल बनाए रखना था।
• इस टु कडी में घुड़सिार सेना (घोड़े सैवनक), पैदल सेना (पैदल सैवनक) और संभवतः युद्ध हाथी भी शानमल थे -
जो मध्ययुगीन भारतीय युद्ध का एक महत्वपूणण घटक था।
• इन टु कनडयों ने नवजयनगर सेना को रीढ़ प्रदान की, नजससे उन्हें अपने क्षेत् का नवस्तार करने और प्रनतद्वं नद्वयों से बचाव
करने में मदद नमली।
कथन 1 सही है : अमरनायकों को नवनशष्ट गााँ वों पर ननयंत्ण नदया गया था, नजन्हें "अमारा गााँ व" कहा जाता था। यह स्वानमत्व के
नलए नहीं था, बल्कि प्रशासननक ननयंत्ण का एक रूप था।
• इन गााँ वों की भूनम पर उनके पास स्वानमत्व अनिकार का अभाव था, नजसका अथण है नक उनके पास स्वयां भूवम नही ां
थी। भूवम अांततः राज्य की थी।
• हालााँ नक, उन्हें इन गााँ वों से होने वाली आय से संबंनित महत्वपूणण नवशेषानिकार प्राप्त थे। वे अपने क्षेत् के नकसानों,
कारीगरों और व्यापाररयों से कर और बकाया वसूल करते थे।
• इस आय से कई उद्दे श्य पूरे हुए:
• एक नहस्से ने सैननकों की ननरं तर आपूनतण सुनननित करते हुए, अपने स्वयां के सैन्य दल के रखरखाि को वित्त
पोवित वकया।
• एक अन्य भाग ने नवजयनगर के केंद्रीय खिाने में योगदान वदया।
• शेष भाग का उपयोग अमरनायक व्यक्तक्तगत खर्च और प्रशासन के वलए कर सकता था।
• राजस्व का कुछ नहस्सा मांवदरोां के रखरखाि और वसांर्ाई कायों के नलए भी इस्तेमाल नकया जाता था
कथन 2 सही है : अमर नायक वास्तव में नवजयनगर साम्राज्य के भीतर अपने पदों के वंशानुगत िारक थे। अथाण त यह पररवार
के भीतर एक पीढ़ी से दू सरी पीढ़ी तक चला जाता है । इससे व्यवस्था के भीतर ननष्ठा की भावना पैदा हुई। हालााँनक, यह ध्यान
रखना महत्वपूणण है नक राजा के पास अभी भी कुछ ननयंत्ण था। शल्कि संतुलन बनाए रखने और नकसी एक पररवार को
अत्यनिक शल्किशाली बनने से रोकने के नलए वह कभी-कभी अमरनायकों को अलग-अलग क्षेत्ों में स्थानां तररत कर सकता था।
कथन 3 सही है : पुतणगाली यात्ी फनाण ओ नुननज और डोनमंगो पेस ने नवजयनगर साम्राज्य की अपनी यात्ाओं के दौरान अमर
नायक प्रणाली में अमूल्य अंतर्दण नष्ट प्रदान की, विन्ोांने 16िी ां शताब्दी में तुलुिा राििांश के कृष्णदे ि राय और अच्युत राय के
शासनकाल के दौरान दौरा वकया था।
उनकी नटप्पनणयााँ साम्राज्य की प्रशासननक जनटलताओं और उसके भीतर नायकों की भूनमका पर प्रकाश डालती हैं ।
नुवनि और पेस ने नायकों को मुख्य रूप से रायस (नवजयनगर की केंद्रीय सरकार) के एजेंटों या प्रनतनननियों के रूप में नचनत्त
नकया, जो नायकों और राजशाही के बीच सामंती-जैसे संबंिों पर प्रकाश डालते हैं ।
यह संबंि दानयत्वों पर आिाररत था, नजसमें नायकों द्वारा रायों को नकया गया भुगतान भी शानमल था, नजसमें शासन की एक ऐसी
प्रणाली का सुझाव नदया गया था नजसमें केंद्रीय प्रानिकरण के नलए प्रशासननक नजम्मेदाररयां और नवत्तीय योगदान दोनों शानमल
थे, जो दु ननया के अन्य नहस्सों में सामंती प्रथाओं की नवशेषता थी।
ज्ञानधार:
अमर नायक प्रणाली के बारे में अन्य महत्वपूणण तथ्ों में शानमल हैं :

Forum Learning Centre: Delhi - 2nd Floor, IAPL House, 19 Pusa Road, Karol Bagh, New Delhi - 110005 | Patna - 2nd floor, AG Palace, E Boring Canal
Road, Patna, Bihar 800001 | Hyderabad - 1st & 2nd Floor, SM Plaza, RTC X Rd, Indira Park Road, Jawahar Nagar, Hyderabad, Telangana 500020
9311740400, 9311740900 | https://academy.forumias.com | admissions@forumias.academy | helpdesk@forumias.academy

[1]
https://t.me/Forum_SFG5
PTS 2024 | Test Code: 121405 - Solutions |

• सत्ता का विकेंद्रीकरण: नवजयनगर साम्राज्य, जो नवशाल और सां स्कृनतक रूप से नवनवि था, नवनभन्न क्षेत्ों को
कुशलतापूवणक प्रबंनित करने के नलए स्थानीय शासकों पर ननभणर था। अमर नायक साम्राज्य की केंद्रीय सत्ता और
स्थानीय आबादी के बीच मध्यस्थ के रूप में काम करते थे।
• इस प्रणाली ने शासन में कुछ हद तक क्तथथरता और लर्ीलेपन को बढािा वदया, नजससे साम्राज्य को अपने
नवशाल क्षेत्ों को प्रभावी ढं ग से प्रबंनित करने की अनुमनत नमली।
• कुछ शक्तक्तशाली अमर नायक धीरे -धीरे विियनगर की केंद्रीय सत्ता से स्वतांत्र हो गए, नजससे कभी-कभार
संघषण और सत्ता संघषण हुआ।
• 16वीं शताब्दी में नवजयनगर साम्राज्य के पतन के दौरान कई अमर नायकों ने अपनी स्वतांत्रता पर िोर वदया और
अपने स्वयां के िेत्रीय साम्राज्य थथावपत वकए।
Source: https://www.ncert.nic.in/ncerts/l/lehs203.pdf
https://www.egyankosh.ac.in/bitstream/123456789/73304/1/Unit-7.pdf

Q.2)
Ans) a
Exp) विकल्प a सही उत्तर है ।
1947 में निनटश शासन से स्वतंत्ता प्राप्त करने के बाद, भारत और पावकस्तान में विभािन के साथ भारतीय उपमहाद्वीप में
एक महत्वपूणण पररवतणन हुआ।
हालााँ नक, यह नवभाजन व्यापक नहीं था, क्ोंनक पूरे क्षेत् में कई ररयासतें वबखरी हुई थी ां, नजन्हें भारत या पानकस्तान में शानमल
होने या स्वतंत् रहने का नवकल्प नदया गया था।
इनमें से अनिकां श ररयासतों ने अंततः भारत या पानकस्तान में शानमल होने का नवकल्प चुना, नजससे उनमें से अनिकां श का
भारतीय संघ में एकीकरण हो गया।
हालााँ नक, कुछ ररयासतोां ने एकीकरण का विरोध वकया, इन राज्यों में कश्मीर, है दराबाद, जूनागढ़, जोिपुर, त्ावणकोर और
भोपाल उल्लेखनीय थे, नजन्होंने राष्टर को मजबू त करने के नलए नई भारतीय सरकार के प्रयासों के नलए अनूठी चुनौनतयााँ पेश कीं।
एकीकरण प्रविया में इन ररयासतोां को भारतीय सांघ में लाने के नलए बातचीत, कूटनीनत और कभी-कभी सैन्य/पुनलस
कारण वाई शानमल थी।
िम्मू और कश्मीर (अक्टू बर 1947):
क्तथथवत: जम्मू और कश्मीर एक ररयासत थी, नजस पर महाराजा हरर नसंह का शासन था, जो एक नहं दू राजा थे , लेनकन उनकी
आबादी मुल्किम बहुल थी।
विकल्प: शासक के पास भारत या पानकस्तान में शानमल होने का नवकल्प था।
ननणणय: मुल्किम बहुमत के बावजूद, महाराजा ने अक्टू बर 1947 में भारत के साथ नवलय पत् पर हस्ताक्षर नकए। इस समझौते ने
भारत को रक्षा, संचार और बाहरी मामलों पर ननयंत्ण नदया।
संघषण: पानकस्तान ने नवलय का नवरोि नकया, नजसके कारण प्रथम कश्मीर युद्ध (1947-1949) हुआ जो अनसुलझा है ।
िूनागढ (फरिरी 1948):
क्तथथवत: जूनागढ़ एक मुल्किम शानसत ररयासत थी, जो भारतीय क्षेत् से नघरी हुई थी और इसमें मुख्य रूप से नहं दू आबादी थी।
विकल्प: जूनागढ़ के नवाब, मुहम्मद महाबत खानजी III ने नहन्दू जनसां ल्कख्यकी बहुल होने के बावजूद पानकस्तान में शानमल होने
का फैसला नकया।
प्रवतविया: भारत ने भौगोनलक ननकटता और लोगों की इच्छा के कारण इसे स्वीकार करने से इनकार कर नदया। राजननयक
दबाव और जूनागढ़ के भीतर नवद्रोह के बाद, नवाब भाग गया और भारत ने सैन्य रूप से हस्तक्षेप नकया।
संकल्प: फरवरी 1948 में आयोनजत एक जनमत संग्रह में भारी बहुमत से भारत में शानमल होने का समथणन नकया गया और
उसी वषण जूनागढ़ को औपचाररक रूप से एकीकृत नकया गया।
है दराबाद (वसतांबर 1948):

Forum Learning Centre: Delhi - 2nd Floor, IAPL House, 19 Pusa Road, Karol Bagh, New Delhi - 110005 | Patna - 2nd floor, AG Palace, E Boring Canal
Road, Patna, Bihar 800001 | Hyderabad - 1st & 2nd Floor, SM Plaza, RTC X Rd, Indira Park Road, Jawahar Nagar, Hyderabad, Telangana 500020
9311740400, 9311740900 | https://academy.forumias.com | admissions@forumias.academy | helpdesk@forumias.academy

[2]
https://t.me/Forum_SFG5
PTS 2024 | Test Code: 121405 - Solutions |

ल्कस्थनत: है दराबाद एक बडी ररयासत थी, नजस पर मुल्किम राजा ननजाम का शासन था, नजसकी आबादी नहं दू बहुसंख्यक थी।
विकल्प: ननजाम ने शुरू में स्वतंत्ता का नवकल्प चुना, लेनकन भारत के साथ बातचीत जारी रही।
संघषण: ननजाम के अिणसैननक बल, नजसे रजाकार कहा जाता है , के उदय से तनाव बढ़ गया। नसतंबर 1948 में, भारत ने एक
पुनलस कारण वाई "ऑपरे शन पोलो" शुरू की, नजसने है दराबाद को तुरंत सुरनक्षत कर नलया।
पररणाम: ननजाम ने आत्मसमपणण कर नदया, जबनक एकीकरण प्रनिया दू सरों की तुलना में अनिक जनटल था। राज्य के भारत में
पूरी तरह एकीकृत होने से पहले उन्हें शुरुआत में कुछ समय के नलए है दराबाद का राज प्रमुख (संवैिाननक प्रमुख) बनाया गया
था।
गोवा (1961):
ल्कस्थनत: गोवा भारत के पनिमी तट पर एक पुतणगाली उपननवेश था, जो यूरोपीय उपननवेशवाद का अवशेष था।
बातर्ीत: भारत ने राजननयक चैनलों के माध्यम से बार-बार गोवा की वापसी की मां ग की, लेनकन पुतणगाल ने इनकार कर नदया।
कारच िाई: नदसंबर 1961 में, भारत ने एक त्वररत सैन्य अनभयान "ऑपरे शन नवजय" शुरू नकया, नजसने गोवा को पुतणगाली शासन
से मुि कराया।
एकीकरण: गोवा और अन्य दो पुतणगाली एन्लेव दमन और दीव को 1961 में एक एकल केंद्र शानसत प्रदे श (यूटी) में नमला नदया
गया था। नफर 1987 में गोवा को राज्य का दजाण नदया गया, जबनक दमन और दीव यूटी बने रहे ।
Source: India since Independence by Bipan Chandra
https://ncert.nic.in/textbook/pdf/leps201.pdf

Q.3)
Ans) b
Exp) विकल्प b सही उत्तर है
युग्म 1 गलत है : खाद्य अपनशष्ट सूचकां क ररपोटण :
प्रकावशत: संयुि राष्टर पयाण वरण कायण िम (यूएनईपी) खाद्य अपनशष्ट सूचकांक ररपोटण जारी करने के नलए निटे न ल्कस्थत गै र-
लाभकारी संगठन WRAP (अपनशष्ट और संसािन कारण वाई कायणिम) के साथ सहयोग करता है ।
उद्दे श्य: इस ररपोटण का उद्दे श्य खुदरा, खाद्य सेवा और घरे लू क्षेत्ों में वैनिक खाद्य बबाण दी को मापना और समझना है । यह भोजन
की बबाण दी को कम करने के नलए मजबूत अनुमान और मागणदशणन प्रदान करता है ।
महत्व: खाद्य अपनशष्ट की मात्ा ननिाण ररत करके, यह उन क्षेत्ों की पहचान करने में मदद करता है जहां अपनशष्ट को कम करने ,
ल्कस्थरता में सुिार करने और भूख और कुपोषण को संबोनित करने में हस्तक्षेप प्रभावी हो सकता है ।
खाद्य अपवशष्ट सूर्काांक ररपोर्च 2024:
• खाद्य अपनशष्ट सूचकां क ररपोटण 2024 का लक्ष्य खाद्य अपनशष्ट को कम करने और एसडीजी 12.3 प्राप्त करने की
नदशा में आवश्यक कारण वाई को उत्प्रेररत करना है ।
• ररपोटण खुदरा, खाद्य सेवा और घरे लू क्षेत्ों में खाद्य अपनशष्ट माप पर मागणदशणन को बढ़ाती है ।
• भोजन की अनिकां श बबाण दी घरों से होती है
• 2022 में वैनिक स्तर पर 1.05 नबनलयन टन खाद्य अपनशष्ट उत्पन्न हुआ, जो उपभोिाओं के नलए उपलब्ध कुल भोजन
के लगभग पां चवें नहस्से के बराबर है ।
• दु ननया भर के घरों में प्रत्येक व्यल्कि औसतन प्रनत वषण 79 नकलोग्राम भोजन बबाण द करता है , जबनक भारत में प्रनत
व्यल्कि प्रनत वषण यह दर 55 नकलोग्राम है ।
युग्म 2 गलत है : विश्व िायु गुणित्ता ररपोर्च :
प्रकावशत : IQAir वर्ल्ण एयर क्वानलटी ररपोटण वायु गुणवत्ता डे टा में नवशेषज्ञता रखने वाला एक ल्कस्वस संगठन IQAir द्वारा एक
वानषणक प्रकाशन है ।
सामग्री : ररपोटण दु ननया भर में नवनभन्न स्थानों पर 30,000 से अनिक वायु गुणवत्ता ननगरानी स्टे शनों के डे टा का नवश्लेषण करती
है ।

Forum Learning Centre: Delhi - 2nd Floor, IAPL House, 19 Pusa Road, Karol Bagh, New Delhi - 110005 | Patna - 2nd floor, AG Palace, E Boring Canal
Road, Patna, Bihar 800001 | Hyderabad - 1st & 2nd Floor, SM Plaza, RTC X Rd, Indira Park Road, Jawahar Nagar, Hyderabad, Telangana 500020
9311740400, 9311740900 | https://academy.forumias.com | admissions@forumias.academy | helpdesk@forumias.academy

[3]
https://t.me/Forum_SFG5
PTS 2024 | Test Code: 121405 - Solutions |

फोकस : यह वायु प्रदू षण के स्तर पर प्रकाश डालता है , सबसे प्रदू नषत दे शों की पहचान करता है , और वायु गुणवत्ता में सुिार
के नलए कारण वाई की आवश्यकता पर जोर दे ता है ।
िैवश्वक िागरूकता : ररपोटण नीनत ननमाण ताओं, शोिकताण ओं और जनता के नलए वायु गुणवत्ता चुनौनतयों को समझने और स्वच्छ
वातावरण की वकालत करने के नलए एक मूल्यवान संसािन के रूप में कायण करती है ।
नवि वायु गुणवत्ता ररपोटण 2023;
ऑस्टर े नलया, एस्टोननया, ऩिनलैंड, ग्रेनाडा, आइसलैंड, मॉरीशस और न्यूजीलैंड सनहत सात दे श WHO की वानषणक PM2.5
गाइडलाइन (िाविचक औसत 5 µg/m³ या उससे कम) को पूरा करते हैं ।
भारत का प्रदशचन: वायु गुणवत्ता के मामले में भारत नवि स्तर पर तीसरा सबसे प्रदू नषत दे श है । भारत में PM2.5 (पानटण कुलेट
मैटर) की औसत वानषणक सां द्रता 54.4 माइिोग्राम प्रनत घन मीटर है , जो नवि स्वास्थ्य संगठन के नदशाननदे श3 से 10 गुना
अनिक है ।
नवि वायु गुणवत्ता ररपोटण में सबसे अनिक प्रदू षण स्तर वाला दे श बां ग्लादे श है , उसके बाद पानकस्तान है |
युग्म 3 सही है : भ्रष्टाचार िारणा सूचकां क (सीपीआई):
प्रकावशत: टर ां सपेरेंसी इं टरनेशनल वानषणक रूप से सीपीआई जारी करता है ।
कायचप्रणाली: सूचकां क सावणजननक क्षेत् के भ्रष्टाचार के कनथत स्तरों के आिार पर 180 दे शों और क्षेत्ों को रैं क करता है ।
स्कोररां ग: दे शों को 0 (अत्यनिक भ्रष्ट) से 100 (बहुत स्वच्छ) के पैमाने पर स्कोर नकया जाता है ।
उद्दे श्य: ररपोटण नवि स्तर पर भ्रष्टाचार के रुझानों पर प्रकाश डालती है , सुिार के क्षेत्ों पर प्रकाश डालती है , और जवाबदे ही,
पारदनशणता और प्रभावी न्याय प्रणानलयों के महत्व को रे खां नकत करती है ।
िकालत: यह भ्रष्टाचार नवरोिी प्रयासों को प्रोत्सानहत करती है और शासन में सत्यननष्ठा को बढ़ावा दे ती है |
भ्रष्टार्ार धारणा सूर्काांक (सीपीआई) 2023:
वैनिक औसत 100 में से 43 पर ल्कस्थर बना हुआ है ।
दो-नतहाई से अनिक दे शों का स्कोर 50 से नीचे है , जो प्रचनलत और व्यापक भ्रष्टाचार की समस्या का संकेत है ।
हालााँ नक कुछ प्रगनत हुई है , 155 दे शों ने 2012 के बाद से भ्रष्टाचार के ल्कखलाफ कोई महत्वपूणण प्रगनत नहीं की है या नगरावट आई
है |
भारत की रैं क और स्कोर: सीपीआई 2023 में भारत 180 दे शों में से 93वें स्थान पर था। 2023 में भारत का कुल स्कोर 39 था,
जो 2022 में 40 से मामूली कमी है और 2022 में 85वें स्थान पर है ।
सबसे कम भ्रष्ट होने के मामले में डे नमाकण पहले स्थान पर है , उसके बाद नफनलैंड और न्यूजीलैंड हैं
सोमानलया को 180 रैं क और 11 के न्यूनतम स्कोर के साथ सबसे भ्रष्ट स्थान नदया गया था
युग्म 4 सही है : सडक सुरक्षा पर वैनिक ल्कस्थनत ररपोटण :
प्रकावशत: नवि स्वास्थ्य संगठन (डब्ल्यूएचओ) समय-समय पर सडक सुरक्षा पर वैनिक ल्कस्थनत ररपोटण जारी करता है ।
सामग्री: ररपोटण दु ननया भर में सडक यातायात से होने वाली मौतों का आकलन करती है और सडक सुरक्षा में कनमयों को दू र
करने के नलए आवश्यक कायों का मूल्यां कन करती है ।
कारच िाई का दशक: 2023 की ररपोटण सडक सुरक्षा के नलए कारण वाई के दशक 2011-2020 के दौरान हुई प्रगनत को दशाण ती
है ।
प्रभाि: यह सुरनक्षत सडक बुननयादी ढां चे, प्रवतणन और सावणजननक जागरूकता अनभयानों के महत्व पर जोर दे ते हुए सडक
यातायात में होने वाली मौतों और चोटों को कम करने के नलए नीनतयों, रणनीनतयों और हस्तक्षेपों की जानकारी दे ता है ।
सड़क सुरिा पर िैवश्वक क्तथथवत ररपोर्च 2023:
• वानषणक सडक यातायात मृत्यु थोडी कम होकर 1.19 नमनलयन हो गई है ।
• सडक सुरक्षा में सुिार के प्रयासों का असर हो रहा है , लेनकन 2030 तक सडक यातायात में होने वाली मौतों और
चोटों को आिा करने के लक्ष्य को प्राप्त करने के नलए तत्काल कारण वाई की आवश्यकता है ।
• दो-नतहाई मौतें कामकाजी उम्र के लोगों में होती हैं ।

Forum Learning Centre: Delhi - 2nd Floor, IAPL House, 19 Pusa Road, Karol Bagh, New Delhi - 110005 | Patna - 2nd floor, AG Palace, E Boring Canal
Road, Patna, Bihar 800001 | Hyderabad - 1st & 2nd Floor, SM Plaza, RTC X Rd, Indira Park Road, Jawahar Nagar, Hyderabad, Telangana 500020
9311740400, 9311740900 | https://academy.forumias.com | admissions@forumias.academy | helpdesk@forumias.academy

[4]
PTS 2024 | Test Code: 121405 - Solutions |

• भारत का सडक सुरक्षा प्रदशणन: कुल सडक मृत्यु दर में भारत की नहस्सेदारी 11% से बढ़कर 13% हो गई। (वैनिक
स्तर पर उच्चतम)
Source: https://forumias.com/blog/important-reports-and-index-in-news/

Q.4)
Ans) a
Exp) विकल्प a सही उत्तर है ।
1858 के भारत सरकार अनिननयम ने भारत के शासन में एक महत्वपूणण मोड लाया, नजससे निनटश ईस्ट इं नडया कंपनी के
शासन का अंत हो गया और भारत का प्रशासन सीिे निनटश िाउन को स्थानां तररत कर नदया गया। इस अनिननयम ने प्रभावी
रूप से निनटश राज की स्थापना की, जो औपननवेनशक शासन की एक प्रणाली थी, जो 1947 में भारत को स्वतंत्ता नमलने तक
चली।
कथन 1 सही है : 1858 के भारत सरकार अवधवनयम से पहले , ईस्ट इां वडया कांपनी लांदन में वनयांत्रण बोडच और वनदे शक
न्यायालय की दे खरे ख में भारत पर शासन कर रही थी। इस व्यवस्था को अक्सर "दोहरी सरकार" कहा जाता था, क्ोंनक
भारत के शासन की दे खरे ख करने वाली दो संस्थाएाँ थीं।
1858 के अवधवनयम ने वनयांत्रण बोडच और वनदे शक न्यायालय को समाप्त कर वदया। कांपनी के वनदे शक न्यायालय की
शक्तक्तयााँ भारत के राज्य सवर्ि में वनवहत थी ां। इससे भारत में कंपनी शासन का युग समाप्त हो गया और दे श के प्रशासन पर
सीिे निनटश ननयंत्ण स्थानपत हो गया।
कथन 2 गलत है : अनिननयम ने वास्तव में भारत की एक पररषद की स्थापना की, लेनकन इसका उद्दे श्य भारत के राज्य सनचव
की सहायता करना था, न नक भारत के वायसराय की। पररषद में 15 सदस्य शानमल थे और यह एक सलाहकार ननकाय थी।
भारत के राज्य सनचव निनटश कैनबनेट के सदस्य थे और भारतीय मामलों की दे खरे ख के नलए नजम्मेदार थे। भारतीय पररषद
भारत के शासन से संबंनित मामलों में राज्य सनचव को सलाह और सहायता प्रदान करती थी।
कथन 3 गलत है : नसनवल सेवकों के चयन और भती के नलए खुली प्रनतयोनगता प्रणाली वास्तव में 1853 के चाटण र अनिननयम
द्वारा पहले शुरू की गई थी। 1853 के चाटण र अनिननयम ने संरक्षण प्रणाली को समाप्त कर नदया और भारत की नसनवल सेवाओं
में प्रवेश के नलए प्रनतस्पिी परीक्षाओं की शुरुआत की।
कथन 4 गलत है : भारत के वायसराय ने , न नक भारत के राज्य सनचव ने , भारत में निनटश िाउन के प्रत्यक्ष प्रनतनननि के रूप में
कायण नकया। भारत का राज्य सनचव निनटश कैनबनेट का सदस्य था और भारत के प्रशासन के नलए निनटश संसद के प्रनत
उत्तरदायी था।
भारत के नलए राज्य सनचव के कायाण लय के ननमाण ण का उद्दे श्य निनटश सरकार के तहत भारत के प्रशासन को केंद्रीकृत और
सुव्यवल्कस्थत करना था। राज्य सनचव के पास भारतीय मामलों पर अनिकार था और वह निनटश सरकार और भारत में वायसराय
और उनकी सरकार के बीच संचार का प्रमुख माध्यम था।
Source: Indian Polity by M. Laxmikanth, Chapter-1.
https://cbc.gov.in/cbcdev/crown/crown1.html

Q.5)
Ans) b
Exp) विकल्प b सही उत्तर है।
कथन 1 सही है : असहयोग आं दोलन 1920 से 1922 तक महात्मा गां िी के नेतृत्व में भारतीय स्वतंत्ता संग्राम में एक महत्वपूणण
चरण था। इसका प्राथनमक उद्दे श्य भारतीयों को अपना सहयोग वापस लेने के नलए प्रोत्सानहत करके अनहं सक तरीकों से भारत
में निनटश शासन का नवरोि करना था। इस आं दोलन ने भारत में निनटश अथण व्यवस्था और प्रशासन को कमजोर करने और
अंततः भारतीय स्व-शासन (स्वराज) का मागण प्रशस्त करने की मांग की।
असहयोग आां दोलन की मुख्य विशेिताएां शावमल हैं :

Forum Learning Centre: Delhi - 2nd Floor, IAPL House, 19 Pusa Road, Karol Bagh, New Delhi - 110005 | Patna - 2nd floor, AG Palace, E Boring Canal
Road, Patna, Bihar 800001 | Hyderabad - 1st & 2nd Floor, SM Plaza, RTC X Rd, Indira Park Road, Jawahar Nagar, Hyderabad, Telangana 500020
9311740400, 9311740900 | https://academy.forumias.com | admissions@forumias.academy | helpdesk@forumias.academy

[5]
PTS 2024 | Test Code: 121405 - Solutions |

• विवर्श िस्तुओ ां का बवहष्कार : गां िी ने भारतीयों से निनटश नननमणत वस्तुओं का बनहष्कार करने का आग्रह नकया,
इसके बजाय भारतीय नननमणत उत्पादों (स्वदे शी) के उपयोग को बढ़ावा नदया। इससे न केवल अंग्रेजों पर आनथणक प्रभाव
पडा बल्कि भारतीय उद्योगों के नवकास और आत्मननभणरता को भी बढ़ावा नमला।
• विवर्श सांथथानोां का बवहष्कार : असहयोग आं दोलन ने स्कूलों, कॉलेजों और अदालतों जै से निनटश-संचानलत
संस्थानों के बनहष्कार का आह्वान नकया। औपननवेनशक व्यवस्था पर ननभणरता कम करने के नलए भारतीयों को राष्टरीय
शैक्षनणक संस्थानों की स्थापना और समथणन करने के नलए प्रोत्सानहत नकया गया।
• सरकारी पदोां से इस्तीफा : भारत में निनटश प्रशासन को कमजोर करने के तरीके के रूप में पुनलस अनिकाररयों
और सैननकों सनहत भारतीय सरकारी कमणचाररयों से अपने पदों से इस्तीफा दे ने का आग्रह नकया गया। भारतीय राष्टरीय
कां ग्रेस के सदस्यों सनहत कई प्रमुख नेताओं ने निनटश सरकार द्वारा दी गई अपनी उपानियााँ और सम्मान त्याग नदए।
• र्ुनािोां का बवहष्कार : असहयोग आं दोलन ने 1920 में नविान पररषदों के चुनावों के बनहष्कार की वकालत की।
चुनावों में भाग न लेकर, भारतीय नेताओं ने भारत में निनटश राजनीनतक व्यवस्था को अवैि बनाने और औपननवेनशक
सरकार में नविास की कमी को प्रदनशणत करने की कोनशश की।
• शाांवतपूणच विरोध और प्रदशचन: पूरे दे श में बडे पैमाने पर नवरोि प्रदशणन, माचण और सभाएं आयोनजत की गईं, नजसमें
प्रनतभानगयों ने अनहं सा और शांनतपूणण प्रनतरोि का पालन करने का वचन नदया। इन आयोजनों ने जीवन के सभी क्षेत्ों
के लोगों को एकजुट करने, राष्टरीय एकता और साझा उद्दे श्य की भावना पैदा करने में मदद की।
कथन 2 गलत है : एनसीएम को लॉन्च करने वाला आनिकाररक प्रस्ताव नसतंबर 1920 में कलकत्ता में आयोनजत भारतीय राष्टरीय
कां ग्रेस (आईएनसी) के नवशेष सत् में पाररत नकया गया था। यह एक महत्वपूणण मोड था क्ोंनक कां ग्रे स ने औपचाररक रूप से
गां िी की अनहं सक नवरोि की रणनीनत को अपनाया। अमृतसर अनिवेशन (पंनडत मोतीलाल नेहरू की अध्यक्षता में) नदसंबर
1919 में हुआ, लेनकन यह मुख्य रूप से जनलयांवाला बाग नरसंहार की ननंदा पर केंनद्रत था।
कलकत्ता अनिवेशन के अध्यक्ष: कलकत्ता अनिवेशन की अध्यक्षता लाला लाजपत राय ने की थी
महत्वपूणच प्रस्ताि पाररत: कलकत्ता सत् में कई प्रमुख प्रस्ताव पाररत नकए गए, नजनमें शानमल हैं :
• सरकारी संस्थानों का बनहष्कार: इसमें स्कूल, कॉलेज, अदालतें और नविान पररषदें शानमल थीं।
• निनटश सरकार द्वारा प्रदत्त उपानियों का समपणण।
• निनटश आयात का मुकाबला करने के नलए स्वदे शी (घरे लू रूप से उत्पानदत वस्तुओं का उपयोग) को बढ़ावा दे ना।
• निनटश नशक्षा का नवकल्प प्रदान करने वाले संस्थानों के साथ एक राष्टरीय नशक्षा प्रणाली की स्थापना।
कथन 3 सही है : एनसीएम ने शैक्षनणक संस्थानों सनहत सरकार द्वारा संचानलत संस्थानों के बनहष्कार को प्रोत्सानहत नकया। इस
बनहष्कार के कारण कई राष्टरवादी शै क्षनणक संस्थानों की स्थापना हुई, नजनमें शानमल हैं :
जानमया नमनलया इिानमया (1920): मौलाना मुहम्मद अली जौहर और हकीम अजमल खान द्वारा अलीगढ़ में स्थानपत।
काशी विद्यापीठ (1921): नशव प्रसाद गुप्ता और भगवान दास द्वारा वाराणसी में स्थानपत, इसने नशक्षा में भारतीय भाषाओं और
संस्कृनत को बढ़ावा दे ने पर ध्यान केंनद्रत नकया।
एनसीएम के दौरान स्थानपत अन्य शैक्षनणक संस्थान:
ऊपर उल्कल्लल्कखत संस्थानों के अलावा, एनसीएम के दौरान स्थानपत अन्य प्रमुख संस्थानों में शानमल हैं :
गुिरात विद्यापीठ (1920): महात्मा गां िी द्वारा अहमदाबाद में स्थानपत, इसने आत्मननभणरता और व्यावसानयक प्रनशक्षण को
बढ़ावा दे ने पर ध्यान केंनद्रत नकया।
वबहार विद्यापीठ (1921): महात्मा गां िी द्वारा पटना में स्थानपत, इसका उद्दे श्य नबहार में राष्टरवादी नशक्षा प्रदान करना था।
Source: A Brief History of Modern India by Rajiv Ahir, Chapter 9.

Q.6)
Ans) d
Exp) विकल्प d सही उत्तर है।

Forum Learning Centre: Delhi - 2nd Floor, IAPL House, 19 Pusa Road, Karol Bagh, New Delhi - 110005 | Patna - 2nd floor, AG Palace, E Boring Canal
Road, Patna, Bihar 800001 | Hyderabad - 1st & 2nd Floor, SM Plaza, RTC X Rd, Indira Park Road, Jawahar Nagar, Hyderabad, Telangana 500020
9311740400, 9311740900 | https://academy.forumias.com | admissions@forumias.academy | helpdesk@forumias.academy

[6]
PTS 2024 | Test Code: 121405 - Solutions |

युग्म 1 गलत सुमेवलत है : अल्कखल भारतीय अस्पृश्यता नवरोिी लीग की शुरुआत 1932 में महात्मा गां िी द्वारा की गई थी।
उन्होंने जेल में रहते हुए अनभयान शुरू नकया था। लीग का प्राथनमक उद्दे श्य अस्पृश्यता से लडना और ननचले और नपछडे वगों
की सामानजक ल्कस्थनत को ऊपर उठाना था।
डॉ. अम्बेडकर ने दनलत वगों के उत्थान के नलए भी कई पहल कीं। उन्होंने 1936 में इं नडपेंडेंट लेबर पाटी का गठन नकया और
दनलत वगों के मुद्दों को संबोनित करने के नलए 1927 में "बवहष्कृत भारत" समार्ार पत्र शुरू वकया।
युग्म 2 सही सुमेवलत है : 1873 में, ज्योनतबा फुले ने सत्यशोिक समाज की स्थापना की, नजसका अनुवाद "सत्य शोिक समाज"
है । इस संगठन का उद्दे श्य समाज में उच्च जानत के वचणस्व और िाह्मणवादी वचणस्व को चुनौती दे ना था। समाज का नेतृत्व मुख्य
रूप से माली, तेली, कुनबी, साडी और िनगर जैसे नपछडे वगों से आया था। सत्यशोधक समाि के दो प्राथवमक उद्दे श्य थे -
a) वशिा का प्रसार, और b) समाि सेिा। फुले के कायों, विशेि रूप से "सािचिवनक सत्यधमच" (िनता का सत्य) और
"गुलामवगरी" (गुलामी) का भारतीय समाि पर गहरा प्रभाि पड़ा है , िो आम िनता, विशेिकर वनर्ली िावतयोां के
लोगोां के वलए प्रेरणा स्रोत के रूप में काम कर रहे हैं ।
युग्म 3 सही सुमेवलत है : प्राथणना समाज (1867) की स्थापना आत्माराम पां डुरं ग ने बॉम्बे में की थी। इसका उद्दे श्य एकेिरवाद
को बढ़ावा दे ना, मनहलाओं की ल्कस्थनत को ऊपर उठाना, जानतगत भेदभाव का उन्मूलन और पूजा और सामानजक सुिार के
माध्यम से िानमणक रूनढ़वाद को चुनौती दे ना था। प्राथणना समाज से जुडे अन्य प्रमुख व्यल्कित्व थे:
केशि र्ांद्र सेन: उन्होंने आत्माराम पां डुरं ग को बॉम्बे में प्राथणना समाज स्थानपत करने में मदद की।
महादे ि गोवबांद रानाडे : िह 1870 में सांगठन में शावमल हुए। उनके समपणण और प्रयासों ने समाज की पहुं च को व्यापक
बनाने और इसे दे शव्यापी उपल्कस्थनत के साथ एक आं दोलन में बदलने में महत्वपूणण भूनमका ननभाई|
युग्म 4 सही सुमेवलत है : ई.िी. रामास्वामी नायकर, विन्ें पेररयार के नाम से िाना िाता है , ने 1925 में तवमलनाडु में
आत्म-सम्मान आां दोलन की थथापना की। पेररयार ने गैर-िाह्मणों को सशि बनाने, उनकी द्रनवड नवरासत पर जोर दे ने और
िाह्मणवादी प्रभुत्व को चुनौती दे ने के नलए आं दोलन शुरू नकया। इस आं दोलन का उद्दे श्य समानता और आत्म-सम्मान पर जोर
दे ते हुए लोगों को जानत के आिार पर एकजुट करना था।
युग्म 5 सही सुमेवलत है : 1820 के दशक के अांत और 1830 के दशक की शुरुआत में हे नरी विवियन डे रोवियो के
ने तृत्व में यांग बांगाल आां दोलन ने वहां दू कॉले ि में छात्रोां के बीर् स्वतांत्रता और समानता के विर्ारोां को बढािा वदया। इस
आं दोलन ने मनहलाओं के अनिकारों और नशक्षा सनहत सामानजक सुिारों की वकालत की। हालााँ नक, सामानजक पररल्कस्थनतयों
और जनता के साथ जुडाव की कमी के कारण इसे व्यापक समथणन या स्थायी प्रभाव हानसल करने के नलए संघषण करना पडा।
बहरहाल, उनके प्रयासों ने बंगाल में भनवष्य के सुिार आं दोलनों की नींव रखी।
Source: https://pib.gov.in/PressReleaseIframePage.aspx?PRID=1916229
https://egyankosh.ac.in/bitstream/123456789/20645/1/Unit-13.pdf
Spectrum’s A Brief History of Modern India

Q.7)
Ans) b
Exp) विकल्प b सही उत्तर है।
गौतम बुद्ध, नजन्हें नसद्धाथण गौतम के नाम से भी जाना जाता है , 566 से 486 ईसा पूवण तक जीनवत रहे । उनका जन्म नेपाल के
लुल्कम्बनी में हुआ था और बाद में वे आध्याल्कत्मक गुरु बन गये। बुद्ध की नशक्षाएाँ चार आयण सत्यों पर आिाररत हैं , जो दु ख की
प्रकृनत और इसे कैसे दू र नकया जाए, और अष्टां नगक पथ की व्याख्या करते हैं , जो व्यल्कियों को नैनतक आचरण, सचेतनता और
आं तररक शां नत के जीवन की ओर मागणदशणन करते हैं ।
विकल्प 1 सही है : वबक्तिसार: िह गौतम बुद्ध के समकालीन थे और उन्ोांने लगभग 543 ईसा पूिच से 492 ईसा पूिच तक
मगध साम्राज्य पर शासन वकया था। नबल्कम्बसार बौद्ध िमण के शुरुआती समथणक थे और गौतम बुद्ध के साथ उनकी कई
बातचीत हुई थीं।

Forum Learning Centre: Delhi - 2nd Floor, IAPL House, 19 Pusa Road, Karol Bagh, New Delhi - 110005 | Patna - 2nd floor, AG Palace, E Boring Canal
Road, Patna, Bihar 800001 | Hyderabad - 1st & 2nd Floor, SM Plaza, RTC X Rd, Indira Park Road, Jawahar Nagar, Hyderabad, Telangana 500020
9311740400, 9311740900 | https://academy.forumias.com | admissions@forumias.academy | helpdesk@forumias.academy

[7]
PTS 2024 | Test Code: 121405 - Solutions |

विकल्प 2 सही है : अिातशत्रु: िह वबांवबसार का पुत्र था और उसने लगभग 492 ईसा पूिच से 460 ईसा पूिच तक मगध पर
शासन वकया था। अजातशत्ु भी गौतम बुद्ध के समकालीन थे और बौद्ध ग्रंथों में बुद्ध के साथ उनकी बातचीत का उल्लेख
नकया गया है ।
विकल्प 3 गलत है : महापद्म नांद: वह गौतम बुद्ध के समकालीन नहीं थे। वह नंद वंश से थे और चौथी शताब्दी ईसा पूवण में
मगि पर शासन नकया था, जो गौतम बुद्ध के समय के बाद है ।
विकल्प 4 सही है : प्रसेनवित: वह कोशल राज्य का शासक था, नजसकी राजिानी श्रावस्ती थी। प्रसेननजत गौतम बुद्ध के
समकालीन थे और उनके साथ उनकी महत्वपूणण बातचीत हुई थी। उन्होंने लगभग 6ठी शताब्दी ईसा पूवण से 5वीं शताब्दी ईसा
पूवण के मध्य तक शासन नकया।
विकल्प 5 गलत है : वबांदुसार: वह चंद्रगुप्त मौयण का पुत् था और उसके बाद मौयण साम्राज्य का शासक बना। नबन्दु सार का
शासनकाल लगभग 298 ईसा पूवण शुरू हुआ और लगभग 273 ईसा पूवण तक जारी रहा, और इस प्रकार, वह गौतम बुद्ध के
समकालीन नहीं थे।
Source: https://egyankosh.ac.in/bitstream/123456789/64798/1/BLOCK%204.pdf
https://www.worldhistory.org/Ajatashatru/#:~:text=Both%20the%20Jain%20and%20Buddhist%20sources
%20claim%20that%20Ajatashatru%20followed,of%20Magadha%20at%20that%20time।
http://www.buddhanet.net/e-learning/buddhism/lifebuddha/2_14lbud.htm
https://egyankosh.ac.in/bitstream/123456789/35229/1/Unit-3.pdf

Q.8)
Ans) c
Exp) विकल्प c सही उत्तर है ।
कथन 1 सही है:
ऐहोल वशलालेख
थथान: एहोल, हुं गुंड तालुक, बागलकोट नजला, कनाण टक में मेगुती जैन मंनदर।
यह नशलालेख बताता है नक बादामी के चालुक् नकतने शल्किशाली थे और कनाण टक पर उनका कब्जा था। यह वंशावली और
राजा पुलकेनशन नद्वतीय के राजनीनतक कारनामों का नवस्तृत नववरण दे ता है , नजसमें नए क्षेत्ों पर कब्जा करके, सम्राट हषण की
हार सनहत नए सामंत बनाए गए।
अंत में इसमें रनवकीनतण द्वारा नजनेन्द्र मल्किर का ननमाण ण अंनकत है ।
कथन 2 सही है:
िूनागढ वशलालेख
थथान: जूनागढ़, गुजरात।
इस वशलालेख को सबसे पुराना ज्ञात सांस्कृत स्तिन माना िाता है । इसमें सुदशणन झील के इनतहास, उसके मरम्मत कायण
का उल्लेख है और राजा रुद्रदामन के कररयर और उपलल्कब्धयों पर प्रकाश डाला गया है ।
कथन 3 गलत है :
इलाहाबाद वशलालेख
थथान: प्रयागराज, उत्तर प्रदे श
यह नशलालेख गुप्त राजवंश के सम्राट के रूप में समुद्रगुप्त की उपलल्कब्धयों पर प्रकाश डालता है । इलाहाबाद स्तंभ नशलालेख
की रचना समुद्रगुप्त के दरबार के कनव हररसेन ने की थी। यह नशलालेख शुद्ध संस्कृत भाषा में नलखा गया है (प्राकृत भाषा में
नहीं)। यह राजा की सैन्य सफलता और अनुकरणीय नेतृत्व पर प्रकाश डालता है , उन्हें एक आदशण राजा के रूप में नचनत्त
करता है ।
कथन 4 सही है:
हाथीगुम्फा वशलालेख

Forum Learning Centre: Delhi - 2nd Floor, IAPL House, 19 Pusa Road, Karol Bagh, New Delhi - 110005 | Patna - 2nd floor, AG Palace, E Boring Canal
Road, Patna, Bihar 800001 | Hyderabad - 1st & 2nd Floor, SM Plaza, RTC X Rd, Indira Park Road, Jawahar Nagar, Hyderabad, Telangana 500020
9311740400, 9311740900 | https://academy.forumias.com | admissions@forumias.academy | helpdesk@forumias.academy

[8]
PTS 2024 | Test Code: 121405 - Solutions |

थथान: उदयनगरर पहानडयााँ , भुवनेिर, ओनडशा


इस वशलालेख में महामेघिाहन िांश के रािा खारिेल के कारनामोां और शासनकाल का वििरण वदया गया है । यह
उनकी सैन्य जीत, वास्तुनशल्प योगदान, िानमणक संरक्षण और कनलंग में सां स्कृनतक और िानमणक गनतनवनियों को पुनजीनवत करने
के प्रयासों का वणणन करता है ।
ज्ञान का आधार: श्रिणबेलगोला वशलालेख और िैन वकांिदां वतयााँ एक गांभीर अकाल के बारे में महत्वपूणच िानकारी
प्रदान करती हैं , नजसके कारण जैन समुदाय का उस क्षेत् में प्रवास हुआ, नजसे अब कनाण टक के नाम से जाना जाता है ।
Source: 1. https://www.deccanherald.com/india/karnataka/aiholes-stories-stone-2475388
2.
https://epgp.inflibnet.ac.in/epgpdata/uploads/epgp_content/S000829IC/P001689/M022045/ET/15040725
96P08-M29-JunagadhInscriptionofRudradaman-ET.pdf
3.
https://epgp.inflibnet.ac.in/epgpdata/uploads/epgp_content/S000829IC/P001689/M022047/ET/15040727
12P08-M33-AllahabadPillarInscriptionofSamudragupta-ET.pdf
4.
https://epgp.inflibnet.ac.in/epgpdata/uploads/epgp_content/S000829IC/P001689/M020089/ET/14932888
74P08-M28-HathigumphaInscriptionofKharavela-ET.pdf

Q.9)
Ans) c
Exp) विकल्प c सही उत्तर है ।
सेनगोल' को नए संसद भवन में स्थानपत नकया गया है और भारत सरकार इसे दे श की संप्रभुता और सांस्कृनतक नवरासत का
प्रतीक मानती है ।
कथन a गलत है : 'सेन्गोल' एक सुनहरी तलिार नही ां है , बक्ति यह एक छड़ी है विसे रािा या रानी अपनी शक्तक्त के
प्रतीक के रूप में औपर्ाररक अिसरोां के दौरान ले िाते हैं । इसकी उत्पनत्त चोल राजवंश (चेरा नही ं) से हुई है ।
कथन b गलत है : 'सेनगोल' शब्द तवमल शब्द 'सेम्माई', विसका अथच धावमचकता है , और 'कोल', विसका अथच रािदां ड
है , से वलया गया है ।
कथन c सही है : सांगम सावहत्य 'वसलापवथकारम' में स्पष्ट रूप से 'सेनगोल' शब्द का उल्लेख है । नसलापनथकरम के
अनुसार, पां ड्य राजा ने अपने सेनगोल की सीिी ल्कस्थनत को बहाल करने के नलए अपने जीवन का बनलदान नदया, जो एक आम
आदमी के प्रनत अन्याय के कारण झुक गया था, नजसके पररणामस्वरूप अंततः उस व्यल्कि की मृत्यु हो गई।
कथन d गलत है : 1857 के विद्रोह के दौरान सेनगोल को बहादु र शाह िफर के सामने पेश नही ां वकया गया था। निनटश
शासन से भारत की आजादी के बाद, वायसराय लॉडण माउं टबेटन ने सत्ता के औपचाररक हस्तां तरण के संबंि में जवाहरलाल
नेहरू के सामने एक महत्वपूणण प्रश्न रखा था। सी. राजगोपालाचारी से सलाह लेते हुए, नेहरू ने चोल परं परा को अपनाया।
सनदयों पुराने शैव मठ नतरुववदु थुराई अिीनम ने नेहरू को 'सेनगोल' भेंट नकया, जो निनटश भारत से सत्ता पररवतणन का प्रतीक
था।
Source:
https://pib.gov.in/PressReleasePage.aspx?PRID=1926883#:~:text=has%20the%20%E2%80%9Corder%E2%
80%9D%20(%E2%80%9C-,Aanai,-%E2%80%9D%20in%20Tamil)%20to
https://indianexpress.com/article/explained/explained-history/sengol-new-parliament-significance-nehru-
8626977/

Forum Learning Centre: Delhi - 2nd Floor, IAPL House, 19 Pusa Road, Karol Bagh, New Delhi - 110005 | Patna - 2nd floor, AG Palace, E Boring Canal
Road, Patna, Bihar 800001 | Hyderabad - 1st & 2nd Floor, SM Plaza, RTC X Rd, Indira Park Road, Jawahar Nagar, Hyderabad, Telangana 500020
9311740400, 9311740900 | https://academy.forumias.com | admissions@forumias.academy | helpdesk@forumias.academy

[9]
PTS 2024 | Test Code: 121405 - Solutions |

Q.10)
Ans) b
Exp) विकल्प b सही उत्तर है
विकल्प 1 सही है : स्यादिाद एक प्रमुख िैन दशचन है , िो सांदभच और पररप्रेक्ष्य के आधार पर सत्य की बहुलता की
िकालत करता है । स्यादिाद का सुझाव है नक वास्तनवकता की जनटलता को एक ही र्दनष्टकोण से पूरी तरह से नहीं समझा जा
सकता है । यह इस नवचार को रे खां नकत करता है नक सत्य एकवचन नहीं है बल्कि नवनभन्न र्दनष्टकोणों से दे खने पर नभन्न हो सकता
है । यह मानवीय ज्ञान की सीमाओं और सत्य की नवनविता को स्वीकार करके सनहष्णुता और खुले नदमाग को बढ़ावा दे ता है ।
विकल्प 2 गलत है : शून्यिाद एक बौद्ध दशचन है िो "शून्यता" या "शून्यता" की अिधारणा पर केंवद्रत है । इस वसद्धाांत
को विशेि रूप से नागािुचन द्वारा महायान बौद्ध धमच के माध्यवमक विद्यालय में उिागर वकया गया है । यह मानता है नक
सभी घटनाओं में अंतननणनहत प्रकृनत, सार या स्वयं का अभाव है । यह दावा करता है नक अल्कस्तत्व को अन्योन्यानश्रतता और
अपररवतणनीय पहचान की अनुपल्कस्थनत से पररभानषत नकया गया है । शून्यवाद को बढ़ावा दे ने के बजाय शून्यवाद दु ख से मुल्कि के
नलए शून्यता को आवश्यक मानता है । यह नननित िारणाओं से अलगाव और आत्मज्ञान की ओर ले जाने पर जोर दे ता है ।
विकल्प 3 सही है : अनेकाांतिाद एक िैन दशचन है िो इस धारणा पर िोर दे ता है वक सत्य और िास्तविकता बहुआयामी
हैं और उन्ें एक ही दृवष्टकोण से पूरी तरह से नही ां पकड़ा िा सकता है । "गैर-ननरपेक्षता" की यह अविारणा बताती है नक
समझ हमेशा आं नशक होती है और नवनवि र्दनष्टकोणों को स्वीकार करने से सनहष्णुता और संवाद को बढ़ावा नमलता है । यह
स्यादवाद के साथ ननकटता से जुडा हुआ है , जो बताता है नक इन नवनवि सत्यों को सशतण रूप से कैसे व्यि नकया जाए।
विकल्प 4 गलत है : सिाचक्तस्तिाद एक बौद्ध दशचन है जो क्षनणक अल्कस्तत्व के पारं पररक बौद्ध र्दनष्टकोण से नभन्न अतीत, वतणमान
और भनवष्य में घटनाओं के अल्कस्तत्व की वकालत करता है । यह अनभिमण ग्रंथों पर केंनद्रत है और वास्तनवकता और तत्वमीमां सा
का नवस्तृत दाशणननक नवश्लेषण प्रदान करता है ।
Source: https://ncert.nic.in/pdf/publication/otherpublications/tioi101.pdf
https://egyankosh.ac.in/bitstream/123456789/34656/1/Unit-3.pdf
https://egyankosh.ac.in/bitstream/123456789/35228/1/Unit-2.pdf

Q.11)
Ans) a
Exp) विकल्प a सही उत्तर है ।
विकल्प 1 और 3 सही हैं और विकल्प 2, 4 और 5 ग़लत हैं : छठी शताब्दी ईसा पूवण से भारत का राजनीनतक इनतहास
गणतंत् और राजशाही राज्यों के बीच संघषण का इनतहास है । मल्ल, कुरु, कांभोि, गाांधार, शाक्य और िृक्ति आवद प्रमुख
गणतांत्र राज्य थे । गणतां नत्क राज्य नहमालय की तलहटी और गंगा बेनसन के उत्तरपूवी भाग तक ही सीनमत थे। मगध, अांग,
अिांती, काशी, कोसल, ित्स आनद शल्किशाली राजशाही राज्य थे। वे गंगा के बेनसन तक ही सीनमत थे - जो उत्तर भारत का
सबसे उपजाऊ क्षेत् है । गंगा का बेनसन गणतंत् और राजशाही राज्यों के बीच एक बडा नववाद बन गया। मगि साम्राज्य ने
हनथयारों और कूटनीनत का उपयोग करके अन्य राज्यों को सफलतापूवणक दबा नदया।
Source: https://ncert.nic.in/ncerts/l/fess106.pdf
https://egyankosh.ac.in/bitstream/123456789/22239/5/Unit-7.pdf
https://cbc.gov.in/cbcdev/mahajanpadas/mahajanpadas.html#:~:text=The%20Mahajanapadas%20were%2
0mostly%20monarchical,Gana%20or%20Ganasanghas%2C%20or%20Ganarajya.
https://epgp.inflibnet.ac.in/epgpdata/uploads/epgp_content/S000829IC/P000927/M014493/ET/14573463
12ET08.pdf

Forum Learning Centre: Delhi - 2nd Floor, IAPL House, 19 Pusa Road, Karol Bagh, New Delhi - 110005 | Patna - 2nd floor, AG Palace, E Boring Canal
Road, Patna, Bihar 800001 | Hyderabad - 1st & 2nd Floor, SM Plaza, RTC X Rd, Indira Park Road, Jawahar Nagar, Hyderabad, Telangana 500020
9311740400, 9311740900 | https://academy.forumias.com | admissions@forumias.academy | helpdesk@forumias.academy

[10]
PTS 2024 | Test Code: 121405 - Solutions |

Q.12)
Ans) b
Exp) विकल्प b सही उत्तर है।
दक्कन सल्तनत इिामी राज्यों का एक समूह था जो मध्यकाल के दौरान दनक्षणी भारत के दक्कन क्षेत् में उभरा। प्रमुख
राजवंश थे अहमदनगर (ननजाम शाही वंश), बरार (इमाद शाही वंश), बीदर (बरीद शाही वंश), बीजापुर (आनदल शाही वंश) और
गोलकुंडा (कुतुब शाही वंश)।
कथन 1 सही है : दखानी भाषा का नवकास सानहत्य के क्षेत् में गोलकुंडा सल्तनत के सबसे महत्वपूणण योगदान का प्रतीक है ।
इिानहम कुली कुतुब शाह ने तेलुगु सानहत्य को संरक्षण नदया। मुहम्मद कुली कुतुब शाह ने दखानी, फारसी और तेलुगु में नलखा
और दखानी में एक व्यापक दीवान (खंड) छोडा, नजसे कुल्कल्लयात-ए-मोहम्मद कुली कुतुब शाह के नाम से जाना जाता है ।
कथन 2 गलत है : वबदरी वशल्प धातुकमच से िुड़ा था। इसकी उत्पवत्त बीदर सल्तनत के िेत्र में हुई थी। यह एक प्रकार
की दवमश्क कला है , नजसमें वस्तुओं को सजाने के नलए तां बे और जस्ता के नमश्र िातु में सोना, चां दी या पीतल को शानमल
करना शानमल है ।
नबदरी मेटलवकण

कथन 3 सही है : गोल गुिि, एक शानदार मकबरा, बीिापुर सल्तनत के शासकोां के अधीन बनाया गया था। इसमें
आनदल शाही वंश के सातवें सुल्तान मोहम्मद आनदल शाह के अवशेष हैं । यह दु ननया की सबसे बडी एकल-कक्षीय संरचनाओं
में से एक है ।

गोल गुम्बज
Source: https://www.newworldencyclopedia.org/entry/Deccan_sultanates
https://egyankosh.ac.in/bitstream/123456789/20228/1/Unit-9.pdf

Forum Learning Centre: Delhi - 2nd Floor, IAPL House, 19 Pusa Road, Karol Bagh, New Delhi - 110005 | Patna - 2nd floor, AG Palace, E Boring Canal
Road, Patna, Bihar 800001 | Hyderabad - 1st & 2nd Floor, SM Plaza, RTC X Rd, Indira Park Road, Jawahar Nagar, Hyderabad, Telangana 500020
9311740400, 9311740900 | https://academy.forumias.com | admissions@forumias.academy | helpdesk@forumias.academy

[11]
PTS 2024 | Test Code: 121405 - Solutions |

Q.13)
Ans) a
Exp) विकल्प a सही उत्तर है ।
नलंगायतवाद, नजसे वीरशैववाद के नाम से भी जाना जाता है , बारहवीं शताब्दी सी.ई. में बासवन्ना (1134 - 1196 सी.ई.) द्वारा
भारत में एक सुिार आं दोलन के रूप में शुरू हुआ।
कथन 1 गलत है : वलांगायतिाद िावत व्यिथथा को अस्वीकार करता है और वलांग पहनने िाले सभी लोगोां को समान
घोवित करता है । वे पुनजणन्म में नविास नहीं करते हैं और पररणामस्वरूप, उन्होंने कमण के नसद्धां त को त्याग नदया है (यह
नसद्धां त नक एक जीवन में कमण बाद के अवतारों की प्रकृनत ननिाण ररत करते हैं )।
कथन 2 गलत है : सांत अप्पार और मवणक्किसागर तवमलनाडु में शैि धमच के भीतर नयनार सांत थे , कनाचर्क में
वलांगायत आां दोलन के नही ां। नलंगायतवाद में संतों और दाशणननकों की अपनी श्रृंखला है , नजनमें बसवा, अल्लामा प्रभु और
अक्का महादे वी शानमल हैं ।
कथन 3 सही है : वलां गायत दाह सांस्कार की वहां दू परां परा का पालन नही ां करते हैं । इसके बजाय, वे अपने मृतकों को
दफनाते हैं । वे अपने पूवणजों के नलये कमणकाण्ड नहीं करते। वे अपने मृतकों को दफनाते हैं , शव को कि में िॉस-लेग्ड बैठाते हैं
(दानहने हाथ में अपना व्यल्किगत नलंग पकडे हुए ध्यान मुद्रा में)।
ज्ञानधार:
नयनार र्ार शैि सांत हैं िो लगभग 1,200 साल पहले तवमलनाडु में रहते थे । दोनों ने भल्कि गीतों की रचना की जो आज
भी सत्संगों और मंनदरों में गाए जाते हैं ।
उनके नाम अप्पार, सुंदरर, संबंदर और मनणक्कवसागर हैं ।
Source: https://ncert.nic.in/ncerts/l/gess108.pdf
https://lingayatkoshi.co.in/lingayatdetails.php
https://www.himalayanacademy.com/view/bd_2018-01-17_path-to-siva_lesson-39_commentary

Q.14)
Ans) c
Exp) विकल्प c सही उत्तर है ।
करार्ी प्रस्ताि भारतीय राष्टरीय काांग्रेस द्वारा 1931 के करार्ी सत्र में पाररत वकया गया था। कां ग्रेस द्वारा कल्पना की गई
स्वराज में जनता की वास्तनवक आनथणक स्वतंत्ता शानमल होनी चानहए।
विकल्प 1, 2, 3 और 5 सही हैं : 1931 के करार्ी सत्र में , काांग्रेस ने घोिणा की वक कोई भी सांविधान उसे तब तक
स्वीकायच नही ां होगा जब तक नक वह स्वराज सरकार को ननम्ननलल्कखत प्राविान प्रदान करने या सक्षम करने में सक्षम न हो:
1. अनभव्यल्कि, संघ और बैठक की स्वतंत्ता|
2. िमण की स्वतंत्ता।
3. सभी संस्कृनतयों और भाषाओं का संरक्षण।
4. कानून के समक्ष सभी नागररक समान होंगे।
5. िमण, जानत या नलंग के कारण रोजगार या व्यापार या पेशे में कोई नवकलां गता नही ं।
6. सावणजननक कुओं, स्कूलों आनद के संबंि में सभी के नलए समान अनिकार और कतणव्य।
7. सभी को ननयमों के अनुसार हनथयार रखने का अनिकार है ।
8. कानून के अनुसार नकसी भी व्यल्कि को संपनत्त या स्वतंत्ता से वंनचत नही ं नकया जाएगा।
9. राज्य की िानमणक तटस्थता|
10. वयस्क मतानिकार|
11. ननः शुि अननवायण प्राथनमक नशक्षा।
12. कोई उपानि प्रदान नही ं की जाएगी|

Forum Learning Centre: Delhi - 2nd Floor, IAPL House, 19 Pusa Road, Karol Bagh, New Delhi - 110005 | Patna - 2nd floor, AG Palace, E Boring Canal
Road, Patna, Bihar 800001 | Hyderabad - 1st & 2nd Floor, SM Plaza, RTC X Rd, Indira Park Road, Jawahar Nagar, Hyderabad, Telangana 500020
9311740400, 9311740900 | https://academy.forumias.com | admissions@forumias.academy | helpdesk@forumias.academy

[12]
PTS 2024 | Test Code: 121405 - Solutions |

13. मृत्युदंड समाप्त नकया जाए।


14. भारत के प्रत्येक नागररक के नलए आवागमन की स्वतंत्ता और बसने और संपनत्त अनजणत करने का अनिकार|
15. उसके नकसी भी भाग में, और कानून का समान संरक्षण।
16. औद्योनगक श्रनमकों के नलए उनचत जीवन स्तर और ननपटान के नलए उपयुि मशीनरी|
17. ननयोिाओं और श्रनमकों के बीच नववाद और बुढ़ापे , बीमारी आनद से सुरक्षा।
18. सभी श्रम दास प्रथा की ल्कस्थनतयों से मुि हों।
19. मनहला श्रनमकों की नवशेष सुरक्षा|
20. बच्चों को खदानों और कारखानों में ननयोनजत नहीं नकया जाएगा।
21. नकसानों और श्रनमकों को यूननयन बनाने का अनिकार।
22. भू-राजस्व तथा पट्टे दारी एवं लगान की व्यवस्था में सुिार, लगान एवं लगान में छूट|
23. अलाभकारी जोत के नलए और छोटी जोत के नलए दे य दे य रानश में कमी के नलए।
24. स्नातक पैमाने पर नवरासत कर|
25. सैन्य व्यय को कम से कम आिा कर दे ना।
26. राज्य के नकसी भी कमणचारी को सामान्यतः 500 रुपये प्रनत माह से अनिक वेतन नहीं नदया जाएगा।
27. नमक कर का उन्मूलन|
28. नवदे शी वस्त्ों की प्रनतस्पिाण से स्वदे शी वस्त्ों की सुरक्षा।
29. नशीले पेय और नशीली दवाओं पर पूणण प्रनतबंि।
30. मुद्रा एवं नवननमय राष्टरीय नहत में।
31. प्रमुख उद्योगों और सेवाओं, रे लवे आनद का राष्टरीयकरण।
32. कृनष ऋण से मुल्कि एवं सूदखोरी पर ननयंत्ण।
33. नागररकों के नलए सैन्य प्रनशक्षण| नवकल्प 4 गलत है : भारतीय स्वतंत्ता संग्राम के दौरान अल्पसंख्यकों के नलए अलग
ननवाण चन क्षेत् की मां ग एक नववादास्पद मुद्दा था। कराची प्रस्ताव में अल्पसंख्यकों के नलए अलग ननवाण नचका का उल्लेख
नहीं था।
Source: https://ncert.nic.in/ncerts/l/lesy203.pdf

Q.15)
Ans) b
Exp) विकल्प b सही उत्तर है।
युग्म 1 गलत सुमेवलत है : उिा मेहता एक गाांधीिादी और स्वतांत्रता सेनानी थी ां, विन्ोांने 1942 के भारत छोड़ो आां दोलन
के दौरान काांग्रेस रे वडयो (विसे गुप्त काांग्रेस रे वडयो भी कहा िाता है ) का आयोिन वकया था। वह संनविान सभा की
सदस्य नहीं थीं। संनविान सभा की कुछ प्रमुख मनहला सदस्य सरोनजनी नायडू, सुचेता कृपलानी और नवजयलक्ष्मी पंनडत थीं।
युग्म 2 सही सुमेवलत है : श्यामिी कृष्ण िमाच ने लांदन में इां वडयन होम रूल सोसाइर्ी, इां वडया हाउस और द इां वडयन
सोवशयोलॉविस्ट की थथापना की। उन्होंने दो अलग-अलग अवनियों के दौरान "द इं नडयन सोनशयोलॉनजस्ट" पनत्का के
संपादक के रूप में कायण नकया: 1905 से 1914 तक और नफर 1920 और 1922 के बीच। पनत्का के पन्नों पर अक्सर भारत में
निनटश शासन द्वारा लगाए गए अन्याय की आलोचना होती थी।
युग्म 3 सही सुमेवलत है : राधाकाांत दे ब, धमच सभा (1830) के सांथथापक थे , एक ऐसा समाि िो सामाविक-धावमचक
मामलोां में यथाक्तथथवत बनाए रखने का समथचन करता था और सती प्रथा के उन्मूलन का नवरोि करता था। रािाकां त दे ब
नहं दुओं के बीच नशक्षा, नवशेषकर अंग्रेजी नशक्षा को बढ़ावा दे ने में सनिय रूप से शानमल थे।
Source: Spectrum

Forum Learning Centre: Delhi - 2nd Floor, IAPL House, 19 Pusa Road, Karol Bagh, New Delhi - 110005 | Patna - 2nd floor, AG Palace, E Boring Canal
Road, Patna, Bihar 800001 | Hyderabad - 1st & 2nd Floor, SM Plaza, RTC X Rd, Indira Park Road, Jawahar Nagar, Hyderabad, Telangana 500020
9311740400, 9311740900 | https://academy.forumias.com | admissions@forumias.academy | helpdesk@forumias.academy

[13]
PTS 2024 | Test Code: 121405 - Solutions |

https://amritmahotsav.nic.in/district-reopsitory-
detail.htm?23948#:~:text=Shyamji%20Krishna%20Verma%20served%20as,by%20British%20rule%20in%20
India.
https://amritmahotsav.nic.in/district-reopsitory-detail.htm?5176

Q.16)
Ans) d
Exp) विकल्प d सही उत्तर है।
विकल्प a गलत है | पाल िांश के धमचपाल (770-815 ई.) अपने वपता गोपाल प्रथम के उत्तरावधकारी बने। उन्ोांने बांगाल
और वबहार को अपने शासन में लाया और एक नामाांवकत व्यक्तक्त के माध्यम से कनौि पर भी अपना अवधकार बढाया।
िमणपाल ने परमेिर और महाराजानिराज जैसी उपानियााँ अनजणत कीं, जो उनके अनिकार का प्रतीक थीं। बौद्ध िमण के कट्टर
समथणक, उन्होंने भागलपुर में नविमनशला मठ की स्थापना की। इसके अनतररि, उन्होंने वतणमान बां ग्लादे श के सोमापुरा में भव्य
नवहार का ननमाण ण कराया।
विकल्प b गलत है | पाल वंश का महीपाल प्रथम, नवग्रामपाल नद्वतीय का पुत् था। उनके काल की सबसे महत्वपूणण घर्ना 1020
और 1025 ई.पू. के बीर् रािेंद्र र्ोल द्वारा उत्तरी भारत पर आिमण था। हालााँ नक, गंगा से परे चोलों की प्रगनत को मनहपाल
प्रथम द्वारा रोका गया था।
विकल्प c गलत है | पाल िांश के रामापाल ने राििांश के खोए हुए गौरि को पुनः प्राप्त करने का प्रयास वकया। उनकी
मृत्यु के बाद, पाल राजवंश की उपल्कस्थनत केवल मगि (नबहार) के एक नहस्से तक ही सीनमत रही और केवल थोडे समय के नलए
अल्कस्तत्व में रही।
विकल्प d सही है | दे िपाल पाल िांश के धमचपाल का पुत्र था। उसने पूिच की ओर पाल प्रभाि का विस्तार वकया, िो
कामरूप (असम) तक पहुां र् गया। उन्होंने उन सभी राज्यों के साथ गठबंिन करके राष्टरकूट शासक अमोघवषण को हराया, जो
राष्टरकूटों को अपना सामान्य शत्ु मानते थे। बौद्ध िमण के प्रबल समथणक दे वपाल ने सुवणणदीप के राजा बालापुत्दे व को नालंदा में
एक मठ चलाने के नलए जमीन दी थी। दे वपाल का उत्तरानिकारी नवग्रामपाल हुआ।
Source: TNSCERT-class 11- Chapter-Harsha and the rise of regional kingdoms

Q.17)
Ans) d
Exp) विकल्प d सही उत्तर है।
र्ांदािर की लड़ाई 1194 में गोर के मुहम्मद और गहड़िाला िांश के ियर्ांद्र के बीर् हुई थी। यह ऩिरोजाबाद के पास
चंदावर (आिुननक चंदावल) में , आगरा के पास यमुना नदी पर हुआ था।
कथन I गलत है । 1194 ई. में लड़ी गई र्ांदािर की लड़ाई ने भारतीय इवतहास में एक महत्वपूणच घर्ना को वर्वित वकया
क्ोंनक इसने उत्तरी भारत में मोहम्मद गोरी के तहत तुकी शासन (तैमूर शासन नहीं) की स्थापना को मजबूत नकया।
कथन II सही है । ग़ोरी सेना का नेतृत्व करते हुए मोहम्मद गोरी ने र्ांदािर की लड़ाई में कन्नौि के शासक िय र्ांद को
हराया।
Source: History of Medieval India by Satish Chandra.
Ch:5. The Age of Conflict (Circa 1000–1200)
https://egyankosh.ac.in/bitstream/123456789/61926/1/Unit-12.pdf

Q.18)
Ans) d
Exp) विकल्प d सही उत्तर है।

Forum Learning Centre: Delhi - 2nd Floor, IAPL House, 19 Pusa Road, Karol Bagh, New Delhi - 110005 | Patna - 2nd floor, AG Palace, E Boring Canal
Road, Patna, Bihar 800001 | Hyderabad - 1st & 2nd Floor, SM Plaza, RTC X Rd, Indira Park Road, Jawahar Nagar, Hyderabad, Telangana 500020
9311740400, 9311740900 | https://academy.forumias.com | admissions@forumias.academy | helpdesk@forumias.academy

[14]
PTS 2024 | Test Code: 121405 - Solutions |

पाररल्कस्थनतकी में संगठन का सही िम है -


एकल - इसमें ऑगेनेल या अन्य भागों से बना एक शरीर शानमल है जो जीवन की नवनभन्न प्रनियाओं को पूरा करने के नलए एक
साथ काम करते हैं । उदाहरण के नलए- पौिे , जानवर, कवक, बैक्टीररया आनद
िनसांख्या- यह एक ही प्रजानत के व्यल्कियों का एक समूह है , जो एक दू सरे के साथ प्रजनन करने में सक्षम हैं ।
समुदाय- इसे एक ही भौगोनलक क्षेत् के भीतर दो या दो से अनिक प्रजानतयों की आबादी के रूप में पररभानषत नकया गया है ।
पाररक्तथथवतकी तांत्र- यह अपने अजैनवक घटकों जैसे जल और वायु सनहत एक समुदाय है ।
बायोम- यह समान जलवायु पररल्कस्थनतयों वाला एक बडा पाररल्कस्थनतक क्षेत् है नजसके भीतर कई पाररल्कस्थनतकी तंत् हो सकते हैं ।
िीिमांडल: यह पाररल्कस्थनतकी में संगठन का अंनतम स्तर है ।
Source: https://ncert.nic.in/ncerts/l/lebo113.pdf
ForumIAS Environment Red book

Q.19)
Ans) c
Exp) विकल्प c सही उत्तर है ।
प्रिानमंत्ी उच्चतर नशक्षा अनभयान (पीएम-यूएसएचए) दे श में उच्च नशक्षा की समग्र गुणवत्ता में सुिार के नलए नशक्षा मंत्ालय
द्वारा शुरू नकया गया एक कायणिम है । यह 2013 में शुरू नकए गए राष्टरीय उच्चतर नशक्षा अनभयान (आरयूएसए) कायणिम का
उत्तरानिकारी है । इसका उद्दे श्य उच्च नशक्षा को अनिक सुलभ, न्यायसंगत और उत्कृष्ट बनाना है ।
• प्रदशणन और सुिारों के आिार पर राज्यों को नवत्तीय सहायता प्रदान करता है
• दू रदराज के क्षेत्ों, आकां क्षी नजलों और कम नामां कन वाले संस्थानों को लनक्षत करता है
विकल्प a सही है : इस योजना के नलए संस्थानों की नसफाररश राज्य सरकारों/राज्य उच्च नशक्षा पररषद (एसएचईसी) द्वारा की
जाएगी।
विकल्प b सही है : पीएम-यूएसएचए का एक मुख्य उद्दे श्य मौजूदा राज्य संचानलत उच्च शैक्षनणक संस्थानों (एचईआई) की
समग्र गुणवत्ता को बढ़ाना है । इसमें बुननयादी ढां चे, संकाय नवकास, पाठ्यिम आिुननकीकरण, अनुसंिान सुनविाओं और समग्र
सीखने के अनुभव में सुिार शानमल हैं ।
विकल्प c गलत है :
यह राज्योां और केंद्रशावसत प्रदे शोां के सरकारी और सरकारी सहायता प्राप्त सांथथानोां दोनोां पर ध्यान केंवद्रत कर रहा
है । यह गैर-मान्यता प्राप्त संस्थानों के प्रत्यायन को प्रोत्सानहत कर रहा है और राष्टरीय मूल्यां कन और प्रत्यायन पररषद (NAAC)
के माध्यम से प्रत्यायन में सुिार कर रहा है ।
• योजना के तहत फोकस नजलों को प्राथनमकता दी जाएगी
• राज्य ननम्ननलल्कखत संकेतकों के आिार पर फोकस नजलों की पहचान करें गे:
a. ननम्न जीईआर
b. मनहलाओं, टर ां सजेंडरों, एससी, एसटी और ओबीसी के नलए जनसंख्या अनुपात और नामां कन अनुपात
c. आकां क्षी/सीमावती क्षेत्/वामपंथी उग्रवाद प्रवण नजला
d. नलंग समानता
विकल्प d सही है : PM-USHA एक केंद्र प्रायोवित योिना है ।
फांवडां ग अनुपात: केंद्र प्रायोनजत योजना में केंद्र सरकार और राज्य/केंद्र शानसत प्रदे श सरकारें एक नवनशष्ट अनुपात में नवत्तीय
बोझ साझा करती हैं । यह अनुपात राज्य/केंद्रशानसत प्रदे श की श्रेणी के आिार पर नभन्न होता है
Source: https://pmush.education.gov.in/pm-
ush/#:~:text=Rashtria%20Uchchatar%20Shiksha%20Abhyan%20(RUSA,transparency%2C%20accountabilit
y%2C%20and%20responsiveness।

Forum Learning Centre: Delhi - 2nd Floor, IAPL House, 19 Pusa Road, Karol Bagh, New Delhi - 110005 | Patna - 2nd floor, AG Palace, E Boring Canal
Road, Patna, Bihar 800001 | Hyderabad - 1st & 2nd Floor, SM Plaza, RTC X Rd, Indira Park Road, Jawahar Nagar, Hyderabad, Telangana 500020
9311740400, 9311740900 | https://academy.forumias.com | admissions@forumias.academy | helpdesk@forumias.academy

[15]
PTS 2024 | Test Code: 121405 - Solutions |

Q.20)
Ans) b
Exp) विकल्प b सही उत्तर है।
कथन I सही है: यह कथन ऊष्मागवतकी और मानि शरीर विया विज्ञान के मूलभूत वसद्धाांत को दशाचता है । जब हमारा
शरीर अत्यनिक गमण हो जाता है , तो वे थमोरे ग्यूलेशन के सािन के रूप में पसीना उत्पन्न करते हैं । पसीना हमारी त्वचा की सतह
से वाल्कित हो जाता है , इस प्रनिया में हमारे शरीर से ऊष्मा ऊजाण को अवशोनषत करता है । यह ऊष्मा अवशोषण शरीर को
ठं डा करता है , नजससे आं तररक तापमान ल्कस्थर बनाए रखने में मदद नमलती है ।
िाष्पीकरण की दर, और इस प्रकार इस तांत्र के माध्यम से गमी के नुकसान की दर, आसपास के हिा के तापमान,
आद्रच ता और िायु आां दोलन िैसे पयाचिरणीय कारकोां पर वनभचर करती है । शुष्क हवा में सापेक्ष आद्रण ता कम होती है ,
नजसका अथण है नक हवा में पहले से ही कम नमी मौजूद है । ऐसी ल्कस्थनतयों में , हवा में अनतररि नमी को अवशोनषत करने की
अनिक क्षमता होती है , नजसमें हमारी त्वचा पर पसीने से आने वाली नमी भी शानमल है । पररणामस्वरूप, पसीना शुष्क हवा में
अनिक आसानी से वाल्कित हो जाता है , नजससे हमारे शरीर से गमी की हानन की दर तेज हो जाती है ।
कथन II सही है : गमच और नम हिा, गमच और शुष्क हिा की तुलना में अवधक गमच और असुविधािनक महसूस होती है ।
ऐसा इसनलए है क्ोंनक आद्रण पररल्कस्थनतयों में , हवा में उच्च नमी सामग्री पसीने के वािीकरण के माध्यम से शरीर की खुद को
ठं डा करने की क्षमता में बािा डालती है । पररणामस्वरूप, शुष्क पररल्कस्थनतयों की तुलना में आद्रण पररल्कस्थनतयों में शरीर अनिक
गमण और असहज महसूस करता है ।
हालााँवक, कथन II सीधे तौर पर कथन I की व्याख्या नही ां करता है । जबनक दोनों कथन गमी और आद्रण ता के पहलुओं पर
चचाण करते हैं , वे अलग-अलग घटनाओं को संबोनित करते हैं : कथन I गमी के नुकसान और हवा के सूखेपन के बीच संबंिों पर
ध्यान केंनद्रत करता है , जबनक कथन II आद्रण ता के स्तर के संबंि में तापमान और आराम की व्यल्किपरक िारणा को संबोनित
करता है । इसनलए, दोनों कथन सही हैं , लेनकन कथन II, कथन I की सही व्याख्या नही ं है ।
Source: http://www.atmo.arizona.edu/students/courselinks/fall10/atmo336/lectures/sec1/comfort.html
https://www.livescience.com/why-is-humidity-so-uncomfortable.html
https://myhealth.alberta.ca/Health/pages/conditions.aspx?hwid=tw9037

Q.21)
Ans) a
Exp) विकल्प a सही उत्तर है ।
• 1917 में, सरोनजनी नायडू ने मनहला भारत संघ (डब्ल्यूआईए) की स्थापना में मदद की, जो मनहलाओं के मुद्दों को
संबोनित करने और उनके कल्याण को बढ़ावा दे ने के नलए समनपणत संगठन था।
• WIA ने मनहलाओं के अनिकारों, नशक्षा और अन्य सामानजक सुिारों की वकालत करने में महत्वपूणण भूनमका ननभाई।
• सरोनजनी नायडू ने 1917 में लंदन में मनहलाओं के एक अल्कखल भारतीय प्रनतनननिमंडल का नेतृत्व नकया। इस
प्रनतनननिमंडल का उद्दे श्य राज्य सनचव, एडनवन मोंटागु से नमलना और अपनी मां गों को प्रस्तुत करना था।
• प्राथनमक मां ग मनहला मतानिकार थी, नजसका अथण था मनहलाओं को वोट दे ने और राजनीनतक प्रनिया में भाग लेने के
अनिकार की वकालत करना।
• प्रनतनननिमंडल का उद्दे श्य भारतीय मनहलाओं को मतदान का अनिकार दे ने के महत्व को उजागर करना और निनटश
अनिकाररयों से समथणन प्राप्त करना था।
• 1918 में, सरोनजनी नायडू ने बॉम्बे में आयोनजत नवशेष कां ग्रेस सत् में एक प्रस्ताव पेश नकया, नजसमें पुरुषों और
मनहलाओं दोनों के नलए मतदान के अनिकार की समान पात्ता की वकालत की गई।
• उनके प्रयास राजनीनतक प्रनतनननित्व में लैंनगक समानता की आवश्यकता के बारे में जागरूकता बढ़ाने में सहायक थे।
ज्ञानिार:
सरोविनी नायडू (1879-1949):

Forum Learning Centre: Delhi - 2nd Floor, IAPL House, 19 Pusa Road, Karol Bagh, New Delhi - 110005 | Patna - 2nd floor, AG Palace, E Boring Canal
Road, Patna, Bihar 800001 | Hyderabad - 1st & 2nd Floor, SM Plaza, RTC X Rd, Indira Park Road, Jawahar Nagar, Hyderabad, Telangana 500020
9311740400, 9311740900 | https://academy.forumias.com | admissions@forumias.academy | helpdesk@forumias.academy

[16]
PTS 2024 | Test Code: 121405 - Solutions |

• सरोनजनी नायडू एक भारतीय स्वतंत्ता कायणकताण , कवनयत्ी और राजनीनतज्ञ थीं।


• वह 1925 में कानपु र में आयोनजत कां ग्रेस के अनिवेशन में भारतीय राष्टरीय कांग्रेस (आईएनसी) की अध्यक्ष बनने वाली पहली
भारतीय मनहला थीं।
• वह नकसी भारतीय राज्य (उत्तर प्रदे श) की राज्यपाल बनने वाली पहली मनहला भी थीं।
• उन्हें कैसर-ए-नहं द पदक से भी सम्माननत नकया गया था
• भारत की कोनकला के रूप में जानी जाने वाली सरोनजनी नायडू एक प्रनतभाशाली कवनयत्ी थीं और उनके सानहल्कत्यक कायों ने
उन्हें बहुत प्रशंसा नदलाई।
• उन्होंने असहयोग, सनवनय अवज्ञा (िरसाना में नमक सत्याग्रह का नेतृत्व) और भारत छोडो आं दोलन में सनिय रूप से भाग
नलया।
• सरोनजनी नायडू मनहलाओं के अनिकारों की एक प्रमुख वकील थीं और उन्होंने भारत में मनहला नशक्षा और सशल्किकरण को
आगे बढ़ाने में महत्वपूणण भूनमका ननभाई।
सुर्ेता कृपलानी (1908-1974):
• सुचेता कृपलानी एक भारतीय स्वतंत्ता सेनानी और राजनीनतज्ञ थीं।
• उन्होंने भारतीय स्वतंत्ता आं दोलन में महत्वपूणण भूनमका ननभाई और महात्मा गां िी की करीबी सहयोगी थीं।
• वह 1940 में अल्कखल भारतीय मनहला कां ग्रेस की संस्थापक सदस्य थीं।
• वह भारत छोडो आं दोलन के दौरान अपनी भूनमका के नलए जानी जाती हैं ।
• वह भारत की संनविान सभा की पंद्रह मनहला सदस्यों में से एक थीं और उन्होंने भारतीय संनविान को आकार दे ने में महत्वपूणण
भूनमका ननभाई।
• भारत को स्वतंत्ता नमलने के बाद, वह नकसी भारतीय राज्य की पहली मनहला मुख्यमंत्ी बनीं, उन्होंने 1963 से 1967 तक
उत्तर प्रदे श की मुख्यमंत्ी के रूप में कायण नकया।
एनी बेसेंर् (1847-1933):
• नथयोसोनफकल सोसाइटी की एक प्रमुख हस्ती, बेसेंट ने भारत में शैनक्षक सुिारों की वकालत की। उन्होंने 1898 में सेंटरल नहं दू
कॉलेज की स्थापना की जो बाद में 1916 में बनारस नहं दू नविनवद्यालय बन गया
• 1916 में उन्होंने बाल गंगािर नतलक के साथ ऑल इं नडया होम रूल लीग की शुरुआत की।
• वह 1917 में कलकत्ता अनिवेशन में कां ग्रेस की पहली मनहला अध्यक्ष बनीं।
• वह निटे न और भारत दोनों में मनहलाओं के अनिकारों, नशक्षा और सामानजक सुिार की प्रबल समथणक थीं।
कमलादे िी र्ट्टोपाध्याय (1903-1988):
• कमलादे वी चट्टोपाध्याय एक भारतीय समाज सुिारक, स्वतंत्ता सेनानी और सां स्कृनतक कायणकताण थीं।
• उन्होंने पारं पररक नशल्प को बढ़ावा दे ने और मनहला कारीगरों को सशि बनाने के नलए अल्कखल भारतीय हस्तनशल्प बोडण जैसे
संगठनों की स्थापना की।
• वह चुनाव में (मद्रास ननवाण चन क्षेत् से) खडी होने वाली भारत की पहली मनहला थीं।
• उन्होंने प्रमुख नेताओं के साथ काम करते हुए सनवनय अवज्ञा आं दोलन और भारत छोडो आं दोलन में भाग नलया।
• वह भारत में मनहला अनिकार आं दोलन की अग्रणी हल्कस्तयों में से एक थीं और उन्होंने लैंनगक समानता, मनहला नशक्षा और
सशल्किकरण के नलए अथक प्रयास नकया।
• चट्टोपाध्याय अंतराण ष्टरीय मनहला आं दोलन में भी शानमल थे और उन्होंने नवनभन्न अंतराण ष्टरीय मंचों पर भारत का प्रनतनननित्व नकया।
• उन्होंने राष्टरीय नाट्य नवद्यालय, संगीत नाटक अकादमी और भारतीय नशल्प पररषद जैसे संस्थानों की स्थापना में महत्वपूणण
भूनमका ननभाई।
Source: https://egyankosh.ac.in/bitstream/123456789/1596/1/Unit-3.pdf
A Brief History of Modern India by Rajiv Ahir
https://amritmahotsav.nic.in/district-reopsitory-detail.htm?10567
https://amritmahotsav.nic.in/unsung-heroes-detail.htm?319
https://www.indiatoday.in/india-today-insight/story/recall-sarojini-naidu-the-civil-crusader-2500959-
2024-02-12

Forum Learning Centre: Delhi - 2nd Floor, IAPL House, 19 Pusa Road, Karol Bagh, New Delhi - 110005 | Patna - 2nd floor, AG Palace, E Boring Canal
Road, Patna, Bihar 800001 | Hyderabad - 1st & 2nd Floor, SM Plaza, RTC X Rd, Indira Park Road, Jawahar Nagar, Hyderabad, Telangana 500020
9311740400, 9311740900 | https://academy.forumias.com | admissions@forumias.academy | helpdesk@forumias.academy

[17]
PTS 2024 | Test Code: 121405 - Solutions |

Q. 22)
Ans) b
Exp) विकल्प b सही उत्तर है।
युग्म 1 सही है : ओनडशा राज्य में ल्कस्थत भीतररका, अपने मैंग्रोव वनों के नलए प्रनसद्ध है । ये मैंग्रोव िाह्मणी, बैतरणी और िामरा
जैसी ननदयों के मुहाने क्षेत्ों में फैले हुए हैं ।
• भीतररका का मैंग्रोव पाररल्कस्थनतकी तंत् सुंदरबन के बाद भारत में दू सरा सबसे बडा है ।
• मैंग्रोव वन उष्णकनटबंिीय और उपोष्णकनटबंिीय क्षेत्ों में समुद्र तट के नकनारे पाए जाने वाले अनद्वतीय पाररल्कस्थनतक तंत् हैं ।
• वे खारे पानी और ज्वारीय ल्कस्थनतयों में पनपते हैं , खारे पानी के मगरमच्छ, जैतून ररडले कछु ए, सां भर नहरण, मछली पकडने
वाली नबल्कल्लयों आनद सनहत नवनभन्न प्रकार की प्रजानतयों के नलए महत्वपूणण आवास प्रदान करते हैं ।
• भीतराननका के मैंग्रोव तटीय क्षेत्ों को कटाव, तूफान और सुनामी से बचाने के नलए महत्वपूणण हैं , साथ ही नवनभन्न समुद्री
प्रजानतयों के नलए प्रजनन स्थल के रूप में भी काम करते हैं ।
युग्म 2 गलत है : आं ध्र प्रदे श के पू वी घाट में ल्कस्थत पानपकोंडा राष्टरीय उद्यान की नवशेषता मुख्यतः शुष्क पणणपाती वन हैं ।
• इन जंगलों में अलग-अलग गीले और सूखे मौसम का अनुभव होता है , पानी के संरक्षण के नलए पेड सूखे मौसम के दौरान
अपने पत्ते नगरा दे ते हैं ।
• पानपकोंडा के शुष्क पणणपाती जंगलों में अिण -शुष्क पररल्कस्थनतयों के अनुकूल वनस्पनतयों और जीवों की समृद्ध नवनविता मौजूद
है ।
• अभयारण्य बाघ, तेंदुए, हाथी, भारतीय गौर, लकडबग्घा आनद सनहत नवनभन्न वन्यजीव प्रजानतयों का घर है ।
• वनस्पनत में सागौन, बां स, टनमणननलया और नमनश्रत झानडयााँ जैसी प्रजानतयााँ शानमल हैं , जो पूवी घाट में एक महत्वपूणण
पाररल्कस्थनतकी तंत् का ननमाण ण करती हैं ।
युग्म 3 गलत है: नदनहं ग पटकाई, नजसे अक्सर "पूवण का अमेज़ॅन" कहा जाता है , पूवोत्तर राज्य असम में ल्कस्थत है । इसमें तराई के
वषाण वन या सदाबहार वन का नवशाल नवस्तार शानमल है ।
• यह क्षेत् अपनी असािारण उच्च जैव नवनविता के नलए जाना जाता है , नजसमें घनी वनस्पनत, नवशाल वृक्ष और वन्य जीवन की
बहुतायत है ।
• नदनहं ग पटकाई जंगल कई स्थाननक, लुप्तप्राय प्रजानतयों और एनशयाई हाथी, हूलॉक नगब्बन, चीनी पैंगोनलन, सां भर और
बानकिंग नहरण, गौर, मलायन नवशाल नगलहरी, लाउडे ड तेंदुए, तेंदुए, गोर्ल्न नबल्ली और अन्य प्रमुख प्रजानतयों के नलए एक
महत्वपूणण ननवास स्थान के रूप में कायण करता है । पनक्षयों और सरीसृपों की नवनभन्न प्रजानतयााँ ।
• जंगल स्थानीय जलवायु , जल चि और नमट्टी की उवणरता को नवननयनमत करने में भी महत्वपूणण भूनमका ननभाते हैं , नजससे यह
पाररल्कस्थनतक रूप से महत्वपूणण हो जाता है ।
युग्म 4 सही है : रणथंभौर राष्टरीय उद्यान 392 वगण नकमी के क्षेत् में फैला हुआ है । इसके आसपास के अभयारण्यों जैसे माननसं ह
अभयारण्य और कैला दे वी अभयारण्य के साथ। यह पाकण अपने बाघों के नलए प्रमुख रूप से प्रनसद्ध है और अपने प्राकृनतक
आवास में नशकाररयों को दे खने के नलए भारत में सबसे अच्छे स्थानों में से एक है । बाघों को नदन के समय भी आसानी से दे खा
जा सकता है , वे नशकार में व्यस्त रहते हैं और अपने बच्चों की उनचत दे खभाल करते हैं ।
इसमें घुमावदार पहानडयााँ और चट्टानें , और घास के मैदान, झीलें और नाले, और शुष्क-पणणपाती वन प्रणाली का नमश्रण है जो
वनस्पनतयों और जीवों की अनविसनीय नवनविता का घर है । बाघ के अलावा, हम रणथंभौर में िॉथ भालू, तेंदुआ, कैराकल,
नसयार, लोमडी, लकडबग्घा और नेवला भी पा सकते हैं ।
Source:
https://forests.ap.gov.in/papikonda.php#:~:text=The%20predominant%20and%20most%20extensive,are%
20mainly%20dry%20deciduous%20species.
http://www.eodishatourism.com/attraction/bhitarkanika-mangroves-crocodile/1094
http://www.ecoindia.com/jungles/chotanagpur-dry-deciduous.html
https://kaziranganationalparkassam.in/dehing-patkai-national-park/
https://ntca.gov.in/assets/uploads/briefnote/palamau.pdf
https://www.tourism.rajasthan.gov.in/ranthambore.html
Forum Learning Centre: Delhi - 2nd Floor, IAPL House, 19 Pusa Road, Karol Bagh, New Delhi - 110005 | Patna - 2nd floor, AG Palace, E Boring Canal
Road, Patna, Bihar 800001 | Hyderabad - 1st & 2nd Floor, SM Plaza, RTC X Rd, Indira Park Road, Jawahar Nagar, Hyderabad, Telangana 500020
9311740400, 9311740900 | https://academy.forumias.com | admissions@forumias.academy | helpdesk@forumias.academy

[18]
PTS 2024 | Test Code: 121405 - Solutions |

Q.23)
Ans) a
Exp) विकल्प a सही उत्तर है ।
जोडी 1 सही है : अजंता की गुफाएाँ
• स्थान: औरं गाबाद नजला, महाराष्टर, सह्यादरी रें ज (पनिमी घाट) में
• अजंता की गुफाएं 29 चट्टानों को काटकर बनाए गए बौद्ध गुफा स्मारकों का एक समूह है , जो दू सरी शताब्दी ईसा पूवण से
लेकर लगभग 6वीं शताब्दी ईस्वी तक के हैं । वे अपने शानदार नचत्ों और मूनतणयों के नलए प्रनसद्ध हैं , नजन्हें बौद्ध िानमणक कला की
उत्कृष्ट कृनतयााँ माना जाता है , नजनमें बुद्ध की आकृनतयााँ और जातक कथाओं का नचत्ण है ।
• इन गुफाओं का ननमाण ण मुख्य रूप से सातवाहन और वाकाटक राजवंशों द्वारा नकया गया था
• नदी संघ: चट्टानों को काटकर बनाई गई ये गु फाएाँ दक्कन के पठार में वाघोरा नदी के तट पर ल्कस्थत हैं । यह नदी बौद्ध नभक्षुओं
को ध्यान और िानमणक प्रथाओं के नलए एक एकां त स्थान प्रदान करती थी।
• यूनेस्को नवि िरोहर स्थल: गुफाओं को 1983 से यूनेस्को नवि िरोहर स्थल के रूप में मान्यता दी गई है , जो वैनिक सां स्कृनतक
नवरासत में उनके महत्व को उजागर करता है ।
जोडी 2 ग़लत है : मीनाक्षी मंनदर
• स्थान: मदु रै, तनमलनाडु
• मीनाक्षी मंनदर को मीनाक्षी अम्मन या मीनाक्षी सुंदरे िर मंनदर के नाम से भी जाना जाता है , जो दे वी मीनाक्षी, पावणती का एक
रूप, और उनके पनत भगवान नशव (यहां सुंदरे िर के नाम से जाना जाता है ) को समनपणत है । वतणमान संरचना का बडे पैमाने पर
पुनननणमाण ण 16वीं से 17वीं शताब्दी में नायक राजवंश के शासकों द्वारा नकया गया था।
• नदी संघ: यह वैगई नदी के दनक्षणी तट पर ल्कस्थत है । नदी मंनदर के जीवन और त्योहारों का अनभन्न अंग है , खासकर
'तेप्पाकुलम' त्योहार के दौरान, जहां दे वताओं को जुलूस के रूप में वैगई नदी पर ले जाया जाता है ।
• वास्तुनशल्प चमत्कार: यह द्रनवड वास्तुकला का एक प्रमुख उदाहरण है । यह मंनदर अपने नवशाल गोपुरम (प्रवेश द्वार टावर),
समृद्ध नक्काशी, स्वणण कमल टैं क, हजार स्तंभों वाले हॉल के नलए प्रनसद्ध है ।
जोडी 3 गलत है : अमरावती स्तूप
• स्थान: अमरावती, गुंटूर नजला, आं ध्र प्रदे श
• अमरावती स्तूप, नजसे महाचैत्य के नाम से भी जाना जाता है , भारत के सबसे बडे स्तूपों में से एक है और बौद्ध कला और
संस्कृनत का एक महत्वपूणण केंद्र था। तीसरी शताब्दी ईसा पूवण और तीसरी शताब्दी ईस्वी के बीच चरणों में नननमणत, यह भारत में
स्तूपों के नडजाइन के नवकास को दशाण ता है ।
• नदी संघ: स्तूप कृष्णा नदी के तट पर ल्कस्थत है ।
• वास्तुकला की नवशेषताएं : अमरावती स्तूप अपनी गोलाकार वेनदका (बाड) के नलए प्रनसद्ध है जो बुद्ध के जीवन को दशाण ती
जनटल नक्काशी से सुसल्कित है । जबनक स्तूप की अनिकांश संरचना सनदयों से नष्ट हो गई है , कई मूनतणकला अवशेष भारत और
निटे न के नवनभन्न संग्रहालयों में संरनक्षत नकए गए हैं ।
Source: https://whc.unesco.org/en/list/242/
https://m.economictimes.com/news/india/5-things-to-know-about-madurai-meenakshi-amman-temple-
where-pm-modi-prayed/meenakshi-templenbspnbspnbspnbsp/slideshow/108074370.cms
https://timesofindia.indiatimes.com/travel/destinations/exploring-amaravati-an-important-buddhist-
centre-in-andhra-pradesh/articleshow/97958763.cms
Indian Art and Culture by Nitin Singhania

Q.24)
Ans) d
Exp) विकल्प d सही उत्तर है।

Forum Learning Centre: Delhi - 2nd Floor, IAPL House, 19 Pusa Road, Karol Bagh, New Delhi - 110005 | Patna - 2nd floor, AG Palace, E Boring Canal
Road, Patna, Bihar 800001 | Hyderabad - 1st & 2nd Floor, SM Plaza, RTC X Rd, Indira Park Road, Jawahar Nagar, Hyderabad, Telangana 500020
9311740400, 9311740900 | https://academy.forumias.com | admissions@forumias.academy | helpdesk@forumias.academy

[19]
PTS 2024 | Test Code: 121405 - Solutions |

नवकल्प d सही है : समग्र जल प्रबंिन सूचकां क नीनत आयोग द्वारा प्रकानशत नकया जाता है । इसे नीनत आयोग द्वारा राज्यों के
बीच सहकारी और प्रनतस्पिी संघवाद की भावना पैदा करने के एक उपकरण के रूप में पहली बार 2018 में लॉन्च नकया गया
था। सूचकां क नौ नवषयों के आिार पर राज्य सरकारों को रैं क करता है
जल ननकायों की बहाली; भूजल; प्रमुख एवं मध्यम नसंचाई; जलसंभर नवकास; सहभागी नसंचाई प्रथाएाँ ; खेत पर नटकाऊ जल
उपयोग प्रथाएाँ ; ग्रामीण पेयजल; शहरी जल आपूनतण एवं स्वच्छता; नीनत और शासन.
ज्ञानिार:
केंद्रीय भूजल प्रानिकरण (सीडब्ल्यूजीए) का गठन पयाण वरण (संरक्षण) अनिननयम, 1986 की िारा 3 के तहत नकया गया है ।
सीडब्ल्यूजीए दे श में भूजल संसािनों के नवकास और प्रबंिन को ननयंनत्त और ननयंनत्त करता है । यह दे श में भूजल संसािनों
को नवननयनमत, ननयंनत्त, प्रबंिन और नवकनसत करने के नलए ननदे श जारी कर सकता है और दं डात्मक प्राविान लागू कर
सकता है ।
भारतीय कृनष अनुसंिान पररषद-भारतीय जल प्रबंिन संस्थान (ICAR-IIWM) एक स्वायत्त संगठन है । आईसीएआर कृनष
अनुसंिान और नशक्षा नवभाग, कृनष और नकसान कल्याण मंत्ालय के अिीन है । आईसीएआर-आईआईडब्ल्यूएम का लक्ष्य
नटकाऊ कृनष उत्पादन के नलए बेहतर जल प्रबंिन प्रौद्योनगनकयों को नवकनसत करना और उन्हें प्रसाररत करना है । केंद्रीय जल
आयोग जल शल्कि मंत्ालय के जल संसािन, नदी नवकास और गंगा कायाकल्प नवभाग के एक संलग्न कायाण लय के रूप में कायण
कर रहा है । यह संबंनित राज्य सरकारों के परामशण से बाढ़ ननयंत्ण, नसंचाई, नेनवगेशन, पेयजल आपूनतण और जलनवद् युत नवकास
सनहत दे श भर में जल संसािन योजनाओं के समन्वय और आगे बढ़ाने के नलए नजम्मेदार है ।
Source: https://indianexpress.com/article/india/niti-weighs-discontinuing-key-water-report-launched-5-
years-ago-9015084/
https://vikaspedia.in/energy/environment/composite-water-management-index#:~:text=Policy%20and-
,governance,-15

Q.25)
Ans) b
Exp) विकल्प b सही उत्तर है।
लोरीन और िोमीन जैसी प्रनतनियाशील गैसों की प्रचुरता के अलावा, अंटाकणनटक ओजोन नछद्र के ननमाण ण के नलए ध्रुवीय
समतापमंडलीय बादल (पीएससी) बनाने के नलए पयाण प्त कम तापमान, अन्य समतापमंडलीय क्षेत्ों में हवा से अलगाव और सूयण
के प्रकाश की आवश्यकता होती है । इन सभी कारकों को ध्रुवीय भंवर द्वारा प्रत्यक्ष और अप्रत्यक्ष रूप से बढ़ावा नदया जाता है
ध्रुवीय वाताग्र कम दबाव और ठं डी हवा का एक बडा क्षेत् है जो पृथ्वी के ध्रुवों (आकणनटक और अंटाकणनटका दोनों) को घेरता
है , खासकर सनदण यों के महीनों में।
यह वाताग्र समतापमंडल और क्षोभमंडल दोनों में मौजूद है
समतापमंडलीय ध्रुवीय भंवर:
o पृथ्वी की सतह से लगभग 15 नकमी से 50 नकमी ऊपर ल्कस्थत, समतापमंडलीय ध्रुवीय वाताग्र सनदण यों के दौरान सबसे मजबूत
होता है ।
o यह तब बनता है जब ध्रुवीय रात शुरू होते ही आकणनटक या अंटाकणनटक का तापमान तेजी से ठं डा हो जाता है ।
क्षोभमंडलीय ध्रुवीय वाताग्र:
o क्षोभमंडलीय ध्रुवीय वाताग्र सतह से लगभग 10 नकमी से 15 नकमी तक फैला हुआ है ।
o इसे क्षोभमंडलीय जेट स्टर ीम के ध्रुव की ओर वाले क्षेत् के रूप में पररभानषत नकया गया है ।
o समतापमंडलीय भंवर के नवपरीत, क्षोभमंडलीय वाताग्र साल भर मौजूद रहता है लेनकन सनदण यों के दौरान सबसे मजबूत होता
है जब ध्रुवीय क्षेत् सबसे ठं डे होते हैं ।
ओजोन ररिीकरण:
o ध्रुवीय वाताग्र, नवशेष रूप से दनक्षणी गोलािण में, ओजोन क्षरण में महत्वपूणण भूनमका ननभाते हैं ।

Forum Learning Centre: Delhi - 2nd Floor, IAPL House, 19 Pusa Road, Karol Bagh, New Delhi - 110005 | Patna - 2nd floor, AG Palace, E Boring Canal
Road, Patna, Bihar 800001 | Hyderabad - 1st & 2nd Floor, SM Plaza, RTC X Rd, Indira Park Road, Jawahar Nagar, Hyderabad, Telangana 500020
9311740400, 9311740900 | https://academy.forumias.com | admissions@forumias.academy | helpdesk@forumias.academy

[20]
PTS 2024 | Test Code: 121405 - Solutions |

o वसंत के दौरान, इन भंवरों के भीतर ओजोन की कमी अनिकतम तक पहुाँ च जाती है ।


नवकल्प b सही है : ध्रुवीय भंवर अत्यनिक ठं डे तापमान से जुडा होता है , नजससे ध्रुवीय समतापमंडलीय बादलों (पीएससी) का
ननमाण ण होता है ।
ध्रुवीय भंवर ध्रुवीय समताप मंडल के अंदर या बाहर हवा की पयाण प्त गनत को रोकता है । इस प्रकार, एक बार जब पीएससी पर
प्रनतनियाओं से ध्रुवीय क्षेत्ों में रासायननक पररवतणन होते हैं , तो अलगाव उन पररवतणनों को कई हफ्ों से लेकर महीनों तक
संरनक्षत रखता है
• ध्रुवीय समतापमंडलीय बादल (नजसे नैनियस बादल भी कहा जाता है ) अनद्वतीय बादल संरचनाएं हैं जो ध्रुवीय सनदण यों के दौरान
समतापमंडल में होती हैं ।
• वे बेहद कम तापमान पर बनते हैं , आमतौर पर -78°C (-108°F) से नीचे , जो आसपास के समताप मंडल की तुलना में ठं डा
होता है ।
• पीएससी छोटे बफण के निस्टल और नाइनटर क एनसड हाइडरेट्स से बने होते हैं ।
• नाइनटर क एनसड (HNO₃) अणु PSC बफण निस्टल की सतहों पर संघननत होते हैं ।
• ये पीएससी ओजोन-क्षयकारी प्रनतनियाओं के नलए एक मंच प्रदान करते हैं ।
• प्रमुख प्रनतनियाओं में लोरीन और िोमीन यौनगक शानमल हैं :
o लोरीन लोरोफ्लोरोकाबणन (सीएफसी) से ननकलता है , जो कभी रे निजरें ट, एरोसोल और अन्य औद्योनगक अनुप्रयोगों में
व्यापक रूप से उपयोग नकया जाता था।
o जब सूयण का प्रकाश समताप मंडल में पहुं चता है , तो यह सीएफसी को तोड दे ता है , नजससे लोरीन परमाणु मुि हो जाते हैं ।
o ये लोरीन परमाणु नफर ओजोन (O₃) अणुओं के साथ प्रनतनिया करते हैं , नजससे वे नष्ट हो जाते हैं ।
o समग्र प्रनिया को इस प्रकार संक्षेप में प्रस्तुत नकया जा सकता है :
Cl + O₃ → ClO + O₂
ClO + O → Cl + O₂
इसका शुद्ध पररणाम ओजोन सां द्रता में कमी है ।
जैसे-जैसे अंटाकणनटक वसंत आगे बढ़ता है , यह तंत् अंटाकणनटका के ऊपर ओजोन नछद्र बनाता है ।
आकणनटक क्षेत् में भी इस तंत् के कारण ओजोन की कमी का अनुभव होता है , लेनकन यह अंटाकणनटका की तुलना में कम
गंभीर है ।
नवकल्प ए गलत है : ध्रुवीय भंवर की नवशे षता अत्यनिक ठं डे तापमान है , न नक तीव्र गमी। तेज हवाएाँ घषणण उत्पन्न करती हैं ,
लेनकन समताप मंडल (जहााँ ओजोन क्षय होता है ) में प्रभाव न्यूनतम होता है । उच्च तापमान वास्तव में भंवर को कमजोर करता
है और ओजोन ररिीकरण के नलए आवश्यक ल्कस्थनतयों को बानित करता है ।
नवकल्प सी गलत है : जबनक ध्रुवीय भंवर वास्तव में एक कम दबाव वाली प्रणाली है , यह सीिे तौर पर अशां नत का कारण नही ं
बनता है जो ओजोन टू टने को तेज करता है । ओजोन ररिीकरण के प्राथनमक तंत् में ध्रुवीय समतापमंडलीय बादल (पीएससी)
शानमल हैं , जो ओजोन-क्षयकारी प्रनतनियाओं के नलए सतह प्रदान करते हैं ।
नवकल्प डी गलत है : ध्रुवीय जेट स्टर ीम, भंवर के चारों ओर तेज हवाओं का एक बैंड, वास्तव में सीएफसी जैसे ओजोन-क्षयकारी
रसायनों का पररवहन कर सकता है । हालााँ नक, ध्रुवीय भंवर सीिे तौर पर जेट स्टर ीम को मजबूत नही ं करता है । इसके बजाय, यह
वायुमंडलीय तरं गों के साथ संपकण करता है और जेट स्टर ीम के व्यवहार को प्रभानवत करता है । इसके अलावा, वास्तनवक
ओजोन-क्षयकारी प्रनतनियाओं के घनटत होने के नलए भंवर के भीतर पीएससी की उपल्कस्थनत अनिक महत्वपूणण है ।
Source: https://ozonewatch.gsfc.nasa.gov/facts/vortex_NH.html

Q.26)
Ans) d
Exp) विकल्प d सही उत्तर है।

Forum Learning Centre: Delhi - 2nd Floor, IAPL House, 19 Pusa Road, Karol Bagh, New Delhi - 110005 | Patna - 2nd floor, AG Palace, E Boring Canal
Road, Patna, Bihar 800001 | Hyderabad - 1st & 2nd Floor, SM Plaza, RTC X Rd, Indira Park Road, Jawahar Nagar, Hyderabad, Telangana 500020
9311740400, 9311740900 | https://academy.forumias.com | admissions@forumias.academy | helpdesk@forumias.academy

[21]
PTS 2024 | Test Code: 121405 - Solutions |

कथन 1 और कथन 2 गलत हैं : XPoSat (एक्स-रे पोलाररमीर्र सैर्ेलाइर्) भारत का (दु वनया का नही ां) पहला समवपचत
पोलाररमेर्री वमशन है जो चरम ल्कस्थनतयों में उज्ज्वल खगोलीय एक्स-रे स्रोतों की नवनभन्न गनतशीलता का अध्ययन करता है ।
अंतररक्ष यान दो वैज्ञाननक पेलोड को कम पृथ्वी की कक्षा (भू -ल्कस्थर कक्षा नही ं) में ले जाएगा। प्राथनमक पेलोड POLIX (एक्स-रे में
पोलाररमीटर इं स्टूमेंट) खगोलीय मूल के 8-30 keV फोटॉन के मध्यम एक्स-रे ऊजाण रें ज में ध्रुवणनमनत मापदं डों (ध्रुवीकरण की
नडग्री और कोण) को मापेगा। एक्सएसपीईसीटी (एक्स-रे स्पेक्टरोस्कोपी और टाइनमंग) पेलोड 0.8-15 केवी की ऊजाण रें ज में
स्पेक्टरोस्कोनपक जानकारी दे गा।
दू सरी ओर, 2021 में लॉन्च वकया गया नासा का इमेविांग एक्स-रे पोलाररमेर्री एक्सप्लोरर (IXPE) वमशन, विवभन्न
िह्ाांडीय स्रोतोां से एक्स-रे के ध्रुिीकरण को मापने के वलए समवपचत पहला उपग्रह वमशन था।
Source: Dipin Sir’s Current Affairs classes
https://www.isro.gov.in/XPoSat_X-Ray_Polarimetry_Mission.html
https://www.isro.gov.in/XPoSat.html#:~:text=XPoSat%20(X%2Dray%20Polarimeter%20Satellite,in%20a%2
0low%20earth%20orbit.
https://indianexpress.com/article/explained/explained-sci-tech/isro-launches-xposat-9090416/

Q.27)
Ans) d
Exp) विकल्प d सही उत्तर है।
बायोस्फीयर ररजवण एक संरनक्षत क्षेत् है जो स्थलीय, तटीय और समुद्री पाररल्कस्थनतक तंत् को शानमल करता है , जहां संरक्षण के
प्रयास सतत नवकास प्रथाओं के साथ संतुनलत होते हैं । ये भंडार जैव नवनविता, पाररल्कस्थनतकी तंत् की गनतशीलता और नटकाऊ
संसािन प्रबंिन पर शोि और ननगरानी के नलए जीनवत प्रयोगशालाओं के रूप में काम करते हैं । एक बायोस्फीयर ररजवण में कई
संरनक्षत क्षेत् शानमल हो सकते हैं जो सामूनहक रूप से एक नननदण ष्ट क्षेत् के भीतर पाररल्कस्थनतक तंत् और जैव नवनविता के संरक्षण
और नटकाऊ प्रबंिन में योगदान करते हैं । इन संरनक्षत क्षेत्ों में राष्टरीय उद्यान, वन्यजीव अभयारण्य, प्रकृनत भंडार और अन्य
संरक्षण क्षेत् शानमल हो सकते हैं ।
विकल्प a गलत है : कान्ा र्ाइगर ररििच मध्य प्रदे श में ल्कस्थत है लेनकन पचमढ़ी बायोस्फीयर ररजवण का नहस्सा नही ं है ।
विकल्प b गलत है : केन-घवड़याल अभयारण्य पन्ना बायोस्फीयर ररििच का वहस्सा है ।
विकल्प c गलत है : पन्ना राष्टरीय उद्यान और गां गऊ अभयारण्य पन्ना बायोस्फीयर ररििच का वहस्सा हैं ।
विकल्प d सही है : पर्मढी बायोस्फीयर ररििच में तीन सांरवित िेत्र शावमल हैं । सतपुड़ा राष्टरीय उद्यान (524.37
वकमी2), बोरी (485.72 वकमी2) और पर्मढी (439.15 वकमी2) अभयारण्य और RF और PF का िेत्र (105.99
वकमी2)।
पचमढ़ी बायोस्फीयर ररजवण, 1999 में स्थानपत, भारत के मध्य प्रदे श में सतपुडा रें ज के भीतर ल्कस्थत है । यह 4,926 िगच
वकलोमीर्र के विशाल िेत्र को किर करता है , जो इसे राज्य का सबसे बडा बायोस्फीयर ररजवण बनाता है । इसमें नवनभन्न
प्रकार के वन हैं , नजनमें उष्णकनटबंिीय नम पणणपाती वन, उष्णकनटबंिीय शुष्क पणणपाती वन और पवणतीय वन शानमल हैं , नजनमें
से प्रत्येक नवनभन्न पौिों और जानवरों का समथणन करता है । अपनी प्राकृनतक नवनविता के अलावा, पचमढ़ी बायोस्फीयर ररजवण
सां स्कृनतक रूप से भी महत्वपूणण है । यहां बैगा, गोंड और कोरकू जैसे स्वदे शी आनदवासी समुदाय रहते हैं , नजन्हें स्थानीय
पयाण वरण की गहरी समझ है और वे पीनढ़यों से प्रकृनत के साथ सद्भाव में रहते आए हैं ।
Source: https://epco.mp.gov.in/pachmarhi-bio-reserves
https://timesofindia.indiatimes.com/city/nagpur/ngt-notices-issued-for-violation-of-environmental-
norms-in-pachmarhi-airstrip-expansion/articleshow/105947260.cms
https://www.unesco.org/en/mab/pachmarhi#:~:text=Pachmarhi%20comprises%20three%20protection%2
0sites,as%20the%20Satpura%20Tiger%20Reserve.

Forum Learning Centre: Delhi - 2nd Floor, IAPL House, 19 Pusa Road, Karol Bagh, New Delhi - 110005 | Patna - 2nd floor, AG Palace, E Boring Canal
Road, Patna, Bihar 800001 | Hyderabad - 1st & 2nd Floor, SM Plaza, RTC X Rd, Indira Park Road, Jawahar Nagar, Hyderabad, Telangana 500020
9311740400, 9311740900 | https://academy.forumias.com | admissions@forumias.academy | helpdesk@forumias.academy

[22]
PTS 2024 | Test Code: 121405 - Solutions |

Q.28)
Ans) c
Exp) विकल्प c सही उत्तर है ।
विकल्प 1 सही है : इन-विर्र ो तकनीक, एक एक्स-सीर्ू सांरिण विवध, पौधोां, िानिरोां और रोगाणुओ ां की विविधता को
सांरवित करने के वलए उपयोग की िाने िाली प्रयोगशाला विवधयोां को शावमल करती है । नटशू कल्चर एक इन-नवटर ो
तकनीक है जो पौिों की कोनशकाओं की "टोनटपोटें सी" की अविारणा पर आिाररत है । सेलुलर टोनटपोटें सी एक कोनशका की
एक पूरे नए पौिे को जन्म दे ने की क्षमता है । मूल्यवान संकटग्रस्त प्रजानतयों के भंडारण के नलए ऊतक संविणन नवनियााँ तेजी से
लोकनप्रय हो रही हैं । ऐसा इसनलए है क्ोंनक वे प्रजनन में आसानी के अनतररि लाभ के साथ, एक सुरनक्षत और रोगजनक़ मुि
वातावरण में अपेक्षाकृत छोटी जगह में कई मूल्यवान प्रजानतयों को संरनक्षत करने के अनद्वतीय अवसर प्रदान करते हैं ।
विकल्प 2 सही है : कैविि प्रिनन को एक पूिच-थथाने सांरिण तकनीक माना िाता है । इसमें जीवों को उनके प्राकृनतक
संकटों के बाहर ननयंनत्त वातावरण, जैसे नचनडयाघरों या नवशेष प्रजनन सुनविाओं में प्रजनन और पालन-पोषण करना शानमल
है । यह इन-सीटू संरक्षण से नभन्न है , जो प्रजानतयों को उनके प्राकृनतक आवासों के भीतर संरनक्षत और संरनक्षत करने पर केंनद्रत
है ।
विकल्प 3 गलत है : पवित्र उपिन िांगलोां के र्ु कड़े हैं िो थथानीय समुदायोां द्वारा उनके साांस्कृवतक या धावमचक महत्व के
कारण सांरवित हैं । जबनक पनवत् उपवन स्वस्थाने (उनके प्राकृनतक आवास के भीतर) जैव नवनविता की रक्षा करते हैं , उन्हें
वाह्य स्थाने संरक्षण तकनीक नहीं माना जाता है क्ोंनक उनमें प्रजानतयों को उनके प्राकृनतक आवास से हटाना या स्थानां तररत
करना शानमल नहीं है ।
विकल्प 4 सही है : िनस्पवत उद्यान ऐसी सुविधाएां हैं िहाां िैज्ञावनक, शैविक और सांरिण उद्दे श्योां के वलए पौधोां की एक
विस्तृत श्रृांखला की खेती की िाती है । वे अक्सर दु लणभ और लुप्तप्राय प्रजानतयों सनहत व्यापक जीनवत पौिों का संग्रह रखते
हैं । वनस्पनत उद्यान अपने प्राकृनतक आवासों के बाहर पौिों की नवनविता को संरनक्षत करके वाह्य स्थाने संरक्षण में महत्वपूणण
भूनमका ननभाते हैं ।
विकल्प 5 गलत है : िन्यिीि अभयारण्य िन्यिीिोां और उनके आिासोां के सांरिण के वलए अलग रखी गई भूवम के
सांरवित िेत्र हैं। वे मुख्य रूप से वन्यजीवों के नलए संरनक्षत वातावरण प्रदान करके स्वस्थाने संरक्षण पर ध्यान केंनद्रत करते हैं ।
विकल्प 6 सही है : िायोवप्रििेशन में िैविक सामग्री (िैसे बीि, भ्रूण, या ऊतक) को बहुत कम तापमान पर सांरवित
करना शावमल है , आमतौर पर तरल नाइटर ोजन का उपयोग करके, नवस्ताररत अवनि के नलए उनकी व्यवहायणता बनाए रखने के
नलए। इसका उपयोग आमतौर पर दु लणभ और लुप्तप्राय प्रजानतयों की आनुवंनशक सामग्री को संग्रहीत करने के नलए पूवण -स्थाने
संरक्षण में नकया जाता है ।
Source: https://ebooks.inflibnet.ac.in/esp03/chapter/strategies-for-biodiversity-conservation/

Q.29)
Ans) a
Exp) विकल्प a सही उत्तर है ।
लैगून पानी का एक उथला भाग है िो गहरे पानी के भाग, आमतौर पर महासागर से अिरोध द्वीपोां, सैंडबार या र्ट्टानोां
द्वारा अलग होता है । लैगून की नवशेषता उनके शां त और अपेक्षाकृत बंद वातावरण, खारे या खारे पानी के कारण होती है । वे
अक्सर नवनभन्न समुद्री और पक्षी प्रजानतयों के नलए आवास के रूप में काम करते हैं और तटीय पाररल्कस्थनतकी तंत् और मछली
पकडने, पयणटन और मनोरं जन जैसी मानवीय गनतनवनियों के नलए महत्वपूणण हो सकते हैं ।
विकल्प 1 गलत है : दीपोर बील:
• दीपोर बील असम राज्य में ल्कस्थत है । यह गुवाहाटी शहर के दनक्षण पनिम में ल्कस्थत है ।
• दीपोर बील एक मीठे पानी की झील है , लैगून नहीं। यह असम की िह्मपुत् घाटी में सबसे बडी बीलों (झील के नलए एक
स्थानीय शब्द) में से एक है ।
• दीपोर बील अंतराण ष्टरीय महत्व का एक आद्रण भूनम है और इसे 2002 में रामसर साइर् के रूप में नावमत वकया गया था

Forum Learning Centre: Delhi - 2nd Floor, IAPL House, 19 Pusa Road, Karol Bagh, New Delhi - 110005 | Patna - 2nd floor, AG Palace, E Boring Canal
Road, Patna, Bihar 800001 | Hyderabad - 1st & 2nd Floor, SM Plaza, RTC X Rd, Indira Park Road, Jawahar Nagar, Hyderabad, Telangana 500020
9311740400, 9311740900 | https://academy.forumias.com | admissions@forumias.academy | helpdesk@forumias.academy

[23]
PTS 2024 | Test Code: 121405 - Solutions |

• झील नवनभन्न प्रकार के ननवासी और प्रवासी पनक्षयों के नलए एक महत्वपूणण ननवास स्थान है , विसमें ग्रेर्र एडिुर्ेंर् स्टॉकच,
स्पॉर्-वबल्ड पेवलकन और वबयसच पोर्ाडच िैसी कई लुप्तप्राय प्रिावतयाां शावमल हैं ।
विकल्प 2 सही है : पुवलकर् झील:
• पुवलकर् झील आां ध्र प्रदे श और तवमलनाडु की सीमा पर क्तथथत है । यह चेन्नई से लगभग 60 नकमी उत्तर में ल्कस्थत है ।
• नचिा झील के बाद यह भारत का दू सरा सबसे बडा खारे पानी का लैगून है ।
• यह श्रीहररकोर्ा के बैररयर द्वीप द्वारा बंगाल की खाडी से अलग होता है ।
• यह दु ननया के नवनभन्न नहस्सों से आने वाले प्रवासी पनक्षयों सनहत नवनभन्न पक्षी प्रजानतयों के नलए एक महत्वपूणण ननवास स्थान है ।
यह राजहं स, पेनलकन और पेंटेड स्टॉकण सनहत नवनभन्न पक्षी प्रजानतयों को आवास प्रदान करता है ।
• झील और इसके आसपास के क्षेत् को पुनलकट झील पिी अभयारण्य के रूप में नावमत वकया गया है ।
• झील के पास नेलापट् टू पक्षी अभयारण्य, एक महत्वपूणण पक्षी ननवास स्थान है ।
विकल्प 3 सही है : वर्िा झील:
• नचिा झील भारत के पूिी तर् पर ओवडशा राज्य में क्तथथत है । यह पुरी शहर के पास ल्कस्थत है ।
• नचिा झील एक लैगून है , जो एनशया का सबसे बडा खारे पानी का लैगून और दु ननया का दू सरा सबसे बडा लैगून है ।
• झील दया नदी के मुहाने पर क्तथथत है
• इसे 1981 में अं तरराष्टरीय महत्व की आद्रण भूनम, रामसर साइट के रूप में नानमत नकया गया था।
• पक्षी: झील 160 से अनिक प्रजानतयों के पनक्षयों के नलए शीतकालीन आश्रय स्थल है , विनमें साइबेररया और मध्य एवशया
तक के प्रिासी पिी भी शावमल हैं । कुछ उल्लेखनीय प्रजानतयों में राजहं स, पेनलकन, इनबस, बत्तख और गीज शानमल हैं ।
• नचिा झील गंभीर रूप से लुप्तप्राय इरावदी डॉल्कल़्िन के आवास के रूप में भी काम करती है ।
• झील में कई द्वीप हैं , नजनमें प्रनसद्ध नालाबन भी शानमल है , जो एक पक्षी अभयारण्य है । कुछ अन्य द्वीपों में कृष्णाप्रसाद,
िेकफास्ट द्वीप, हनीमून द्वीप, कालीजई और सातपाडा द्वीप शानमल हैं ।
विकल्प 4 गलत है : अनसुपा झील:
• अांसुपा झील ओवडशा राज्य के कर्क विले में क्तथथत है । यह राजिानी भुवनेिर से लगभग 50 नकलोमीटर दू र ल्कस्थत है ।
• यह मीठे पानी की झील है , लैगून नहीं। यह एक घोडे की नाल के आकार की बैल की िनुष झील है जो महानदी के घुमावदार
बहाव से बनी है ।
• यह एक र्ैनल द्वारा सीधे महानदी से िुड़ा हुआ है ।
• यह सारं डा और नबष्णुपुर पहानडयों से नघरा हुआ है , दोनों पूवी घाट के नहस्से हैं ।
• इसे 2022 में रामसर साइर्ोां की सूर्ी में शावमल वकया गया है ।
• झील तीन लुप्तप्राय प्रजानतयों इं नडयन स्कीमर, ब्लैक-बेनलड टनण और वागुर, और कमजोर नदी टनण और हे लीकॉप्टर कैटऩिश
को एक सुरनक्षत आवास प्रदान करती है ।
Source: https://rsis.ramsar.org/ris/1207
https://forests.ap.gov.in/pulicat.php
https://timesofindia.indiatimes.com/travel/destinations/andhra-pradeshs-kolleru-lake-is-a-wildlife-
hotspot-heres-why/articleshow/96503892.cms
https://rsis.ramsar.org/RISapp/files/RISrep/IN229RIS.pdf
https://rsis.ramsar.org/ris/2487#:~:text=Ansupa%2C%20a%20small%20freshwater%20oxbow,scenic%20be
auty%20and%20rich%20biodiversity.

Q.30)
Ans) b
Exp) विकल्प b सही उत्तर है .

Forum Learning Centre: Delhi - 2nd Floor, IAPL House, 19 Pusa Road, Karol Bagh, New Delhi - 110005 | Patna - 2nd floor, AG Palace, E Boring Canal
Road, Patna, Bihar 800001 | Hyderabad - 1st & 2nd Floor, SM Plaza, RTC X Rd, Indira Park Road, Jawahar Nagar, Hyderabad, Telangana 500020
9311740400, 9311740900 | https://academy.forumias.com | admissions@forumias.academy | helpdesk@forumias.academy

[24]
PTS 2024 | Test Code: 121405 - Solutions |

काबणन प्रकृनत में व्यापक रूप से पाया जाता है , अपने शुद्ध रूप में और अन्य तत्वों के साथ संयुि रूप में। काबणन कोयला,
ग्रेफाइट और हीरे सनहत नवनभन्न रूपों में मौजूद है । हीरा और ग्रेफाइट काबणन की दो प्रनसद्ध निस्टलीय संरचनाएाँ हैं ।
कथन 1 सही है : हीरे में विस्टलीय िाली सांरर्ना होती है । हीरे में , प्रत्येक काबचन परमाणु sp3 सांकरण से गुिरता है
और टे टराहे डरल व्यवस्था में चार अन्य काबणन परमाणुओं से बंि जाता है । संरचना अंतररक्ष में फैली हुई है , नजससे काबणन
परमाणुओं का एक कठोर नत्-आयामी नेटवकण बनता है । पूरे जाली में हीरे के नदशात्मक सहसंयोजक बंिन इसे तोडना बेहद
कनठन बनाते हैं , नजससे यह पृ थ्वी पर ज्ञात सबसे कठोर पदाथण बन जाता है ।

कथन 2 गलत है : कोरां डम प्राकृवतक रूप से पाया िाने िाला दू सरा सबसे कठोर ज्ञात पदाथच है , इसके बाद पुखराि,
क्वार्टच ि आवद हैं। इसके नवपरीत, ग्रेफाइट इसकी परतों के बीच आसानी से नचपक जाता है , नजससे यह नरम और नफसलन भरा
हो जाता है । इस कारण से , ग्रेफाइट का उपयोग आमतौर पर उच्च तापमान वाली मशीनरी में सूखे स्नेहक के रूप में नकया जाता
है जहां तेल स्नेहक अनुपयुि होते हैं ।
कथन 3 ग़लत है : हीरे में एक विशाल सहसांयोिक सांरर्ना होती है विसमें प्रत्येक काबचन परमाणु मिबूत सहसांयोिक
बांधोां द्वारा र्ार अन्य काबचन परमाणुओ ां से िुड़ा होता है और कोई मुक्त इलेक्टरॉन नही ां छोड़ता है । यह हीरे को नबजली का
उत्कृष्ट कुचालक बनाता है , नजसका अथण है नक वे नबजली के खराब संवाहक हैं । दू सरी ओर, ग्रेफाइट नबजली का एक अच्छा
संवाहक है क्ोंनक ग्रेफाइट में इलेक्टरॉन गनतशील होते हैं और इनका उपयोग बैटरी और औद्योनगक इलेक्टरोनलनसस में इलेक्टरोड
के नलए नकया जाता है ।
कथन 4 सही है : हीरा अपने मिबूत सहसांयोिक बांधन के कारण ऊष्मा का अच्छा सांिाहक है , और ग्राफीन भी उच्च
तापीय र्ालकता प्रदवशचत करता है । वास्तव में, ग्राफीन की एक परत में काबणन नैनोट्यूब (सीएनटी) और हीरे की तुलना में
बेहतर तापीय चालकता होती है ।
Source: https://www.nps.gov/articles/mohs-hardness-
scale.htm#:~:text=The%20title%2C%20Mohs%20Hardness%20Scale,2%3B%20and%20Talc%2C%201.
https://ciet.nic.in/moocspdf/Chemistry02/kech_21102_etext.pdf (Pg no 7)
https://www.rsc.org/images/Diamond%20Materials%20-
%20Diamonds%20Are%20For%20Everything_tcm18-206502.pdf (Pg no 49)
https://www.ncbi.nlm.nih.gov/pmc/articles/PMC8326494/#:~:text=many%20applications.%20A-,Single,-
%2Dlayer%20of%20graphene
https://phys.org/news/2020-10-scientists-electrifying-diamond.html#:~:text=are%20also%20exceptional-
,thermal,-conductors%20and%20electrical

Forum Learning Centre: Delhi - 2nd Floor, IAPL House, 19 Pusa Road, Karol Bagh, New Delhi - 110005 | Patna - 2nd floor, AG Palace, E Boring Canal
Road, Patna, Bihar 800001 | Hyderabad - 1st & 2nd Floor, SM Plaza, RTC X Rd, Indira Park Road, Jawahar Nagar, Hyderabad, Telangana 500020
9311740400, 9311740900 | https://academy.forumias.com | admissions@forumias.academy | helpdesk@forumias.academy

[25]
PTS 2024 | Test Code: 121405 - Solutions |

Q.31)
Ans) c
Exp) विकल्प c सही उत्तर है ।
विकल्प a गलत है : िैि उपर्ारण वमट्टी, भूिल, कीर्ड़ और ठोस पदाथों में काबचवनक सांदूिकोां को नष्ट करने के वलए
सूक्ष्मिीिोां का उपयोग करता है । सूक्ष्मजीव प्रदू षकों को ऊजाण स्रोत के रूप में उपयोग करके उन्हें तोड दे ते हैं ।
विकल्प b गलत है : बायोमैविवफकेशन को खाद्य िाल के माध्यम से पदाथों के सांर्य और थथानाांतरण के रूप में
पररभावित वकया गया है । इसके पररणामस्वरूप खाद्य श्रृंखला में अगले स्तरों पर जीवों में नवषाि पदाथों की सां द्रता बढ़ जाती
है ।
विकल्प c सही है : बायोफोवर्च वफकेशन में उच्च स्तर के विर्ावमन, खवनि, या उच्च प्रोर्ीन और स्वथथ िसा के साथ
फसलोां का प्रिनन शावमल है । सावणजननक स्वास्थ्य में सुिार के नलए बायोफोनटण नफकेशन को सबसे व्यावहाररक सािन माना
जाता है । उदाहरण के नलए, भारतीय कृनष अनुसंिान संस्थान, नई नदल्ली ने कई सब्जी फसलें भी जारी की हैं जो नवटानमन और
खननजों से भरपूर हैं , जैसे, नवटानमन A से समृद्ध गाजर, पालक, कद् दू आनद है ।
विकल्प d गलत है : िैि सांर्य एक िीि में रसायनोां के सांर्य की एक प्रविया है िो तब होती है िब सेिन की दर
उत्सिचन की दर से अवधक हो िाती है ।
Source: https://ncert.nic.in/ncerts/l/lebo109.pdf

Q.32)
Ans) a
Exp) विकल्प a सही उत्तर है ।
ग्रीन एनिी कॉररडोर (GEC) पररयोिना 2015 में लागू की गई थी। इसका उद्दे श्य नवीकरणीय संसािनों से उत्पानदत
नबजली को नग्रड में पारं पररक नबजली स्टे शनों के साथ नसंिनाइज करना है । इसमें अंतर-राज्य और अंतर-राज्य दोनों टर ां सनमशन
नसस्टम शानमल हैं , जो दे श के भीतर नवीकरणीय ऊजाण (RE) अनिशेष क्षेत्ों से घाटे वाले क्षेत्ों में नबजली के नवतरण को सक्षम
करे गा। उदाहरण के नलए, हाल ही में, भारत सरकार ने लद्दाख में 13 गीगािॉर् निीकरणीय ऊिाच पररयोिना के वलए
िीईसी र्रण- II- अांतर-राज्य र्र ाांसवमशन वसस्टम को मांिूरी दी।
इसमें एक नवीकरणीय ऊजाण प्रबंिन केंद्र (REMC) की स्थापना और भंडारण प्रणानलयों जैसे ननयंत्ण बुननयादी ढां चे की स्थापना
भी शानमल है । यह पहल इसमें ननम्न मदद नमलेगी:
2030 तक 450 गीगावॉट स्थानपत नवीकरणीय ऊजाण क्षमता और गैर-जीवाश्म ईंिन से 500 गीगावॉट स्थानपत नबजली क्षमता
का लक्ष्य प्राप्त करना।
Source: https://forumofregulators.gov.in/Data/study/Report-Green-Energy-Tr.-corridor.pdf
https://mnre.gov.in/green-energy-corridor-0verview/
https://powermin.gov.in/en/content/green-energy-corridor
https://www.financialexpress.com/business/sustainability-cabinet-nod-to-rs-22774-cr-green-energy-
corridor-in-ladakh-3276988/
https://www.livemint.com/news/india/cabinet-approves-rs-20-773-70-crore-for-13-gw-green-energy-
corridor-project-in-ladakh-11697639781976.html

Q.33)
Ans) c
Exp) विकल्प c सही उत्तर है ।
विकल्प 1 सही है : बुगुन वलओवसर्ला (Liocichla bugunorum) एक पिी प्रिावत है विसे सांकर्ग्रस्त प्रिावतयोां की
IUCN लाल सूर्ी द्वारा गांभीर रूप से लुप्तप्राय घोवित वकया गया है । यह नवशेष रूप से ईगलनेस्ट वन्यजीव अभयारण्य

Forum Learning Centre: Delhi - 2nd Floor, IAPL House, 19 Pusa Road, Karol Bagh, New Delhi - 110005 | Patna - 2nd floor, AG Palace, E Boring Canal
Road, Patna, Bihar 800001 | Hyderabad - 1st & 2nd Floor, SM Plaza, RTC X Rd, Indira Park Road, Jawahar Nagar, Hyderabad, Telangana 500020
9311740400, 9311740900 | https://academy.forumias.com | admissions@forumias.academy | helpdesk@forumias.academy

[26]
PTS 2024 | Test Code: 121405 - Solutions |

(अरुणाचल प्रदे श) में पाया जाता है जो इसे भारत के नलए स्थाननक बनाता है । हाल ही में अरुणाचल प्रदे श में ईगलनेस्ट वन्यजीव
अभयारण्य के पास रहने वाली एक प्रमुख जनजानत बुगुन ने बुगुन नलओनसचला पक्षी के संरक्षण के नलये 1,470 हे क्टेयर भूनम
दान की है ।
विकल्प 2 और 3 सही हैं : भारतीय प्राणी सिेिण (ZSI) ने "भारत के 75 थथावनक पिी" शीिचक से एक प्रकाशन िारी
वकया है । ररपोर्च में कहा गया है वक भारत में कुल 1,353 पिी प्रिावतयोां में से 78 (5%) दे श में थथावनक हैं । इन स्थाननक
पनक्षयों में नहमालयन क्वेल और जेडणन कोसणर हैं , दोनों को IUCN द्वारा गंभीर रूप से लुप्तप्राय के रूप में सूचीबद्ध नकया गया है ।
नहमालयन क्वेल को 1876 के बाद से नही ं दे खा गया है , जबनक जेडणन के कोसणर को आल्कखरी बार 2009 में दे खा गया था।
Source: https://eaglenestmemoryproject.in/?page_id=201#:~:text=)%2C%20found%20nowhere-,else,-
in%20the%20world
https://www.thehindu.com/sci-tech/energy-and-environment/5-of-birds-in-india-are-endemic-reveals-
zoological-survey-of-india-publication/article67162268.ece#:~:text=in%201907%3B%20the-,Himalayan,-
Quail%20(Ophrysia

Q.34)
Ans) d
Exp) विकल्प d सही उत्तर है।
विकल्प 1 सही है : ग्लोबल बायोफ्यूल एलायांस (GBA) सांयुक्त राज्य अमेररका और िािील के सहयोग से भारत के
नेतृत्व में एक सहयोगी पहल है । भारत की G20 अध्यक्षता में नई नदल्ली में आयोनजत G20 नशखर सम्मेलन 2023 के दौरान
इसका आनिकाररक उद् घाटन नकया गया। जीबीए वैकल्कल्पक ऊजाण स्रोत के रूप में नटकाऊ जैव ईंिन के नवकास और तैनाती
को बढ़ावा दे ता है ।
विकल्प 2 सही है : आपदा प्रवतरोधी बुवनयादी ढाांर्े के वलए गठबांधन (CDRI) की थथापना 2019 में भारत द्वारा की गई
थी। CDRI का लक्ष्य उनकी आपदा और जलवायु लचीलापन सुनननित करने के नलए बुननयादी ढां चे प्रणानलयों को उन्नत करना
है । यह सतत नवकास लक्ष्यों, जलवायु पररवतणन पर पेररस समझौते और सेंडाई िेमवकण के अनुरूप है ।
विकल्प 3 सही है : भारत ने दु बई, सांयुक्त अरब अमीरात में COP28 सांयुक्त राष्टर िलिायु पररितचन सम्मेलन में ग्लोबल
ग्रीन िेवडर् पहल शुरू की। ग्रीन िेनडट कायणिम व्यल्कियों को पेड लगाने , जल संरक्षण, अपनशष्ट प्रबंिन और वायु प्रदू षण
को कम करने जैसे पयाण वरण-अनुकूल कायों में संलग्न होकर िेनडट अनजणत करने की अनुमनत दे ता है । इस कायणिम का उद्दे श्य
उन गनतनवनियों को प्रोत्सानहत करना है नजनका पयाण वरण पर सकारात्मक प्रभाव पडता है ।
विकल्प 4 सही है : अां तराचष्टरीय सौर गठबां धन (ISA) भारत और फ्ाांस द्वारा सौर ऊिाच समाधानोां को बढािा दे कर
िलिायु पररितचन से वनपर्ने के वलए बनाया गया था। इसकी उत्पनत्त वषण 2015 में पेररस में जलवायु पररवतणन पर संयुि
राष्टर िेमवकण कन्वें शन (UNFCCC) के नलये पानटण यों के 21वें सम्मेलन (COP21) के दौरान हुई थी। 2020 से, संयुि राष्टर के सभी
सदस्य राज्य ISA में शानमल हो सकते हैं , और वतणमान में, 116 दे शों ने ISA िेमवकण समझौते पर हस्ताक्षर नकए हैं ।
Source:
https://www.fao.org/one-country-one-priority-product/ocop-countries/pilot-
countries/en#:~:text=OCOP%20demonstration-,country,-projects
https://sdgs.un.org/partnerships/coalition-disaster-resilient-infrastructure-
cdri#:~:text=CDRI)%20%E2%80%93%20established%20in-,2019,-under%20the%20leadership
https://energy.economictimes.indiatimes.com/news/oil-and-gas/india-launches-global-biofuel-alliance-
at-g20-a-paradigm-shift-in-sustainable-energy-and-economic-strategy/103547199
https://pib.gov.in/PressReleasePage.aspx?PRID=1989495
https://isolaralliance.org/about/background

Forum Learning Centre: Delhi - 2nd Floor, IAPL House, 19 Pusa Road, Karol Bagh, New Delhi - 110005 | Patna - 2nd floor, AG Palace, E Boring Canal
Road, Patna, Bihar 800001 | Hyderabad - 1st & 2nd Floor, SM Plaza, RTC X Rd, Indira Park Road, Jawahar Nagar, Hyderabad, Telangana 500020
9311740400, 9311740900 | https://academy.forumias.com | admissions@forumias.academy | helpdesk@forumias.academy

[27]
PTS 2024 | Test Code: 121405 - Solutions |

Q.35)
Ans) c
Exp) विकल्प c सही उत्तर है ।
बायोचार तकनीक जलवायु पररवतणन को संबोनित करने , नमट्टी की गुणवत्ता बढ़ाने, अपनशष्ट को कम करने और उपोत्पाद के
रूप में ऊजाण उत्पन्न करने का वादा करती है ।
कथन 1 गलत है : Biochar एक ठूलो सतह िेत्रको साथ एक काला, अत्यवधक झरझरा, हिा पदाथच हो। इसमें मुख्य
रूप से नाइर्र ोिन, हाइडर ोिन और ऑक्सीिन िैसे अन्य तत्वोां के साथ लगभग 70% काबचन होता है। यह रासायननक
उपयोग की जाने वाली सामनग्रयों और लागू पायरोनलनसस नवनियों के आिार पर नभन्न होता है ।
कथन 2 सही है : बायोर्ार एक र्ारकोल िैसा पदाथच है िो पायरोवलवसस नामक वनयांवत्रत प्रविया में काबचवनक पदाथों
को िलाने से बनता है । पायरोनलनसस के दौरान, काबणननक पदाथण, जैसे लकडी के नचप्स, पत्ती के कूडे , या मृत पौिों को बहुत
कम ऑक्सीजन वाले कंटे नर में जला नदया जाता है । पायरोनलनसस प्रनिया के दौरान, काबणननक पदाथण बायोचार में पररवनतणत हो
जाता है , जो काबणन का एक ल्कस्थर रूप है जो आसानी से वायुमंडल में नही ं जा सकता है ।
कथन 3 सही है : वमट्टी में बायोर्ार के प्रयोग से नाइर्र ोिन वनिालन को वनयांवत्रत करके वमट्टी की उिचरता में सुधार हो
सकता है । बायोचार नमट्टी के भीतर नाइटर ोजन को बनाए रखने , इसके नुकसान को रोकने और उवणरकों पर ननभणरता को कम
करने में मदद करता है ।
कथन 4 सही है : बायोर्ार उत्पादन में काबचवनक पदाथच शावमल होते हैं िो अन्यथा काबचन डाइऑक्साइड को उत्सविचत
करते हैं यवद स्वाभाविक रूप से विघवर्त होने के वलए छोड़ वदया िाता है । बायोचार उत्पादन काबणन-नकारात्मक है ,
नजसका अथण है नक यह वातावरण में CO2 के स्तर को कम करता है । क्षयकारी पौिे के पदाथण में अल्कस्थर काबणन उत्पादन प्रनिया
के दौरान बायोचार में संग्रहीत ल्कस्थर काबणन में पररवनतणत हो जाता है ।
बायोचार की जलवायु -शमन क्षमता मुख्य रूप से इसकी अत्यनिक पुनगणणना प्रकृनत से उपजी है जो उस दर को िीमा कर दे ती
है नजस पर प्रकाश संश्लेषक रूप से तय काबणन (C) वायुमंडल में वापस आ जाता है
Source: https://regenerationinternational.org/2018/05/16/what-is-
biochar/#:~:text=Biochar%20is%20a%20charcoal%2Dlike,contamination%20and%20safely%20store%20ca
rbon.

Q.36)
Ans) a
Exp) विकल्प a सही उत्तर है ।
युग्म 1 ग़लत है: SEAMA Ecoregion अफ्ीका के दविणी वहस्सोां में क्तथथत है , दविणी मलािी से उत्तरी मोजाक्तिक तक
फैला हुआ है , नॉवडच क िेत्र में नही ां।
• र्र्ाच में क्योां: उदाहरण के नलए, अिीका के दनक्षणी नहस्सों में हाल ही में नकए गए एक अध्ययन से दनक्षण-पूवण अिीका मोंटाने
द्वीपसमूह (SEAMA) नामक एक नए मान्यता प्राप्त पाररल्कस्थनतकी क्षेत् में पहले से अप्रलेल्कखत जैव नवनविता का पता चला है ।
• वििरण: इसमें ग्रेनाइनटक इं सेलबगण शानमल हैं , जो अनद्वतीय पाररल्कस्थनतक तंत् के साथ पृथक चट्टानी बनहखिंड हैं । इन
पाररल्कस्थनतक तंत्ों में वषाण वन और पवणतीय घास के मैदान शानमल हैं ।
युग्म 2 सही है : अफानसी वनवकवतन सीमाउां र् (ANS) वहांद महासागर में क्तथथत है ।
• र्र्ाच में क्योां: भारत ने अफानसी नननकतन सीमाउं ट की खोज के अनिकार के नलए अंतराण ष्टरीय सीबेड अथॉररटी (ISBA) में
आवेदन नकया है । इस कदम का उद्दे श्य रणनीनतक संसािनों को सुरनक्षत करना और क्षेत् में चीनी गनतनवनि का जवाब दे ना है
• थथान: ANS मध्य भारतीय बेनसन (नहं द महासागर में) की एक प्रमुख संरचनात्मक नवशेषता है , िो भारत के तर् से लगभग
3,000 वकमी दू र क्तथथत है ।
• ANS में एक मुख्य पठार शावमल है , िो आसपास के समुद्र तल (4,800 मीर्र) से 1,200 मीर्र ऊपर है । इसके
अवतररक्त, इसमें वद्वतीयक उन्नत समुद्री पिचत ऊाँर्ाइयााँ हैं , विनमें से दो 1,600- और 2,050-मीर्र पानी की गहराई पर

Forum Learning Centre: Delhi - 2nd Floor, IAPL House, 19 Pusa Road, Karol Bagh, New Delhi - 110005 | Patna - 2nd floor, AG Palace, E Boring Canal
Road, Patna, Bihar 800001 | Hyderabad - 1st & 2nd Floor, SM Plaza, RTC X Rd, Indira Park Road, Jawahar Nagar, Hyderabad, Telangana 500020
9311740400, 9311740900 | https://academy.forumias.com | admissions@forumias.academy | helpdesk@forumias.academy

[28]
PTS 2024 | Test Code: 121405 - Solutions |

क्तथथत हैं । इन ऊंचे समुद्री पवणतों में गयोट् स की आकृनत नवज्ञान है , नजससे पता चलता है नक वे समुद्र तल से ऊपर या उसके
करीब बने थे।
• खवनि भांडार: ANS कोबाल्ट, ननकल, मैंगनीज और तां बे123 के भंडार से समृद्ध है ।
युग्म 3 गलत है : अगालेगा द्वीप नहं द महासागर में ल्कस्थत है , प्रशां त महासागर में नहीं।
• र्र्ाच में क्योां: भारत और मॉरीशस ने अगालेगा द्वीप पर एक हवाई पट्टी और एक जेटी का वस्तुतः उद् घाटन नकया। यह
नवकास समुद्री सुरक्षा और द्वीपों से कनेल्कक्टनवटी को बढ़ाता है । भारत की भागीदारी उसके समुद्री र्दनष्टकोण के अनुरूप है और
क्षेत् में सभी के नलए सुरक्षा और नवकास (सागर) के र्दनष्टकोण के प्रनत उसकी प्रनतबद्धता को रे खां नकत करती है ।
• थथान: अगालेगा द्वीप मॉरीशस के उत्तर में लगभग 650 समुद्री मील की दू री पर ल्कस्थत है , नजसमें दो बाहरी द्वीप शानमल हैं ।
• विशेिताएां : ये द्वीप नाररयल के उत्पादन के नलए जाने जाते हैं और अगालेगा डे गेको का घर हैं ।
युग्म 4 ग़लत है : इरे ज िॉवसांग एक सीमा र्ौकी है िो इजराइल को गािा पट्टी से अलग करती है ।
• र्र्ाच में क्योां: इजराइल ने गाजा में अनिक मानवीय सहायता पहुं चाने के नलए उत्तरी गाजा में इरे ज िॉनसंग को अस्थायी रूप से
नफर से खोलने की मंजूरी दे दी।
• यह लोगों की आवाजाही को सुनविाजनक बनाता है
• इजराइल और गाजा के बीच अन्य सीमा पार;
ओ राफा: गाजा से बाहर ननकलने का सबसे सुदूरवती स्थान। इसकी सीमा नमस्र के नसनाई प्रायद्वीप से लगती है ।
o केरे म शालोम: दनक्षणी गाजा और इजराइल के बीच सीमा पार करना। केवल वानणल्कज्यक माल पारगमन के नलए।
युग्म 5 सही है : बर्ागाइका िेर्र रूस के सुदूर पूिच िेत्र में क्तथथत है
• र्र्ाच में क्योां: बटागाइका िेटर, दु ननया का सबसे बडा पमाण िॉस्ट िेटर, पमाण िॉस्ट के नपघलने के कारण खतरनाक दर से
नवस्तार कर रहा है ।
• यह अपनी अनूठी पमाण िॉस्ट नवशेषताओं के नलए जाना जाता है और यह दु ननया का सबसे बडा पमाण िॉस्ट िेटर है ।
Source: https://www.downtoearth.org.in/news/africa/southern-africa-s-new-ecoregion-brimming-with-
undocumented-life-but-in-urgent-need-of-conservation-95244
https://www.thehindu.com/sci-tech/science/sensing-china-threat-india-joins-race-to-mine-new-sea-
patch/article67995397.ece
https://indianexpress.com/article/explained/expert-explains-how-the-development-of-agalega-figures-in-
indias-vision-for-its-maritime-neighbourhood-9191082/lite/
https://indianexpress.com/article/explained/explained-global/rafah-border-crossing-egypt-gaza-
9009527/lite/
https://www.thehindu.com/sci-tech/energy-and-environment/worlds-biggest-permafrost-crater-in-
russias-far-east-thaws-as-planet-warms/article67108866.ece

Q.37)
Ans) d
Exp) विकल्प d सही उत्तर है .
कथन 1 गलत है : प्रशाांत दशकीय दोलन (PDO) कोई अल्पकावलक घर्ना नही ां है ; बक्ति, यह एक दीघचकावलक
िलिायु पैर्नच है । यह गमण और ठं डे चरणों के बीच बदलता रहता है , प्रत्येक चरण लगभग 20-30 वषों तक चलता है । इसमें
उत्तरी प्रशां त महासागर में समुद्री सतह के तापमान (SST) में नभन्नता शानमल है । एल नीनो और ला नीना के नवपरीत, जो कुछ
महीनों से लेकर एक साल तक चलने वाली अल्पकानलक घटनाएं हैं , PDO बहुत लंबे समय के पैमाने पर काम करता है -
आमतौर पर दशकों तक।
कथन 2 गलत है : पीडीओ का सकारात्मक चरण आम तौर पर अनिक एल नीनोस का पक्ष लेता है और अलास्का तट से दू र
उत्तरी प्रशां त क्षेत् में गमण पानी का कारण बनता है । पीडीओ का नकारात्मक चरण ला-ननना की तरह है और अलास्का की पूवी

Forum Learning Centre: Delhi - 2nd Floor, IAPL House, 19 Pusa Road, Karol Bagh, New Delhi - 110005 | Patna - 2nd floor, AG Palace, E Boring Canal
Road, Patna, Bihar 800001 | Hyderabad - 1st & 2nd Floor, SM Plaza, RTC X Rd, Indira Park Road, Jawahar Nagar, Hyderabad, Telangana 500020
9311740400, 9311740900 | https://academy.forumias.com | admissions@forumias.academy | helpdesk@forumias.academy

[29]
PTS 2024 | Test Code: 121405 - Solutions |

खाडी के ठं डे पानी का कारण बनता है । पीडीओ के सकारात्मक चरण के कारण उत्पन्न अल-नीनो जैसी ल्कस्थनतयों के कारण
भारत में मानसून के मौसम के दौरान औसत से कम और औसत से अनिक वषाण नही ं हो सकती है ।
Source: https://www.thehindu.com/sci-tech/energy-and-environment/warming-pacific-points-to-rise-in-
cyclones-over-india-study/article67245627.ece
https://www.ncei.noaa.gov/access/monitoring/pdo/
https://ui.adsabs.harvard.edu/abs/2014ClDy...42.2397K/abstract

Q.38)
Ans) c
Exp) विकल्प c सही उत्तर है ।
भारत, नेपाल, भूर्ान आवद िैसे दे शोां में फैला वहमालय अपने िलिायु िेत्रोां की विशाल श्रृांखला के कारण
िनस्पवतयोां की अविश्वसनीय विविधता का दािा करता है , आिार पर उष्णकनटबंिीय से लेकर उच्चतम ऊंचाई पर स्थायी
बफण तक। इसके पररणामस्वरूप एक स्तररत वनस्पनत संरचना बनती है जो ऊंचाई के साथ बदलती रहती है ।
वनर्ली ऊांर्ाई पर, लगभग 1,000 मीर्र तक, वहमालय की तलहर्ी उष्णकवर्बांधीय और उपोष्णकवर्बांधीय िांगलोां
से ढकी हुई है। इन क्षेत्ों में पाए जाने वाले आम पेडों में साल (शोररया रोबस्टा), सागौन (टे क्टोना ग्रैंनडस), और बां स और
रोडोडें डरोन की नवनभन्न प्रजानतयााँ शानमल हैं ।
िैसे-िैसे कोई ऊपर िाता है , 1,000 से लेकर लगभग 2,000-3,000 मीर्र तक, समशीतोष्ण वन हावी हो जाते हैं । यह
क्षेत् ओक, रोडोडें डरोन और मेपल नचर पाइन की नवनभन्न प्रजानतयों से समृद्ध है ।
समशीतोष्ण िनोां के ऊपर, 3,000 से 4,000 मीर्र के बीर्, वनस्पनत उपअल्पाइन और अल्पाइन क्षेत्ों में पररवनतणत हो
जाती है । यहााँ , जंगल नवरल हो गए हैं , और शंकुिारी वृक्षों का बोलबाला है , नजनमें स्प्रूस, नसल्वर ऩिर और जुननपर की प्रजानतयााँ
शानमल हैं ।
िृि रे खा से परे , 4,000 मीर्र से अवधक की ऊांर्ाई पर, वनस्पनत कठोर जलवायु पररल्कस्थनतयों के अनुकूल अल्पाइन
झानडयों, काई, लाइकेन, जडी-बूनटयों और घास तक सीनमत है ।
विकल्प 1 सही है : ओक के पेड़ िास्ति में आमतौर पर वहमालय में पाए िाते हैं , विशेि रूप से मध्य ऊांर्ाई पर
समशीतोष्ण िेत्र में। नहमालय में ओक की कई प्रजानतयााँ मौजूद हैं , जैसे क्वाकणस ल्यूकोटर ाइकोफोरा और क्वाकणस
सेमेकानपणफोनलया।
विकल्प 2 गलत है : रोजवुड (डालबनगणया), आमतौर पर नहमालय से जुडा नही ं है । यह आमतौर पर उष्णकनटबंिीय और
उपोष्णकनटबंिीय क्षेत्ों (मुख्य रूप से उष्णकनटबंिीय सदाबहार जंगलों में) में पाया जाता है , लेनकन नहमालय में अनिक ऊंचाई
पर नहीं।
विकल्प 3 सही है : यह फूल िाले पौधोां की एक विविध प्रिावत है , और कई प्रिावतयााँ वहमालय िेत्र की मूल वनिासी हैं ।
रोडोडें डरोन संभवतः नहमालय के सबसे प्रनतनष्ठत फूलों में से एक हैं , जो नवनभन्न रं गों में ल्कखलते हैं । वे नवशेष रूप से वसंत और
गनमणयों की शुरुआत में अपने जीवं त प्रदशणन के नलए प्रनसद्ध हैं ।
विकल्प 4 सही है : दे िदार (सेडरस दे िदारा) वहमालय (विशेिकर पविमी वहमालय) में एक अत्यवधक मूल्यिान थथावनक
प्रिावत है । यह मुख्य रूप से समुद्र तल से 6,300 और 9,000 फीट (1,900 और 2,700 मीटर) के बीच उगता है । यह पेड इस
क्षेत् में सांस्कृनतक रूप से भी महत्वपूणण है और ऐनतहानसक रूप से इसका उपयोग मंनदर ननमाण ण और पारं पररक नचनकत्सा में
नकया जाता रहा है ।
विकल्प 5 सही है : वपवसया िांश से सांबांवधत स्प्रूस के पेड़ वहमालय में पाए िाते हैं , विशेि रूप से अवधक ऊांर्ाई पर। वे
ठं डी जलवायु के नलए अनु कूनलत हैं और पवणतीय वनों का एक महत्वपूणण नहस्सा हैं । स्प्रूस के पेड क्षेत् की जैव नवनविता में
योगदान करते हैं , वन्य जीवन का समथणन करते हैं और पाररल्कस्थनतक संतुलन बनाए रखते हैं ।
Source: India a comprehensive geography by D.R Khullar.

Forum Learning Centre: Delhi - 2nd Floor, IAPL House, 19 Pusa Road, Karol Bagh, New Delhi - 110005 | Patna - 2nd floor, AG Palace, E Boring Canal
Road, Patna, Bihar 800001 | Hyderabad - 1st & 2nd Floor, SM Plaza, RTC X Rd, Indira Park Road, Jawahar Nagar, Hyderabad, Telangana 500020
9311740400, 9311740900 | https://academy.forumias.com | admissions@forumias.academy | helpdesk@forumias.academy

[30]
PTS 2024 | Test Code: 121405 - Solutions |

Q.39)
Ans) a
Exp) विकल्प a सही उत्तर है ।
स्तनिारी आम तौर पर अंडे दे ने के बजाय अपने बच्चों को जन्म दे ते हैं । इस प्रनिया को जीवंतता कहा जाता है , जो तब होती है
जब कोई जीव जीनवत संतान को जन्म दे ता है ।
विकल्प 1 गलत है : सील समुद्री स्तनधाररयोां के एक समूह से सांबांवधत हैं विन्ें वपवनपेडटस कहा िाता है , विसका अथच है
वफन या क्तललपर फुर्े ड। वे अंडे नही ं दे ते। इनका कुल गभणकाल लगभग 9 से 11 महीने का होता है ।
विकल्प 2 गलत है : व्हे ल स्तनधारी हैं । वे समुद्री स्तनिाररयों के नसटानसयन समूह का नहस्सा हैं , नजसमें डॉल्कल़्िन और पोपोइज
भी शानमल हैं । स्तनिारी कशेरुकी जानवरों का एक समूह है नजनके बाल होते हैं , वे हवा में सांस लेते हैं , गमण रि वाले होते हैं ,
जीनवत बच्चों को जन्म दे ते हैं और अपने बच्चों को स्तनपान कराते हैं ।
विकल्प 3 गलत है : डॉक्तफफन समुद्री स्तनधाररयोां के वसर्ावसयन पररिार का वहस्सा हैं , नजसमें व्हे ल और पोरपोइज भी
शानमल हैं । डूबने के जोल्कखम को कम करने के नलए डॉल्कल़्िन जीनवत बच्चों को जन्म दे ती हैं ।
विकल्प 4 सही है : मोनोर्र े म स्तनधारी हैं िो िीवित बच्चे पैदा करने के बिाय अांडे दे ते हैं । मोनोटर े म की तीन प्रजानतयााँ
अभी भी मौजूद हैं : प्लैनटपस और छोटी चोंच और लंबी चोंच वाली इनकडना।
Source: https://www.fisheries.noaa.gov/seals-sea-
lions#:~:text=Seals%20and%20sea%20lions%20belong%20to%20a,live%20in%20the%20ocean%2C%20bu
t%20are%20able
https://seaworld.org/animals/all-about/harbor-seal/care-of-young/
https://oceanservice.noaa.gov/facts/dolphin.html#:~:text=The%20blowhole%20on%20top%20of,entire%2
0lives%20in%20the%20water.
https://australian.museum/learn/species-identification/ask-an-expert/what-is-a-monotreme/
https://www.nationalgeographic.com/animals/article/whales-dont-spray-water-from-their-blowholes-
and-other-myths-
debunked#:~:text=Whales%20have%20hair&text=These%20whiskers%20quickly%20disappear%20becaus
e,hidden%20world%20of%20whale%20culture.)

Q.40)
Ans) b
Exp) विकल्प b सही उत्तर है .
ब्लू फ्लैग सनटण नफकेशन एक नवशेष इको-लेबल या प्रमाणन है जो पयाण वरण सम्मान के रूप में दु ननया भर के तटीय स्थानों को
नदया जाता है ।
कथन 1 गलत है : ब्लू फ्लैग प्रमाणन फाउां डेशन फॉर एनिायनचमेंर्ल एिुकेशन (FEE) द्वारा प्रदान वकया िाता है , सी
शेफडच कांििेशन सोसाइर्ी द्वारा नही ां। फाउं डेशन फॉर एनवायनणमेंटल एजुकेशन (FEE) एक गैर-सरकारी, गैर-लाभकारी
संगठन है जो पयाण वरण नशक्षा के माध्यम से सतत नवकास को बढ़ावा दे ता है । इसकी स्थापना 1981 में हुई थी और इसका
मुख्यालय कोपेनहे गन में है ।
कथन 2 सही है : ब्लू फ्लैग साइर्ोां की सालाना समीिा की िाती है और उन्ें एक ऑपरे वर्ां ग सीजन के वलए सम्मावनत
वकया िाता है , जो पूरे एक वषण तक चल सकता है । ब्लू फ्लैग वेबसाइट के अनुसार, स्थानीय अनिकारी टै ग के नलए आवे दन
कर सकते हैं ।
कथन 3 सही है : भारत में अब तक बारह समुद्र तर्ोां को ब्लू फ्लैग मान्यता प्राप्त हुई है। पूवण में टै ग नकए गए लोगों में
गुजरात में नशवराजपुर, दीव में घोघला, कनाण टक में कासरकोड और पदु नबद्री, केरल में कप्पड, आं ध्र प्रदे श में रुनशकोंडा,
अंडमान और ननकोबार में रािानगर, ओनडशा में गोर्ल्न, तनमलनाडु में कोवलम और पुडुचेरी में ईडन शानमल हैं ।

Forum Learning Centre: Delhi - 2nd Floor, IAPL House, 19 Pusa Road, Karol Bagh, New Delhi - 110005 | Patna - 2nd floor, AG Palace, E Boring Canal
Road, Patna, Bihar 800001 | Hyderabad - 1st & 2nd Floor, SM Plaza, RTC X Rd, Indira Park Road, Jawahar Nagar, Hyderabad, Telangana 500020
9311740400, 9311740900 | https://academy.forumias.com | admissions@forumias.academy | helpdesk@forumias.academy

[31]
PTS 2024 | Test Code: 121405 - Solutions |

हाल ही में, लक्षद्वीप के थुंडी और कदमत समुद्र तटों को सूची में जोडा गया है ।
ज्ञानधार:
तथ्य:
• ब्लू फ्लै ग प्रमाणन के नलए आवेदन एक स्वतंत् राष्टरीय जूरी के पास जाता है जो मानकों के अनुपालन की जां च करता है , और
नफर इसे एक अंतरराष्टरीय जू री को भे ज नदया जाता है ।
• अं तराण ष्टरीय जूरी साल में दो बार आवेदकों की समीक्षा करती है , अप्रैल में उत्तरी गोलािण के नलए और नसतंबर में दनक्षणी गोलािण
के नलए।
• इस जूरी में यूनेस्को, नवि पयणटन संगठन (UNWTO) और संयुि राष्टर पयाणवरण कायणिम (UNEP) जैसे प्रमुख वैनिक संगठन
के सदस्य शानमल हैं ।
Source: https://www.thehindu.com/news/national/blue-flag-beach-award-lakshadweep-certificate-
explain/article66064624.ece
https://www.blueflag.global/sdgs-blueflag

Q.41)
Ans) b
Exp) विकल्प b सही उत्तर है .
विकल्प 1 सही है : राष्टरीय बाघ सांरिण प्रावधकरण (NTCA):
• राष्टरीय बाघ संरक्षण प्रानिकरण (NTCA) का गठन िन्यिीि (सांरिण) अवधवनयम, 1972 की धारा 38 L (1) के तहत
वकया गया है ।
• कायच: बाघ संरक्षण के नवनशष्ट अनिदे श के साथ वैिाननक ननकाय।
• अध्यि: पयाण वरण, वन और जलवायु पररवतणन मंत्ी
• महत्वपूणच विम्मेदाररयााँ:
o बाघ सांरिण योिनाओां की तैयारी और कायाचन्वयन की वनगरानी करना
o बाघ िनगणना के माध्यम से बाघोां की आबादी पर नजर रखता है ।
o पूरे भारत में बाघ अभयारण्यों को तकनीकी और नवत्तीय सहायता प्रदान करता है ।
विकल्प 2 गलत है : भारतीय पशु कल्याण बोडण (AWBI):
• थथावपत: 1960 में (पशु िूरता ननवारण अनिननयम, 1960 के तहत)
• कायच: केवल वन्य जीवन ही नहीं बल्कि सभी प्रजानतयों में पशु कल्याण को बढ़ावा दे ता है ।
• मुख्यालय: बल्लभगढ़ (हररयाणा)
• महत्वपूणच विम्मेदाररयााँ:
o प्रयोगशालाओं, सकणस, पालतू जानवरों की दु कानों और प्रदशणन करने वाले जानवरों जैसी नवनभन्न क्षेत्ों में जानवरों की
दे खभाल, पररवहन और दे खभाल के नलए नदशाननदे श जारी करता है ।
o जानवरों के प्रनत करुणा को बढ़ावा दे ता है और िूरता को रोकता है ।
विकल्प 3 सही है : केंद्रीय वर्वड़याघर प्रावधकरण (CZA):
• थथावपत: 1992 में (WPA, 1972 के तहत)
• कायच: पशु कल्याण और संरक्षण सुनननित करने के नलए पूरे भारत में नचनडयाघरों को नवननयनमत करता है ।
• महत्वपूणच विम्मेदाररयााँ:
o नचनडयाघर प्रबंिन, पशु अनिग्रहण और नवननमय, पशु नचनकत्सा दे खभाल, आगंतुक सुरक्षा और जानवरों के नलए संविणन
गनतनवनियों के नलए नदशाननदे श ननिाण ररत करता है ।
o नचनडयाघरों का ननरीक्षण करता है और सुिार के नलए सुिारात्मक कारण वाइयों की नसफाररश करता है ।
विकल्प 4 गलत है : भारतीय िन्यिीि र्र स्ट (WTI)

Forum Learning Centre: Delhi - 2nd Floor, IAPL House, 19 Pusa Road, Karol Bagh, New Delhi - 110005 | Patna - 2nd floor, AG Palace, E Boring Canal
Road, Patna, Bihar 800001 | Hyderabad - 1st & 2nd Floor, SM Plaza, RTC X Rd, Indira Park Road, Jawahar Nagar, Hyderabad, Telangana 500020
9311740400, 9311740900 | https://academy.forumias.com | admissions@forumias.academy | helpdesk@forumias.academy

[32]
PTS 2024 | Test Code: 121405 - Solutions |

• थथापना: 1998 में एक गैर-सरकारी संगठन के रूप में।


• कायच: भारत में वन्य जीवन और उसके आवास का संरक्षण करना।
• प्रमुख नजम्मेदाररयााँ : डब्ल्यूटीआई दे श भर में संरक्षण पररयोजनाओं को लागू करने के नलए सरकारी ननकायों, गैर सरकारी
संगठनों और स्थानीय समुदायों के साथ नमलकर काम करता है ।
विकल्प 5 सही है : राष्टरीय िन्यिीि बोडच (NBWL):
• थथावपत: 2003 में (WPA, 1972 में संशोिन करके) ।
• कायच: भारत में वन्यजीव संरक्षण के नलए सवोच्च ननकाय।
• महत्वपूणच विम्मेदाररयााँ:
o वन्यजीव नीनत, संरनक्षत क्षेत्ों, संरक्षण रणनीनतयों और वन्यजीव-मानव संघषण को कम करने पर केंद्र सरकार को सलाह दे ता
है ।
o राष्टरीय उद्यानों और अभयारण्यों के भीतर या उनके ननकट पररयोजनाओं को मंजूरी दे ता है ।
o राष्टरीय उद्यानों और वन्यजीव अभ्यारण्यों की सीमा में कोई भी बदलाव इसकी मंजूरी के नबना नहीं नकया जा सकता है
o अध्यिता: भारत के प्रिान मंत्ी
o उपाध्यि की अध्यिता: पयाणवरण, वन और जलवायु पररवतणन मंत्ी
विकल्प 6 सही है : वन्यजीव अपराि ननयंत्ण ब्यूरो (WCCB):
• थथावपत: 2006 में, WPA, 1972 में संशोिन करके।
• कायच: WCCB को भारत में संगनठत वन्यजीव अपराि से ननपटने का अनिकार नदया गया है । यह खुनफया जानकारी इकट्ठा
करने, कानून प्रवतणन एजेंनसयों के साथ समन्वय करने और वन्यजीव प्रजानतयों में अवैि नशकार और अवैि व्यापार सनहत
वन्यजीव-संबंनित अपरािों की जां च और अनभयोजन में सहायता करने का काम करता है ।
Source: Environment by Shankar IAS.

Q.42)
Ans) c
Exp) विकल्प c सही उत्तर है
• भूिैज्ञावनक-विरासत थथल िे थथान हैं िो अपनी भूिैज्ञावनक विशेिताओां के वलए महत्वपूणच हैं , विनका िैज्ञावनक,
शैविक, साांस्कृवतक या सौांदयच मूल्य है । ये साइटें आमतौर पर पृथ्वी के इनतहास और भूवैज्ञाननक प्रनियाओं के महत्वपूणण
पहलुओं का प्रनतनननित्व करती हैं । वे अनद्वतीय अंतर्दण नष्ट प्रदान करते हैं , अनुसंिान, नशक्षा, पयणटन और पयाण वरण संरक्षण को
बढ़ावा दे ते हैं ।
• ितचमान में भारत में 34 भू-विरासत थथल हैं
• भारतीय भूवैज्ञाननक सवेक्षण (GSI), खान मंत्ालय के तहत एक प्रमुख भूवैज्ञाननक संगठन, भारत में भू-नवरासत स्थलों की
पहचान और घोषणा के नलए नजम्मेदार है । GSI इन मूल्यवान भूवैज्ञाननक खजानों की पहचान और सुरक्षा के नलए राज्य सरकारों
के साथ नमलकर काम करता है ।
विकल्प c सही है : पां डवुला गुट्टा, एराण मैटी नडब्बालू और बरण कां ग्लोमरे ट सभी को भू -नवरासत स्थलों के रूप में नानमत नकया
गया है ।
पाण्डिुला गुट्टा:
• तेलांगाना के भूपालपल्ली विले में क्तथथत, पाांडिुला गुट्टा एक भूिैज्ञावनक कररश्माई है िो वहमालय से भी पहले का है ।
• 1990 में खोिा गया, इसका एक समृद्ध इवतहास है िो मेसोवलवथक से लेकर मध्यकाल तक फैला हुआ है ।
• यह साइट कई प्रागैनतहानसक ननवास स्थलों को समेटे हुए है , जो इसे पुरातत्वनवदों और इनतहासकारों के नलए एक खजाना
बनाती है ।
• नवशेष रूप से, पां डवुला गुट्टा प्राचीन काल के पुरापाषाणकालीन गुफा नचत्ों का घर है , जो प्रागैनतहानसक ननवानसयों के जीवन
में मूल्यवान अंतर्दण नष्ट प्रदान करते हैं ।

Forum Learning Centre: Delhi - 2nd Floor, IAPL House, 19 Pusa Road, Karol Bagh, New Delhi - 110005 | Patna - 2nd floor, AG Palace, E Boring Canal
Road, Patna, Bihar 800001 | Hyderabad - 1st & 2nd Floor, SM Plaza, RTC X Rd, Indira Park Road, Jawahar Nagar, Hyderabad, Telangana 500020
9311740400, 9311740900 | https://academy.forumias.com | admissions@forumias.academy | helpdesk@forumias.academy

[33]
PTS 2024 | Test Code: 121405 - Solutions |

एराच मैर्ी वडब्बालु:


• एराण मैटी नडब्बालु आं ध्र प्रदे श में नवशाखापत्तनम और भीमुननपट्टनम के बीच ल्कस्थत है ।
• एराण मैटी नडब्बालु अपने लाल रे त के टीलों के नलए प्रनसद्ध है ।
• ये टीले भूवैज्ञाननक महत्व रखते हैं क्ोंनक ये समय के साथ तलछटी प्रनियाओं और पयाण वरणीय पररवतणनों के बारे में जानकारी
प्रदान करते हैं
• ये तटीय लाल तलछट टीले लगभग 12,000 साल पहले समुद्री भूनम संपकण के कारण बने थे।
बरण समूह:
• बरण कां ग्लोमरे ट नदल्ली सुपरग्रुप का नहस्सा है और भारत के राजस्थान के पाली नजले में ल्कस्थत है ।
• बरण कां ग्लोमरे ट एक वास्तनवक तलछटी चट्टान है जो नवनभन्न प्रकार के कंकड और पत्थरों से बनी है । समूह के भीतर ये
नवस्फोट नवनभन्न प्रकार की चट्टानों को प्रदनशणत करते हैं , नजनमें शानमल हैं : क्वाटण जाइट, ग्रेनाइट-गनीस, चटण , मीका नशस्ट।
• भूवैज्ञाननक अध्ययन इसकी अनूठी नवशेषताओं को प्रकट करते हैं और क्षेत् के प्राचीन भूवैज्ञाननक इनतहास की हमारी समझ में
योगदान करते हैं ।
ज्ञानधार:
• भारत के भू -नवरासत स्थलों के कुछ अन्य प्रमुख उदाहरण;
o पविमी घार्: कई राज्यों में फैला, यह यूनेस्को नवि िरोहर स्थल अपनी जैव नवनविता और अनद्वतीय भूवैज्ञाननक नवशेषताओं
के नलए जाना जाता है ।
o हम्पी: कनाण टक में ल्कस्थत, यह दक्कन के पठार के बीच प्राचीन खंडहरों और भूवैज्ञाननक संरचनाओं को समेटे हुए है ।
o एलोरा और अिांता की गुफाएाँ : महाराष्टर में ल्कस्थत, चट्टानों को काटकर बनाए गए ये वास्तुनशल्प चमत्कार प्राचीन काल से
चली आ रही आकषणक भूवैज्ञाननक संरचनाओं को भी प्रदनशणत करते हैं ।
o कच्छ का महान रण: गुजरात में, यह नवशाल लवणीय दलदल नवनशष्ट भूवैज्ञाननक संरचनाओं और रे नगस्तानी पररर्दश्यों को
प्रदनशणत करता है ।
o सेंर् मैरी द्वीप: कनाण टक में ल्कस्थत, इसमें अनद्वतीय बेसाल्ट चट्टान संरचनाएं और हे क्सागोनल स्तंभ हैं ।
o लोनार झील: महाराष्टर में एक खारी सोडा झील, जो उिानपंड के प्रभाव से बनी है , जो प्राचीन मंनदरों और चट्टानों से नघरी हुई
है ।
o भीमबेर्का रॉक शेल्टर: मध्य प्रदे श में ल्कस्थत, इसमें हजारों साल पुराने प्रागैनतहानसक रॉक पेंनटं ग और भूवैज्ञाननक संरचनाएं
हैं ।
o धोलािीरा: गुजरात में, यह खंडहरों और भूवैज्ञाननक संरचनाओं वाला एक प्राचीन हडप्पा पुराताल्कत्वक स्थल है ।
Source: https://vikaspedia.in/education/childrens-corner/geological-heritage-sites-of-india
https://www.thehindu.com/news/national/telangana/pandavula-gutta-designated-exclusive-geo-
heritage-site-in-telangana/article67949723.ece
https://www.geosocindia.org/index.php/jgsi/article/view/65201
https://www.thehindu.com/news/cities/Visakhapatnam/vizags-erra-matti-dibbalu-cries-for-
attention/article67205817.ece#:~:text=The%20Erra%20Matti%20Dibbalu%2C%20which,the%20land%20d
uring%20the%20LGM.

Q.43)
Ans) b
Exp) विकल्प b सही उत्तर है।
कथन 1 सही है : कृवि िेत्र िास्ति में नाइर्र स ऑक्साइड (N2O) उत्सिचन में सबसे बड़ा मानििवनत योगदानकताच है ,
िो िैवश्वक मानििवनत N2O उत्सिचन का लगभग तीन-र्ौथाई है ।

Forum Learning Centre: Delhi - 2nd Floor, IAPL House, 19 Pusa Road, Karol Bagh, New Delhi - 110005 | Patna - 2nd floor, AG Palace, E Boring Canal
Road, Patna, Bihar 800001 | Hyderabad - 1st & 2nd Floor, SM Plaza, RTC X Rd, Indira Park Road, Jawahar Nagar, Hyderabad, Telangana 500020
9311740400, 9311740900 | https://academy.forumias.com | admissions@forumias.academy | helpdesk@forumias.academy

[34]
PTS 2024 | Test Code: 121405 - Solutions |

• नाइटर स ऑक्साइड कृनष गनतनवनियों जैसे नसंथेनटक उवणरकों के उपयोग, पशुिन प्रबंिन (नवशेष रूप से पशु अपनशष्ट के
माध्यम से), और चावल की खेती जैसी कृनष प्रथाओं के दौरान उत्सनजणत होता है ।
• इन गनतनवनियों से नमट्टी में नाइटर ोजन यौनगक ननकलते हैं , नजन्हें माइिोनबयल प्रनियाओं के माध्यम से नाइटर स ऑक्साइड में
बदला जा सकता है ।
• नाइटर स ऑक्साइड एक शल्किशाली ग्रीनहाउस गैस है नजसमें काबणन डाइऑक्साइड की तुलना में बहुत अनिक ग्लोबल वानमिंग
क्षमता है , हालां नक यह कम मात्ा में उत्सनजणत होती है ।
• इसनलए, कृनष से नाइटर स ऑक्साइड उत्सजणन को कम करने के प्रयास जलवायु पररवतणन को संबोनित करने में महत्वपूणण हैं ।
कथन 2 गलत है : मानििवनत मीथेन उत्सिचन का सबसे बड़ा स्रोत िास्ति में कृवि िेत्र है । यह सभी मानििवनत
मीथेन उत्सिचन के लगभग आधे के वलए विम्मेदार है । इसके बाद ऊजाण क्षेत् (मानवजननत मीथेन उत्सजणन का दू सरा सबसे
बडा योगदानकताण ) है , नजसमें कोयला, तेल, प्राकृनतक गैस और जैव ईंिन से उत्सजणन शानमल है ।
• चावल की खेती, पशुिन खेती (आं त् नकण्वन और खाद प्रबंिन), और बायोमास जलाने सनहत नवनभन्न कृनष गनतनवनियों के
दौरान मीथेन जारी नकया जाता है , जो जलवायु पररवतणन और वायुमंडलीय वानमिंग में योगदान दे ता है ।
कथन 3 सही है : िायुमांडल में सल्फर डाइऑक्साइड (SO2) का सबसे बड़ा मानििवनत स्रोत िास्ति में वबिली सांयांत्रोां
और अन्य औद्योवगक सुविधाओां में िीिाश्म ईांधन का िलना है । SO2 मुख्य रूप से नबजली संयंत्ों और औद्योनगक
सुनविाओं में कोयला, तेल और प्राकृनतक गै स जै से जीवाश्म ईंिन के दहन के दौरान उत्सनजणत होता है । ये स्रोत सल्फर युि
ईंिन को जलाने या सल्फर युि कच्चे माल के प्रसंस्करण के माध्यम से सल्फर डाइऑक्साइड छोडते हैं । SO2 उत्सजणन से
अम्लीय वषाण , िुंि का ननमाण ण और िसन संबंिी समस्याएं हो सकती हैं , नजससे यह ननगरानी और नवननयमन के नलए एक
महत्वपूणण प्रदू षक बन जाता है ।
Source: https://www.iea.org/reports/methane-tracker-2021/methane-and-climate-change
https://www.epa.gov/so2-pollution/sulfur-dioxide-basics
https://www.thehindu.com/news/national/india-largest-so2-emitter-in-world-
greenpeace/article29143646.ece#:~:text=The%20largest%20source%20of%20SO2,plants%20and%20othe
r%20industrial%20facilities.
https://www.epa.gov/co-pollution/basic-information-about-carbon-monoxide-co-outdoor-air-
pollution#:~:text=The%20greatest%20sources%20of%20CO,can%20affect%20air%20quality%20indoors.
https://www.epa.gov/ghgemissions/global-greenhouse-gas-emissions-data
https://www.frontiersin.org/articles/10.3389/fenvs.2021.689985
https://www.frontiersin.org/articles/10.3389/fsufs.2020.518039/full#:~:text=The%20composition%20of%2
0gases%20emitted,al.%2C%20in%20press
https://www.greenpeace.org/india/en/press/4015/india-largest-so2-emitter-in-the-world-says-
greenpeaces-new-analysis/

Q.44)
Ans) c
Exp) विकल्प c सही उत्तर है ।
र्ौधरी र्रण वसांह (1902 -1987) उत्तर प्रदे श के पहले गैर-काांग्रेसी मुख्यमांत्री 1979 में भारत के प्रधान मांत्री के पद तक
पहुां र्े। नकसान अनिकारों के नलए एक नदग्गज के रूप में प्रनसद्ध उन्हें एक अनद्वतीय राजनीनतक वगण बनाने का श्रेय नदया जाता है
नजसने उत्तर भारत में कृनष समुदायों को शानमल नकया ।
कथन 1 सही है : भारत सरकार ने वकसानोां के अवधकारोां और लोकताांवत्रक मूल्योां के प्रवत उनकी प्रवतबद्धता को
पहर्ानते हुए हाल ही में पूिच प्रधान मांत्री र्ौधरी र्रण वसांह को मरणोपराांत भारत रत्न से सम्मावनत वकया। इसनलए, यह
दावा नक उन्हें अपने जीवनकाल के दौरान भारत रत्न पुरस्कार से सम्माननत नकया गया था, सही नही ं है ।

Forum Learning Centre: Delhi - 2nd Floor, IAPL House, 19 Pusa Road, Karol Bagh, New Delhi - 110005 | Patna - 2nd floor, AG Palace, E Boring Canal
Road, Patna, Bihar 800001 | Hyderabad - 1st & 2nd Floor, SM Plaza, RTC X Rd, Indira Park Road, Jawahar Nagar, Hyderabad, Telangana 500020
9311740400, 9311740900 | https://academy.forumias.com | admissions@forumias.academy | helpdesk@forumias.academy

[35]
PTS 2024 | Test Code: 121405 - Solutions |

कथन 2 सही है : र्ौधरी र्रण वसांह ने काांग्रेस पार्ी के बाहरी समथचन से 28 िुलाई 1979 से 14 िनिरी 1980 तक
भारत के प्रधान मांत्री के रूप में कायच वकया। हालााँ नक, असहमनत के कारण, कां ग्रेस ने लोकसभा में नविास मत होने से पहले
अपना समथणन वापस ले नलया, नजसके पररणामस्वरूप राष्टरपनत द्वारा सरकार को भंग कर नदया गया। नतीजतन, चरण नसंह को
भारत के एकमात् प्रिान मंत्ी होने का गौरव प्राप्त है नजन्होंने अपने कायणकाल के दौरान कभी भी संसद को संबोनित नहीं नकया।
कथन 3 गलत है : र्रण वसांह स्वतांत्र भारत के पहले कृवि मांत्री नही ां हैं । बल्कि डॉ. राजेंद्र प्रसाद दे श के पहले कृनष मंत्ी और
स्वतंत् भारत के पहले राष्टरपनत हैं ।
Source: https://theprint.in/pageturner/excerpt/charan-singh-the-only-indian-pm-who-did-not-face-
parliament-even-once/265331/
https://thewire.in/history/charan-singh-a-true-champion-of-the-indian-farmer
https://icar.org.in/node/12375#:~:text=Ratna%2C%C2%A0Dr.%20Rajendra-,Prasad,-
%2C%20the%20first%20Agriculture
https://indianexpress.com/article/explained/explained-history/bharat-ratna-chaudhary-charan-singh-
9152952/

Q.45)
Ans) c
Exp) विकल्प c सही उत्तर है ।
राज्य के नीवत वनदे शक वसद्धाांत (DPSP) उन आदशों और आकाांिाओां का प्रतीक हैं विनकी कल्पना सांविधान
वनमाचताओां ने दे श के सामाविक-आवथचक विकास के वलए की थी।
िे सांविधान के भाग IV में वनवहत हैं , विसमें अनुच्छेद 36 से 51 शावमल हैं ।
हालाांवक ये वसद्धाांत गैर-न्यायसांगत हैं , ये एक कल्याणकारी राज्य की स्थापना के लक्ष्य के साथ दे श के शासन को
ननदे नशत करने में महत्वपूणण भूनमका ननभाते हैं ।
DPSP आयररश सांविधान से प्रेररत थे , िो स्वयां स्पेवनश सांविधान से वलया गया था।
इन्हें भारत में केंद्र और राज्य सरकारों को अनिक न्यायसंगत और न्यायपूणण समाज बनाने के उद्दे श्य से कानून और नीनतयां
बनाने के नलए मागणदशणन करने के नलए ननमाण ण नकया गया है ।
अनुच्छेद 37: इस अनुच्छेद में स्पष्ट रूप से उल्लेख नकया गया है नक DPSP "नकसी भी अदालत द्वारा लागू नहीं नकया
जाएगा," नफर भी वे "दे श के शासन में मौनलक हैं ।" इसका तात्पयण यह है नक हालां नक सरकार को उनका पालन करने के नलए
मजबूर नही ं नकया जा सकता है , नफर भी वे राज्य की नीनत के ननमाण ण के नलए आवश्यक नदशाननदे श हैं ।
DPSP को उनकी वसद्धाांतोां और उनके द्वारा समवथचत दशचन के आधार पर मोर्े तौर पर तीन प्रकारोां में िगीकृत
वकया िा सकता है :
• समाििादी वसद्धाांत: इसका उद्दे श्य सामानजक और आनथणक न्याय प्रदान करना और एक कल्याणकारी राज्य की नदशा में
मागण प्रशस्त करना है ।
• गाांधीिादी वसद्धाांत: महात्मा गां िी के नवचारों को दशाण ते हुए, ये नसद्धां त ग्रामीण नवकास, सत्ता के नवकेंद्रीकरण, कुटीर उद्योगों
को बढ़ावा दे ने और आत्मननभणरता पर केंनद्रत हैं ।
• उदार-बौक्तद्धक वसद्धाांत: ये DPSP उदारवाद की नवचारिारा का प्रनतनननित्व करते हैं
युग्म 1 गलत है : राज्य के नीनत ननदे शक नसद्धां त: ग्राम पंचायतों को संगनठत करना और उन्हें स्वशासन की इकाइयों के रूप में
कायण करने में सक्षम बनाना (अनुच्छेद 40)
• विधान: 73वां संवैिाननक संशोिन अनिननयम (1992), (न नक 74वां संवैिाननक संशोिन अनिननयम (शहरी स्थानीय ननकायों
(यूएलबी) को संवैिाननक दजाण प्रदान नकया गया)।
• 73वें संशोिन अनिननयम ने ग्राम, मध्यवती और नजला स्तरों पर पंचायतों की नत्स्तरीय प्रणाली की स्थापना के प्राविान पेश
नकए।

Forum Learning Centre: Delhi - 2nd Floor, IAPL House, 19 Pusa Road, Karol Bagh, New Delhi - 110005 | Patna - 2nd floor, AG Palace, E Boring Canal
Road, Patna, Bihar 800001 | Hyderabad - 1st & 2nd Floor, SM Plaza, RTC X Rd, Indira Park Road, Jawahar Nagar, Hyderabad, Telangana 500020
9311740400, 9311740900 | https://academy.forumias.com | admissions@forumias.academy | helpdesk@forumias.academy

[36]
PTS 2024 | Test Code: 121405 - Solutions |

• इसने इन ग्रामीण स्थानीय ननकायों को संवैिाननक दजाण प्रदान नकया और इन पंचायतों को शल्कियों और नजम्मेदाररयों के
हस्तां तरण का प्राविान नकया, नजससे वे स्व-शासन की इकाइयों के रूप में कायण करने में सक्षम हो गईं।
• इस कानून का उद्दे श्य सत्ता का नवकेंद्रीकरण करना और ननणणय लेने की प्रनिया में स्थानीय समुदायों को सशि बनाना है ,
नजससे स्वशासन के नलए ग्राम पंचायतों को संगनठत करने के ननदे शक नसद्धां त को पूरा नकया जा सके।
युग्म 2 सही है : राज्य के नीवत वनदे शक वसद्धाांत: काम और मातृत्व राहत के नलए उनचत और मानवीय पररल्कस्थनतयों का
प्राविान करना (अनुच्छेद 42)
• विधान: मातृत्व लाभ अनिननयम 1961
युग्म 3 सही है : राज्य के नीनत ननदे शक नसद्धां त: ग्रामीण क्षेत्ों में व्यल्किगत या सहयोग के आिार पर कुटीर उद्योगों को बढ़ावा
दे ना (अनुच्छेद 43)
• विधान: खादी और ग्रामोद्योग आयोग अनिननयम, 1956।
• खादी और ग्रामोद्योग आयोग अनिननयम, 1956, ग्रामीण क्षेत्ों में खादी (हाथ से बुने हुए कपडे ) और ग्रामोद्योग को बढ़ावा दे ने
और नवकनसत करने के नलए अनिननयनमत नकया गया था। कानून ने खादी और ग्रामोद्योग आयोग (KVIC) की स्थापना की, जो
इन उद्योगों की योजना, प्रचार और आयोजन के नलए नजम्मेदार है । कुटीर उद्योगों को समथणन और सहायता प्रदान करके,
अनिननयम का उद्दे श्य रोजगार के अवसर पैदा करना, आनथणक नवकास को बढ़ावा दे ना और ग्रामीण क्षेत्ों में जीवन स्तर में सुिार
करना है , नजससे ग्रामीण क्षेत्ों में कुटीर उद्योगों को बढ़ावा दे ने के ननदे शक नसद्धां त को पूरा नकया जा सके।
युग्म 4 सही है : राज्य के नीवत वनदे शक वसद्धाांत: राज्य की सािचिवनक सेिाओां में न्यायपावलका को कायचपावलका से
अलग करना (अनुच्छेद 50)।
• विधान: आपरानिक प्रनिया संनहता (CRPC), 1973।
• आपरावधक प्रविया सांवहता (CRPC), 1973, एक प्रनियात्मक कानून है जो भारत में आपरानिक न्याय प्रशासन को ननयंनत्त
करता है ।
• जबनक CRPC स्वयं स्पष्ट रूप से न्यायपानलका को कायणपानलका से अलग नहीं करती है , इसके अनिननयमन ने इस ननदे शक
नसद्धां त को प्राप्त करने की नदशा में एक महत्वपूणण कदम उठाया है ।
• CRPC से पहले, कलेक्टर, उप-नवभागीय अनिकारी और तहसीलदार जैसे नजला अनिकारी कायणकारी शल्कियों के साथ-साथ
न्यानयक शल्कियों का भी प्रयोग करते थे।
हालााँ नक, CRPC ने न्यानयक शल्कियों को नवशेष रूप से नजला न्यानयक मनजस्टर े टों के हाथों में सौंपकर न्याय प्रशासन को
पुनगण नठत नकया, जो राज्य उच्च न्यायालय के सीिे ननयंत्ण में काम करते हैं ।
• इस पृथक्करण का उद्दे श्य कायणपानलका के प्रभाव से न्यायपानलका की स्वतंत्ता सुनननित करना था, नजससे न्यायपानलका को
कायणपानलका से अलग करने के ननदे शक नसद्धां त को पूरा नकया जा सके।
Source: Indian Polity- M.Laxmikanth, Chapter-8, DPSP.

Q.46)
Ans) c
Exp) विकल्प c सही उत्तर है ।
कथन-I सही है : भारत का वनयांत्रक एिां महालेखा परीिक (CAG) सांविधान के अनुच्छेद 148 के तहत वनयुक्त एक
सांिैधावनक पदावधकारी है । उनकी प्राथनमक नजम्मेदारी सरकारों, सरकारी ननकायों और राज्य-संचानलत ननगमों की आय और
व्यय का ऑनडट करना है । उनके कतणव्यों का दायरा ननयंत्क और महालेखा परीक्षक (कतणव्य, शल्कियां और सेवा की शतें)
अनिननयम, 1971 में नचनत्त नकया गया है । सरकार का कामकाज संनविान, दे श के कानून, नविानयका और ननयंत्क द्वारा
नवननयनमत होता है । भारत के महालेखा परीक्षक. CAG सरकार द्वारा नकए गए सभी खचों की औनचत्य, वैिता और वैिता की
जां च करता है । CAG का कायाण लय सरकारी खातों और व्यय, नवशेष रूप से योजनाओं के कायाण न्वयन के बाद, पर प्रभावी
ननयंत्ण रखता है । नतीजतन, CAG का कतणव्य व्यय होने के बाद ही उठता है ।
कथन-II गलत है :

Forum Learning Centre: Delhi - 2nd Floor, IAPL House, 19 Pusa Road, Karol Bagh, New Delhi - 110005 | Patna - 2nd floor, AG Palace, E Boring Canal
Road, Patna, Bihar 800001 | Hyderabad - 1st & 2nd Floor, SM Plaza, RTC X Rd, Indira Park Road, Jawahar Nagar, Hyderabad, Telangana 500020
9311740400, 9311740900 | https://academy.forumias.com | admissions@forumias.academy | helpdesk@forumias.academy

[37]
PTS 2024 | Test Code: 121405 - Solutions |

• भारत के सांविधान का अनुच्छेद 149 वनयांत्रक एिां महालेखा परीिक (CAG) के कतचव्योां और शक्तक्तयोां की रूपरे खा
बताता है । यह CAG को नवनशष्ट कतणव्यों का पालन करने और भारत संघ, राज्यों और नकसी भी अन्य ननकाय या प्रानिकरण के
खातों से संबंनित नवशेष शल्कियों का प्रयोग करने का अनिकार दे ता है । ये कतणव्य और शल्कियााँ संसद द्वारा अनिननयनमत नकसी
भी कानून द्वारा नवननयमन के अिीन हैं । इस अनुच्छेद में नवत्तीय लेखापरीक्षा, अनुपालन लेखापरीक्षा या स्वानमत्व लेखापरीक्षा जैसे
शब्दों का स्पष्ट रूप से उल्लेख नहीं नकया गया है ।
• भारत के सांविधान के अनुच्छेद 150 में कहा गया है वक सांघ और राज्य दोनोां के खातोां को भारत के वनयांत्रक और
महालेखा परीिक (CAG) की सलाह के आधार पर राष्टरपवत द्वारा वनधाचररत तरीके से बनाए रखा िाना र्ावहए। इसनलए,
संक्षेप में, राष्टरपनत को खातों का स्वरूप (CAG की सलाह पर) ननिाण ररत करने का अनिकार है ।
Source: https://cag.gov.in/en/page-constitutional-provisions
https://cag.gov.in/en/page-duties-power-and-conditions-of-services-act

Q.47)
Ans) c
Exp) विकल्प c सही उत्तर है ।
• भारत में नकसी नये राज्य को संघ में शानमल करने की शल्कि भारत की संसद के पास है । यह प्रविया भारत के सांविधान के
अनुच्छेद 2 द्वारा शावसत है ।
• अनुच्छेद 2 नए राज्योां के प्रिेश या थथापना से सांबांवधत है । यह कानून द्वारा संसद को नए राज्यों को भारत संघ में शानमल
करने या ऐसे ननयमों और शतों पर नए राज्यों की स्थापना करने का अनिकार दे ता है जो वह उनचत समझे।
• इसके अनतररि, अनुच्छेद 3 नए राज्योां के गठन और मौिूदा राज्योां के िेत्रोां, सीमाओां या नामोां में पररितचन से सांबांवधत
है । यह संसद को नए राज्यों के ननमाण ण, सीमाओं में पररवतणन या मौजूदा राज्यों के नाम के संबंि में कानून बनाने का अनिकार
दे ता है ।
• अतः , सांघ में वकसी नए राज्य के प्रिेश या मौिूदा राज्योां की सीमाओां में पररितचन से सांबांवधत कोई भी वनणचय भारत
की सांसद द्वारा कानून के माध्यम से वलया िाता है ।
• भारत के सांविधान का अनुच्छेद 4 अनुच्छेद 2 और अनुच्छेद 3 से वनकर्ता से सांबांवधत है क्योांवक यह नए राज्योां के
प्रिेश या थथापना और मौिूदा राज्योां के िेत्रोां, सीमाओां या नामोां में पररितचन के उद्दे श्य से बनाए गए कानूनोां से सांबांवधत
है ।
• अनुच्छेद 4 में कहा गया है : "इस संनविान में नकसी भी बात के बावजूद, नए राज्यों के प्रवेश या स्थापना के नलए अनुच्छेद 2
और 3 के तहत बनाए गए कानून को अनुच्छेद 368 के प्रयोजनों के नलए इस संनविान का संशोिन नहीं माना जाएगा।"
• इसका मतलब यह है नक नए राज्यों के प्रवेश या स्थापना, या मौजूदा राज्यों के क्षेत्ों, सीमाओं या नामों में पररवतणन के नलए
अनुच्छेद 2 और 3 के तहत अवधवनयवमत वकसी भी कानून को अनुच्छेद 368 के तहत सांविधान में सांशोधन के रूप में
नही ां माना िाएगा। अनुच्छेद 2 और 3 के तहत बनाए गए कानून संसद के दोनों सदनों के सािारण बहुमत से पाररत नकए जा
सकते हैं ।
Source: Indian Polity, M.Laxmikanth, Chapter-6.

Q.48)
Ans) d
Exp) विकल्प d सही उत्तर है।
कथन 1 गलत है : भारत में बीमा िे त्र का राष्टरीयकरण एक महत्वपूणच मील का पत्थर था। 1956 में, भारत सरकार ने जीवन
बीमा क्षे त् का राष्टरीयकरण करके एक महत्वपू णण कदम उठाया। इस कदम से भारतीय जीवन बीमा ननगम (LIC) की स्थापना हुई।
LIC ने कई भारतीय और गैर-भारतीय बीमाकताण ओं के साथ-साथ प्रोनवडें ट सोसाइनटयों को भी समानहत कर नलया।

Forum Learning Centre: Delhi - 2nd Floor, IAPL House, 19 Pusa Road, Karol Bagh, New Delhi - 110005 | Patna - 2nd floor, AG Palace, E Boring Canal
Road, Patna, Bihar 800001 | Hyderabad - 1st & 2nd Floor, SM Plaza, RTC X Rd, Indira Park Road, Jawahar Nagar, Hyderabad, Telangana 500020
9311740400, 9311740900 | https://academy.forumias.com | admissions@forumias.academy | helpdesk@forumias.academy

[38]
PTS 2024 | Test Code: 121405 - Solutions |

हालााँवक, यह ध्यान रखना आिश्यक है वक सामान्य बीमा िेत्र का राष्टरीयकरण बाद में वकया गया था, विशेि रूप से
1973 में। सामान्य बीमा ननगम (GIC) का गठन भारत में सामान्य बीमा व्यवसाय की दे खरे ख और प्रबंिन के नलए नकया गया
था। GIC ने अपनी सहायक कंपननयों के साथ जनता को गैर-जीवन बीमा सेवाएं प्रदान करने में महत्वपूणण भूनमका ननभाई।
कथन 2 गलत है : बीमा पैठ से तात्पयच वकसी दे श या िेत्र की समग्र अथचव्यिथथा में बीमा िेत्र के योगदान के माप से है ।
इसे आम तौर पर नकसी नदए गए वषण में दे श के सकल घरे लू उत्पाद (GDP) के नलए बीमा प्रीनमयम (या बीमा कवरे ज के मूल्य)
के अनुपात के रूप में व्यि नकया जाता है ।
• 2023 में भारत की बीमा पहुं च लगभग 4% थी।
हालााँ नक, इस पैठ का अनिकां श नहस्सा जीवन बीमा क्षेत् से आता है , िो कुल आां कड़े में लगभग 3.2% का योगदान दे ता है ।
• इसके नवपरीत, गैर-जीवन बीमा (सामान्य बीमा) क्षेत् की पैठ बहुत कम यानी लगभग 1% है ।
• यह असमानता भारत में जीवन और सामान्य बीमा की पैठ के बीच महत्वपूणण अंतर को उजागर करती है ।
• वैनिक तुलना: भारत की बीमा पहुं च वैनिक औसत से कम है । िबवक िीिन बीमा की पैठ िैवश्वक औसत 3% के अनुरूप
है , गैर-िीिन पैठ िैवश्वक औसत 3.9% की तुलना में बेहद कम है ।
कथन 3 गलत है : बीमा क्षेत् में प्रत्यक्ष नवदे शी ननवेश (FDI) की सीमा वास्तव में संशोनित की गई थी, लेवकन 100% तक नही ां।
2021 में , भारत सरकार ने स्वर्ावलत मागच के माध्यम से FDI सीमा को 49% से बढाकर 74% कर वदया। यह बदलाव
बीमा संशोिन नविेयक 2021 के माध्यम से लागू नकया गया था। इस कदम के पीछे का उद्दे श्य अनिक नवदे शी ननवेश को
आकनषणत करना और भारत में बीमा उद्योग की नवकास क्षमता को बढ़ाना था। हालााँ नक, यह ध्यान रखना आिश्यक है वक
FDI सीमा पूणच 100% अांक तक नही ां पहुां र्ी।
Source: https://www.business-standard.com/amp/finance/insurance/bs-bfsi-summit-general-insurance-
sector-eyeing-1-5-penetration-by-2030-123103101198_1.html
https://licindia.in/history#:~:text=The%20demand%20for%20nationalization%20of,insurance%20in%20Ind
ia%20was%20nationalized.
https://irdai.gov.in/evolution-of-
insurance#:~:text=General%20insurance%20business%20was%20nationalized,effect%20from%201st%20J
anuary%2C%201973.
https://m.economictimes.com/industry/banking/finance/insure/whys-the-insurance-industry-largely-
silent-on-100-fdi-demand/articleshow/97399500.cms

Q.49)
Ans) a
Exp) विकल्प a सही उत्तर है ।
भारतीय संघ की नवनभन्न इकाइयों के बीच उत्पन्न होने वाले नववादों में भारत का सवोच्च न्यायालय अंनतम मध्यस्थ के रूप में
उपलब्ध है । इस शल्कि को नवनशष्ट मूल क्षेत्ानिकार (अनुच्छेद 131) के रूप में जाना जाता है , नजसका अथण है नक भारत में नकसी
अन्य अदालत को इन मामलों को पहले सुनने का अनिकार नही ं है ।
यह नवनशष्ट मूल क्षेत्ानिकार तीन नवनशष्ट पररर्दश्यों पर लागू होता है :
• केंद्र बनाम राज्य: जब केंद्र सरकार (संघ) और एक या अनिक राज्य सरकारें को आपस में टकराव हो जाती हैं ।
• बहु-राज्य गवतरोध: एक तरफ केंद्र सरकार और एक या अनिक राज्यों से जुडे नववाद, दू सरी तरफ एक या अनिक राज्यों के
दू सरे समूह के ल्कखलाफ खडे होते हैं ।
• राज्य बनाम राज्य: दो या दो से अनिक राज्य सरकारों के बीच सीिा नववाद। इनमें सीमा मुद्दे, संसािन साझाकरण, या
संबंनित राज्यों को सीिे प्रभानवत करने वाली कोई अन्य असहमनत शानमल हो सकती है ।
सवोच्च न्यायालय के नवनशष्ट मूल क्षेत्ानिकार के संबंि में मुख्य नबंदु:
• विशेि: नकसी अन्य अदालत को इन मामलों की पहले सुनवाई करने का अनिकार नही ं है । सुप्रीम कोटण केंद्र में है .

Forum Learning Centre: Delhi - 2nd Floor, IAPL House, 19 Pusa Road, Karol Bagh, New Delhi - 110005 | Patna - 2nd floor, AG Palace, E Boring Canal
Road, Patna, Bihar 800001 | Hyderabad - 1st & 2nd Floor, SM Plaza, RTC X Rd, Indira Park Road, Jawahar Nagar, Hyderabad, Telangana 500020
9311740400, 9311740900 | https://academy.forumias.com | admissions@forumias.academy | helpdesk@forumias.academy

[39]
PTS 2024 | Test Code: 121405 - Solutions |

• मूल: यह इन नववादों का पहला पडाव है । वे ननचली अदालतों से अपील नही ं हैं ।


• कानूनी मामले : नववाद को कानूनी प्रश्न के इदण -नगदण घूमना चानहए, जो या तो कानून या तथ् पर आिाररत हो, जो कानूनी
अनिकार ननिाण ररत करता हो। यहां राजनीनतक नववादों पर ध्यान नहीं नदया जाता.
• व्यक्तक्तयोां के वलए नही ां: ननजी नागररक इस क्षेत्ानिकार के तहत केंद्र सरकार या राज्य सरकार के ल्कखलाफ मुकदमा दायर
नहीं कर सकते हैं । उन्हें वैकल्कल्पक कानूनी रास्ते अपनाने चानहए।
विकल्प 1 सही है : संनविान का अनुच्छेद 131 राज्यों के बीच नववादों में सवोच्च न्यायालय को नवशेष मूल क्षेत्ानिकार प्रदान
करता है । जब दो राज्य सरकारों के क्षेत्, संसािनों या अन्य मामलों पर परस्पर नवरोिी दावे हों, तो वे सीिे सवोच्च न्यायालय का
दरवाजा खटखटा सकते हैं ।
विकल्प 2 गलत है : जबनक कई मामलों में सुप्रीम कोटण का मूल क्षेत्ानिकार है , अंतर-राज्य जल नववाद एक अपवाद है । अांतर-
राज्य िल वििाद अवधवनयम, 1956 ने इन वििादोां को सांभालने के वलए विशेि न्यायावधकरण (िैसे कािेरी िल वििाद
न्यायावधकरण) की थथापना की। इन न्यायानिकरणों के पास राज्यों के बीच जल आवंटन पर जां च, मध्यस्थता और बाध्यकारी
ननणणय दे ने की शल्कि है । इसनलए 1956 का अंतर-राज्य जल नववाद अनिननयम अंतर-राज्य जल नववादों में सवोच्च न्यायालय को
बाहर करता है ।
विकल्प 3 सही है : अनुच्छेद 131 सिोच्च न्यायालय को सांघ (केंद्र) और राज्य सरकार के बीर् कानून या तथ्य के वकसी
भी प्रश्न के सांबांध में वकसी भी कानूनी वििाद को हल करने का अवधकार दे ता है । सवोच्च न्यायालय का यह क्षेत्ानिकार
मौनलक एवं नवनशष्ट है ।
विकल्प 4 गलत है : संसद और राज्य नविानसभाओं के सदस्यों के चुनाव से संबंनित नववाद उच्च न्यायालय के मूल क्षेत्ानिकार
के अंतगणत आता है , न नक सवोच्च न्यायालय के।
विकल्प 5 गलत है : सांविधान का अनुच्छेद 32 सिोच्च न्यायालय को मौवलक अवधकारोां को लागू करने का मूल अवधकार
िेत्र प्रदान करता है । नागररक ररट यानचकाओं के माध्यम से सीिे न्यायालय से संपकण कर सकते हैं । न्यायालय व्यल्किगत
अनिकारों की रक्षा के नलए बंदी प्रत्यक्षीकरण, परमादे श, अनिकार पृच्छा, ननषेि और उत्प्रेषण जैसे ररट जारी कर सकता है ।
हालााँ नक, यह क्षेत्ानिकार नवनशष्ट नहीं है ; अनुच्छेद 226 उच्च न्यायालयों को उनके क्षेत्ीय अनिकार क्षेत् के भीतर समान ररट
जारी करने का अनिकार दे ता है ।
Source: Indian Polity, M.Laxmikanth, Chapter 30, Supreme Court.

Q.50)
Ans) d
Exp) विकल्प d सही उत्तर है।
प्रत्यक्ष लोकतंत् शासन की एक प्रणाली है नजसमें नागररक ननवाण नचत प्रनतनननियों को अनिकार सौंपने के बजाय सीिे ननणणय लेने
की प्रनियाओं में भाग लेते हैं । प्रत्यक्ष लोकतंत् में , नागररकों को नीनत प्रस्तावों, कानूनों और अन्य मामलों पर वोट दे ने का अवसर
नमलता है जो समुदाय या राष्टर को सीिे प्रभानवत करते हैं , अक्सर पहल, जनमत संग्रह या टाउन हॉल बैठकों के माध्यम से। यह
मॉडल अपनी सरकार की नीनतयों और नदशा को आकार दे ने में नागररकों की सनिय भागीदारी पर जोर दे ता है ।
विकल्प a गलत है : भारत की सांसद दे श की सिोच्च विधायी सांथथा है , विसमें दो सदन शावमल हैं : लोकसभा (लोगोां का
सदन) और राज्यसभा (राज्योां की पररिद)।
• जबनक संसद भारत के लोकतां नत्क नसद्धां तों का प्रनतनननित्व करती है , यह प्रत्यक्ष लोकतंत् के बजाय एक प्रनतनननि लोकतंत्
के रूप में कायण करती है ।
• संसद सदस्यों (सां सदों) को नागररकों द्वारा समय-समय पर चुनावों के माध्यम से चुना जाता है , और उन्हें लोगों की ओर से
कानून और नीनतयां बनाने का काम सौंपा जाता है ।
• इसनलए, संसद प्रत्यक्ष लोकतंत् का उदाहरण नही ं है , बल्कि एक प्रनतनननि लोकतंत् है ।
विकल्प b गलत है : सांसद के समान, भारत में राज्य विधानसभाएां राज्य स्तर पर कानून और नीवतयाां बनाने के वलए
लोगोां द्वारा र्ुने गए प्रवतवनवधयोां की सांथथाएां हैं।

Forum Learning Centre: Delhi - 2nd Floor, IAPL House, 19 Pusa Road, Karol Bagh, New Delhi - 110005 | Patna - 2nd floor, AG Palace, E Boring Canal
Road, Patna, Bihar 800001 | Hyderabad - 1st & 2nd Floor, SM Plaza, RTC X Rd, Indira Park Road, Jawahar Nagar, Hyderabad, Telangana 500020
9311740400, 9311740900 | https://academy.forumias.com | admissions@forumias.academy | helpdesk@forumias.academy

[40]
PTS 2024 | Test Code: 121405 - Solutions |

• वे प्रनतनननि लोकतंत् के नसद्धां तों के तहत काम करते हैं , जहां नागररक समय-समय पर चुनावों के माध्यम से अपने प्रनतनननियों
का चुनाव करते हैं नजन्हें नविान सभा के सदस्यों (MLA) के रूप में जाना जाता है ।
• इसनलए, राज्य नविान सभाएं प्रत्यक्ष लोकतंत् का नही ं बल्कि प्रनतनननि लोकतंत् का उदाहरण हैं ।
विकल्प c गलत है : ग्राम पांर्ायत भारत में ग्रामीण स्तर पर थथानीय स्वशासन सांथथा को सांदवभचत करता है।
• इसमें ननवाण नचत पंचायत सदस्य शानमल होते हैं जो अपने संबंनित अनिकार क्षेत् के भीतर स्थानीय शासन, प्रशासन और
नवकास गनतनवनियों के नलए नजम्मेदार होते हैं ।
• जबनक ग्राम पंचायतें स्थानीय भागीदारी और ननणणय लेने में शानमल होती हैं , नफर भी वे प्रत्यक्ष लोकतंत् के बजाय प्रनतनननि
लोकतंत् के ढां चे के भीतर काम करती हैं ।
• इसनलए, ग्राम पंचायत प्रत्यक्ष लोकतंत् का उदाहरण नहीं है , बल्कि नवकेन्द्रीकृत प्रनतनननि लोकतंत् का एक रूप है ।
विकल्प d सही है : ग्राम सभा एक ग्राम सभा है विसमें ग्राम समुदाय के सभी ियस्क सदस्य शावमल होते हैं ।
• यह भारत के ग्रामीण क्षेत्ों में स्थानीय स्वशासन और प्रत्यक्ष लोकतंत् की प्राथनमक इकाई के रूप में कायण करता है ।
• ग्राम सभा की बैठकों के दौरान, गााँ व के सभी वयस्क सदस्यों को स्थानीय शासन, नवकास पहल, सामानजक कल्याण कायणिमों
और अन्य सामुदानयक मामलों से संबंनित ननणणय लेने की प्रनियाओं में भाग लेने का अवसर नमलता है ।
• चचाण ओं और सवणसम्मनत ननमाण ण के माध्यम से , नबचौनलयों के नबना, ग्रामीणों द्वारा सामूनहक रूप से ननणणय नलए जाते हैं ।
• ग्राम सभा प्रत्यक्ष लोकतंत् के एक रूप का प्रनतनननित्व करती है जहां नागररक सीिे स्थानीय शासन और ननणणय लेने की
प्रनियाओं में भाग लेते हैं , जो इसे भारत में जमीनी स्तर पर प्रत्यक्ष लोकतंत् का एक प्रमुख उदाहरण बनाता है ।
इसनलए, जबनक संसद, राज्य नविान सभाएं और ग्राम पंचायतें प्रनतनननि लोकतंत् के नसद्धांतों के तहत काम करती हैं और
ननवाण नचत प्रनतनननि लोगों की ओर से ननणणय लेते हैं । ग्राम सभा प्रत्यक्ष लोकतंत् का एक उदाहरण है जहां नागररक स्वयं स्थानीय
स्तर पर ननणणय लेने की प्रनियाओं में सीिे भाग लेते हैं ।
Source: https://ncert.nic.in/ncerts/l/lesy203.pdf

Q.51)
Ans) a
Exp) विकल्प a सही उत्तर है ।
भारतीय संविधान में अनुसूवित जनजावतय ं (ST) के अवधकार ं और वहत ं की रक्षा के विए मजबूत सुरक्षा उपाय ं क
शावमि वकया गया है , जो उनकी विविष्ट स ां स्कृविक पहच न और ऐविह वसक ह विए पर हैं । सांिैध वनक प्र िध नोां क उद्दे श्य
एसटी समुद योां के र जनीविक प्रविवनवधत्व, स म वजक-आवथिक विक स और अवििीय विर सि के सांरक्षण को सुवनविि करन
है ।
कथन 1 सही है: भारतीय संविधान के भीतर राज्य के नीवत वनर्दे शक वसद्ांत (DPSP) राज्य क अनुसूवित जनजावतय ं
(ST) के आवथिक वहत ं क बढािा र्दे ने का आर्दे श र्दे ते हैं । DPSP क अनुच्छेद 46 वििेष रूप से एसटी सवहि सम ज के
कमजोर िर्गों के िैवक्षक और आवथिक वहिोां को बढ ि दे ने के र ज्य के कििव्य पर जोर दे ि है , वजसमें उनकी स म वजक और
आवथिक स्थथवियोां को सुध रने पर ध्य न वदय ज ि है ।
कथन 2 गित है : सरकारी नौकररयों में एसटी के नलए पदोन्ननत में आरक्षण का अनिकार मौनलक अनिकार नहीं है (जैसा नक
मुकेश कुमार बनाम उत्तराखंड राज्य के मामले में सुप्रीम कोटण ने बरकरार रखा है ), यह एक संवैिाननक प्राविान है नजसका
उद्दे श्य हानशए पर रहने वाले समुदायों के नलए प्रनतनननित्व और अवसरों को बढ़ावा दे ना है । सरकारी नौकररयों/सावणजननक क्षेत्
में एसटी की तरह।
जल्कस्टस नागेिर राव की अध्यक्षता वाली सुप्रीम कोटण की बेंच ने कहा नक प्रमोशन में आरक्षण का कोई मौनलक अनिकार नही ं है ।
कथन 3 सही है : संविधान के अनुच्छेर्द 164 में राज्य सरकार में मंविय ं (मुख्यमंिी सवहत) की वनयुक्ति के संबंध में
प्रािधान हैं । इसमें प्र िध न है वक छत्तीसर्गढ, झ रखांड, मध्य प्रदे ि और ओवडि र ज्योां में आवदि सी कल्य ण क प्रभ री मांत्री
होर्ग । अिः , वदय र्गय कथन सही है ।

Forum Learning Centre: Delhi - 2nd Floor, IAPL House, 19 Pusa Road, Karol Bagh, New Delhi - 110005 | Patna - 2nd floor, AG Palace, E Boring Canal
Road, Patna, Bihar 800001 | Hyderabad - 1st & 2nd Floor, SM Plaza, RTC X Rd, Indira Park Road, Jawahar Nagar, Hyderabad, Telangana 500020
9311740400, 9311740900 | https://academy.forumias.com | admissions@forumias.academy | helpdesk@forumias.academy

[41]
PTS 2024 | Test Code: 121405 - Solutions |

कथन 4 ग़ित है : सांविध न की अनुसूची V कुछ र ज्योां में जनज िीय क्षेत्रोां के प्रि सन और ि सन से सांबांवधि है । भ रि के
सांविध न में अनुसूची-V क्षेत्रोां में आवदि वसयोां के प्रथ र्गि अवधक रोां की रक्ष के विए ग्र म सभ को दी र्गई वििेष िस्ियोां क
स्पष्ट रूप से उल्लेख नहीां वकय र्गय है । इसके बज य, इन प्र िध नोां को पांच यि (अनुसूवचि क्षेत्रोां िक विस्त र) अवधवनयम,
1996 (PESA अवधवनयम) में उस्ल्लस्खि वकय र्गय है । पेस अवधवनयम सांविध न के प्र िध नोां को अनुसूवचि क्षेत्रोां िक विस्त ररि
करि है और इन क्षेत्रोां के भीिर ग्र म सभ ओां को थथ नीय सांस धनोां और ि सन पर वनयांत्रण सवहि विविष्ट अवधक रोां और
प्र वधकरणोां के स थ सिि बन ि है । पांच यिोां से सांबांवधि सांविध न के भ र्ग IX के प्र िध न अनुसूची-V क्षेत्रोां पर ि र्गू नहीां होिे
हैं ।
कथन 5 गित है : भ रि के र ष्टरपवि के प स बदििी जनस ां स्िकी के आध र पर अनुसूवचि जनज वियोां (ST) की मौजूद सूची
में सांिोधन करने क अवधक र नहीां है । अनुसूवचि जनज वियोां की सूची भ रि के सांविध न के अनुच्छेद 342 के िहि वनवदि ष्ट है ।
इस सूची में वकसी भी सांिोधन, सम िेिन य बवहष्करण के विए सांसद की मांजूरी की आिश्यकि होिी है ।

Q.52
Ans) a
Exp) विकल्प a सही उत्तर है ।
भ रि के सांविध न की स ििीां अनुसूची सांघ और र ज्योां के बीच विध यी िस्ियोां के वििरण को वचवत्रि करने में महत्वपूणि
भूवमक वनभ िी है । इसमें िीन मुि सूवचय ाँ ि वमि हैं : सांघ सूची, र ज्य सूची और समििी सूची।
• सांघ सूची उन विषयोां की र्गणन करिी है वजन पर केिि केंद्रीय सांसद को क नून बन ने क अवधक र है , जैसे रक्ष , विदे िी
म मिे और मुद्र ।
• र ज्य सूची में पुविस, स ििजवनक स्व स्थ्य और थथ नीय सरक र सवहि र ज्य विध नसभ ओां के अवधक र क्षेत्र के म मिे ि वमि
हैं ।
• समििी सूची में ऐसे विषय ि वमि हैं वजन पर सांघ और र ज्य दोनोां एक स थ क नून बन सकिे हैं , जैसे आपर वधक क नून,
विि ह और वदि विय पन।
• इसके अविररि, अनुसूची VII अिविष्ट म मिोां को भी सांबोवधि करिी है , जो स्पष्ट रूप से वकसी भी सूची में ि वमि नहीां हैं ,
और इसविए केंद्रीय सांसद के अवधक र के अांिर्गिि आिे हैं ।
विकल्प 1 सही है : भारत के संविधान की संघ सूिी में शावमि जनगणना, केंद्रीय संसर्द के विशेष विधायी क्षेि के
अंतगित आती है (प्रविवि 69)। इसमें र ष्टरीय और क्षेत्रीय स्तर पर नीवि वनम ि ण और योजन को सूवचि करने के विए जनसांि
आक र, वििरण और स म वजक-आवथिक वििेषि ओां जैसे जनस ां स्िकीय डे ट की आिवधक र्गणन और ररकॉवडिं र्ग ि वमि है ।
भ रि के रवजस्ट्र र जनरि और जनर्गणन आयुि क क य ि िय दिकीय जनर्गणन के सांच िन के विए वजम्मेद र है । यह
क य ि िय र्गृह मांत्र िय क वहस्स है ।
विकल्प 2 गित है : वर्दिािा और वर्दिावियेपन से संबंवधत मामिे समिती सूिी के अंतगित आते हैं । इसविए, सांसद और
र ज्य विध नमांडि दोनोां वदि विय पन और वदि विय पन म मिोां से सांबांवधि क नून बन सकिे हैं ।
विकल्प 3 सही है : 'बैंवकांर्ग', 'भ रिीय ररजिि बैंक', ड कघर बचि बैंक' और 'बीम ' से सांबांवधि म मिे सांघ सूची में ि वमि हैं ,
वजससे सांसद को ऐसे म मिोां पर क नून बन ने क वििेष अवधक र वमिि है ।
विकल्प 4 गित है : कृवष आय पर कर ं से संबंवधत मामिे राज्य सूिी के अंतगित आते हैं, वजससे र ज्य विध नसभ ओां को
ऐसे म मिोां पर क नून बन ने की अनुमवि वमििी है । कृवष आय के अि ि अन्य आय पर कर, वनर्गम कर और अिविष्ट
कर ध न म मिे सांघ सूची के अांिर्गिि आिे हैं ।
विकल्प 5 गित है : 'िन' समििी सूची के अांिर्गिि सूचीबद्ध हैं । 1976 के 42िें संिैधावनक संश धन अवधवनयम ने पांि
विषय ं क राज्य सूिी से समिती सूिी में स्थानांतररत कर वर्दया:
• विक्ष
• िन
• िज़न और म प

Forum Learning Centre: Delhi - 2nd Floor, IAPL House, 19 Pusa Road, Karol Bagh, New Delhi - 110005 | Patna - 2nd floor, AG Palace, E Boring Canal
Road, Patna, Bihar 800001 | Hyderabad - 1st & 2nd Floor, SM Plaza, RTC X Rd, Indira Park Road, Jawahar Nagar, Hyderabad, Telangana 500020
9311740400, 9311740900 | https://academy.forumias.com | admissions@forumias.academy | helpdesk@forumias.academy

[42]
PTS 2024 | Test Code: 121405 - Solutions |

• जांर्गिी ज निरोां और पवक्षयोां क सांरक्षण


• न्य य प्रि सन
Source:
https://cdnbbsr.s3waas.gov.in/s316026d60ff9b54410b3435b403afd226/uploads/2023/02/2023021535.pdf

Q.53)
Ans) c
Exp) विकल्प c सही उत्तर है ।
कथन 1 गलत है : 'शून्य काल' के दौरान, एक सांसद सदस्य (साांसद) केिल पूिच सूर्ना के साथ ही कोई मामला उठा
सकता है । उदाहरण के नलए, लोकसभा में 'शून्य काल' के दौरान मामले उठाने के नलए, सदस्य प्रवतवदन सुबह 8.30 बिे से
9.00 बिे के बीर् अध्यि को नोवर्स दे ते हैं विसमें स्पष्ट रूप से उस वििय का उल्लेख होता है विसे िे महत्वपूणच मानते
हैं और सदन में उठाना र्ाहते हैं । राज्यसभा में भी पूिच सूर्ना का ऐसा ही प्रािधान है ।
भारतीय संसद में शून्यकाल एक अनूठी नवशेिता है विसका उल्लेख न तो सांविधान, न ही नकसी क़ानून या सदन की
प्रनियाओं के ननयमों में नकया गया है । लेनकन नपछले कुछ वषों में , दोनों सदनों के पीठासीन अनिकाररयों ने इसे और भी प्रभावी
बनाने के नलए शून्यकाल के कामकाज को सुव्यवल्कस्थत करने के ननदे श नदए हैं ।
कथन 2 सही है : प्रश्नकाि संसर्द के र्द न ं सर्दन ं की प्रविया और कायि संिािन वनयम ं के अनुसार आय वजत वकया
जाता है । प्रत्येक सांसदीय बैठक क प्र रां वभक घांट प्रश्नक ि के विए समवपिि होि है , वजसके दौर न सांसद सदस्य (स ां सद)
जि बदे ही सुवनविि करने के विए मांवत्रयोां से सि ि करिे हैं । प्रश्नकाि के र्दौरान, प्रश्न वनजी सर्दस् ं (ऐसे सांसर्द ज मंिी
नही ं हैं) की ओर भी वनर्दे वशत वकए जा सकते हैं । 2014 में, राज्यसभा ने अपना प्रश्नकाि सुबह 11 बजे से र्द पहर 12
बजे तक स्थानांतररत कर वर्दया। दोनोां सदनोां, अथ ि ि् र ज्यसभ और िोकसभ के पीठ सीन अवधक ररयोां के प स इसकी
क यिि ही पर अांविम अवधक र है ।
कथन 3 सही है : तारांवकत प्रश्न, तारांकन द्वारा पहिाना जाता है , इसमें मौक्तिक प्रवतविया की आिश्यकता ह ती है ।
क ई सर्दस् उस पर पूरक प्रश्न भी पूछ सकि है । िोकसभ में, मौस्खक उत्तर के विए एक वदन में केिि 20 प्रश्न सूचीबद्ध
वकए ज सकिे हैं , जबवक र ज्यसभ में मौस्खक उत्तर के विए केिि 15 प्रश्न सूचीबद्ध वकए ज सकिे हैं ।
कथन 4 सही है : एक अि र ां वकि प्रश्न क उत्तर मौस्खक रूप से दे ने की आिश्यकि नहीां है और इसक उत्तर विस्खि उत्तर
के म ध्यम से वदय ज ि है । इसके अि ि , उस पर कोई पूरक प्रश्न नहीां पूछ ज सकि है । अिः , वदय र्गय कथन सही है ।
Source: https://cms.rajyasabha.nic.in/UploadedFiles/Procedure/RajyaSabhaAtWork/English/639-
649/CHAPTER19.pdf
https://sansad.in/rs/questions/general-information
https://loksabhadocs.nic.in/our%20parliament/Question%20Hour%20in%20Lok%20sabha.pdf
https://sansad.in/uploads/rule_procedure_56ae75ddab.pdf?updated_at=2022-11-28T11:56:47.059Z

Q.54)
Ans) a
Exp) विकल्प a सही उत्तर है ।
एवशया-प्रशांत आवथिक सहय ग (APEC) एवशया-प्रशांत की बढती परस्पर वनभिरता का िाभ उठाने के विए 1989 में
स्थावपत एक क्षेिीय आवथिक मंि है । APEC क उद्दे श्य सांिुविि, सम िेिी, वटक ऊ, निीन और सुरवक्षि विक स को बढ ि
दे कर और क्षेत्रीय आवथिक एकीकरण में िेजी ि कर क्षेत्र के िोर्गोां के विए अवधक समृस्द्ध पैद करन है ।
सर्दस्: APEC की 21 सदस्य अथिव्यिथथ एाँ हैं :

Forum Learning Centre: Delhi - 2nd Floor, IAPL House, 19 Pusa Road, Karol Bagh, New Delhi - 110005 | Patna - 2nd floor, AG Palace, E Boring Canal
Road, Patna, Bihar 800001 | Hyderabad - 1st & 2nd Floor, SM Plaza, RTC X Rd, Indira Park Road, Jawahar Nagar, Hyderabad, Telangana 500020
9311740400, 9311740900 | https://academy.forumias.com | admissions@forumias.academy | helpdesk@forumias.academy

[43]
PTS 2024 | Test Code: 121405 - Solutions |

ऑस्ट्रे विया; ब्रुनेई द रुस्सि म; कन ड ; वचिी; पीपुल्स ररपस्िक ऑफ च इन ; ह ां र्गक ां र्ग, चीन; इां डोनेविय ; ज प न; कोररय न
र्गणिन्त्र; मिेविय ; मेस्िको; न्यूज़ीिैंड; प पुआ न्यू वर्गनी; पेरू; वफिीपी ांस; रूसी सांघ; वसांर्ग पुर; चीनी ि इपे; थ ईिैंड; सांयुि
र ज्य अमेररक ; िथ वियिन म।
सवििािय: APEC प्रविय को वसांर्ग पुर स्थथि थथ यी सवचि िय ि र समवथिि वकय ज ि है ।
APEC पॉविसी सपोटि यूवनट ि र प्रक विि एक ररपोटि के अनुस र, 2022 में APEC अथिव्यिथथ ओां क िैविक न मम त्र सकि
घरे िू उत्प द क 62 प्रवििि और िैविक व्य प र क 47 प्रवििि वहस्स थ ।
कथन 1 सही है : 1991 की वसयोि घोषण के दौर न, APEC की सदस्य अथिव्यिथथ ओां ने पूरे प्रि ां ि क्षेत्र में एक उद रीकृि
मुि व्य प र क्षेत्र की थथ पन को अपन मुि िक्ष्य घोवषि वकय ।
कथन 2 सही है : APEC एक सहकारी, बहुपक्षीय आवथिक और व्यापार मंि के रूप में कायि करता है । APEC में, सभी
अथिव्यिथथ ओां को सम न अवधक र प्र प्त है और वनणिय सििसम्मवि से विय ज ि है । सदस्योां के विए कोई ब ध्यक री
प्रविबद्धि एाँ य सांवध द वयत्व नहीां हैं । प्रविबद्धि एाँ स्वेच्छ से की ज िी हैं ।
कथन 3 सही है : ह ि ही में सैन फ् ां वसस्को, सांयुि र ज्य अमेररक में सांपन्न APEC नेि विखर सम्मेिन में, र्गोल्डन र्गेट घोषण
को अपन य र्गय थ । घोषण क उद्दे श्य वपछिे APEC मेजब नोां के प्रय सोां के आध र पर क्षेत्रीय िचीि पन, स्थथरि ,
कनेस्िविटी और नि च र को बढ न है । घोषण में सदस्य दे िोां ि र की र्गई प्रवतबद्ताओं में से एक में 2030 तक िैविक
स्तर पर निीकरणीय ऊजाि क्षमता क तीन गुना करने का प्रयास शावमि है।
कथन 4 ग़ित है : भ रि एविय -प्रि ां ि आवथिक सहयोर्ग (APEC) क सदस्य नहीां है । िििम न में भ रि को APEC में पयििेक्षक
क दज ि प्र प्त है ।
ज्ञानक ष:
• पुिजया विजन 2040: APEC पुत्रजय विजन 2040 को 2020 में नेि ओां ि र अपन य र्गय थ , जो आर्ग मी दो दिकोां के
विए APEC के उद्दे श्योां को रे ख ां वकि करने ि ि एक रणनीविक ढ ां च है । इस दृवष्टकोण क िक्ष्य 2040 िक एक खुि ,
र्गवििीि, िचीि और ि ां विपूणि एविय -प्रि ां ि क्षेत्र बन न है , वजससे सभी वनि वसयोां और भ िी पीवढयोां की समृस्द्ध सुवनविि हो
सके।
• एओटे र आ कायिय जना: 2021 में, APEC की 21 सदस्य अथिव्यिथथ ओां ने एओटे रोआ एक्शन प्ल न िैय र वकय । यह
योजन पुत्रजय विजन 2040 में वनध ि ररि िक्ष्योां को स क र करने के विए एक रोडमैप के रूप में क यि करिी है , जो 1994
बोर्गोर िक्ष्य और 1995 ओस क एक्शन एजेंड में थथ वपि APEC के मूिभूि वसद्ध ां िोां पर आध ररि है ।
• ग ल्डन गेट घ षणा 2023: घोषण में सदस्य दे िोां ि र वनम्नविस्खि प्रविबद्धि एाँ की र्गई हैं ।
o सैन फ्ांवसस्क वसद्ांत- ये वसद्ध ां ि व्य प र और वनिेि नीवि में सम िेविि और स्थथरि को एकीकृि करने पर ध्य न
केंवद्रि करिे हैं । िे र्गोल्डन र्गेट घोषण में की र्गई व्य पक प्रविबद्धि क वहस्स हैं ।
o बहु-विषयक वनिेश िातािरण- र्गोल्डन र्गेट घोषण दस्त िेज़ में नेि ओां ने एक स्विांत्र, खुि , वनष्पक्ष, र्गैर-भेदभ िपूणि,
प रदिी, सम िेिी और पूि ि नुम वनि व्य प र और वनिेि ि ि िरण दे ने के अपने दृढ सांकल्प की पुवष्ट की है ।
Source: https://history.state.gov/milestones/1989-1992/apec
https://www.apec.org/about-us/about-apec

Q.55)
Ans) b
Exp) विकल्प b सही उत्तर है।
वित्त आयोर्ग, एक सांिैध वनक वनक य, की थथ पन केंद्र और र ज्य सरक रोां के बीच कुछ र जस्व सांस धनोां को आिांवटि करने के
विए की र्गई थी।
कथन 1 सही है : भारतीय संविधान का अनुच्छेर्द 280 हर पांि साि में कम से कम एक बार आय ग की स्थापना का
आर्दे श र्दे ता है । यह वनयुस्ि प्रविय को वनवदि ष्ट करि है , वजसके िहि र ष्टरपवि को सांविध न के ि र्गू होने के दो स ि के भीिर
और उसके ब द हर प ां च स ि में य उससे पहिे, जैस आिश्यक समझ ज ए, एक वित्त आयोर्ग क र्गठन करने की

Forum Learning Centre: Delhi - 2nd Floor, IAPL House, 19 Pusa Road, Karol Bagh, New Delhi - 110005 | Patna - 2nd floor, AG Palace, E Boring Canal
Road, Patna, Bihar 800001 | Hyderabad - 1st & 2nd Floor, SM Plaza, RTC X Rd, Indira Park Road, Jawahar Nagar, Hyderabad, Telangana 500020
9311740400, 9311740900 | https://academy.forumias.com | admissions@forumias.academy | helpdesk@forumias.academy

[44]
PTS 2024 | Test Code: 121405 - Solutions |

आिश्यकि होिी है । इसक मििब यह है वक भ रि के र ष्टरपवि को हर प ां च स ि में कम से कम एक ब र वित्त आयोर्ग क


र्गठन करन होर्ग ।
कथन 2 गित है : वित्त आय ग राज्य सरकार ं के बजट के प्रबंधन से संबंवधत वकसी भी कारि िाई की वसफाररश कर
सकता है । उद हरण के विए, केंद्र सरक र ि र ब रहिें वित्त आयोर्ग को केंद्र और र ज्योां दोनोां की वित्तीय स्थथवि की समीक्ष
करने क क म सौांप र्गय थ । आयोर्ग को व्य पक आवथिक स्थथरि , ऋण में कमी और सम न विक स प्र प्त करने के विए केंद्र
और र ज्य सरक रोां के बजटीय सांिुिन को बह ि करने के उद्दे श्य से र जकोषीय पुनर्गिठन के विए एक योजन क प्रस्त ि दे ने
क अवधदे ि वदय र्गय है ।
कथन 3 सही है: नीवि आयोर्ग के पूिि उप ध्यक्ष अरविांद पनर्गवढय को सोिहिें वित्त आयोर्ग के अध्यक्ष के रूप में वनयुि वकय
र्गय थ ।
Source: https://www.constitutionofindia.net/articles/article-280-finance-commission/
https://fincomindia.nic.in/archive/writereaddata/html_en_files/oldcommission_html/fcreport/Report_of_12
th_Finance_Commission/Chapter%2001.pdf (pg no 2)

Q.56)
Ans) c
Exp) विकल्प c सही उत्तर है ।
अनुच्छेर्द 103 में कहा गया है वक यवर्द क ई प्रश्न उठता है वक क्या संसर्द के वकसी भी सर्दन का क ई सर्दस् अनुच्छेर्द
102 (1) में उक्तिक्तित अय ग्यता के अधीन ह गया है , वजसमें भ रि सरक र के िहि ि भ क पद ध रण करन ,
वदि विय पन, म नवसक रूप से अस्वथथि जैसे आध र ि वमि हैं , वनणिय के विए म मि र ष्टरपवि के प स भेज ज एर्ग ।
अनुच्छेद 103(2) में प्र िध न है वक ऐसे प्रश्न पर वनणिय िेने से पहिे र ष्टरपवि को भ रि के चुन ि आयोर्ग की र य िेनी होर्गी और
र ष्टरपवि चुन ि आयोर्ग ि र दी र्गई र य के अनुस र क यि करें र्गे। इसविए, भ रि क चुन ि आयोर्ग सांसद सदस्योां की अयोग्यि
से सांबांवधि म मिोां में महत्वपूणि भूवमक वनभ ि है ।
ज्ञानक ष:
भारत के संविधान के अनुच्छेर्द 102 (1) में उक्तिक्तित प्रािधान संसर्द के वकसी भी सर्दन के सर्दस् के रूप में वकसी
व्यक्ति की अय ग्यता के विवभन्न आधार ं क वनवर्दि ि करते हैं । इसमे शावमि है :
a) भ रि सरक र य वकसी र ज्य सरक र के अधीन ि भ क कोई पद ध रण करन , जब िक वक सांसद ने क नून ि र घोवषि
नहीां वकय हो वक ऐस पद उसके ध रक को अयोग्य नहीां ठहर ि है ।
b) म नवसक रूप से अस्वथथ होन और सक्षम न्य य िय ि र ऐस घोवषि वकय ज न ।
c) घोवषि वदि विय होन ।
d) भ रि क न र्गररक न होन , स्वेच्छ से वकसी विदे िी र ज्य की न र्गररकि प्र प्त करन , य वकसी विदे िी र ज्य के प्रवि वनष्ठ
य प िन की वकसी स्वीकृवि के अधीन होन ।
e) सांसद ि र बन ए र्गए वकसी क नून ि र य उसके िहि अयोग्य ठहर य ज न ।
अनुच्छेद 102(2) में प्र िध न है वक कोई व्यस्ि सांसद के वकसी भी सदन क सदस्य होने के विए अयोग्य होर्ग यवद िह दसिीां
अनुसूची के िहि अयोग्य है ।

Q.57)
Ans) c
Exp) विकल्प c सही उत्तर है ।
ह ि ही में, र्गृह मांत्र िय ने 'मॉडि जेि अवधवनयम, 2023' ज री वकय है वजसक उद्दे श्य जेि प्रबांधन में सुध र करन और
कैवदयोां को क नून क प िन करने ि िे न र्गररकोां में बदिन और सम ज में उनक पुनि ि स सुवनविि करन है ।

Forum Learning Centre: Delhi - 2nd Floor, IAPL House, 19 Pusa Road, Karol Bagh, New Delhi - 110005 | Patna - 2nd floor, AG Palace, E Boring Canal
Road, Patna, Bihar 800001 | Hyderabad - 1st & 2nd Floor, SM Plaza, RTC X Rd, Indira Park Road, Jawahar Nagar, Hyderabad, Telangana 500020
9311740400, 9311740900 | https://academy.forumias.com | admissions@forumias.academy | helpdesk@forumias.academy

[45]
PTS 2024 | Test Code: 121405 - Solutions |

कथन 1 ग़ित है : 'जेि 'ं और 'उनमें वनरुद् व्यक्तिय 'ं का विषय भारतीय संविधान की सातिी ं अनुसूिी में सूिी II
(राज्य सूिी) के तहत प्रर्दान वकया गया है । इसक मििब है वक जेि प्रि सन की प्र थवमक वजम्मेद री र ज्य सरक रोां की है ।
जस्स्ट्स मुल्ल सवमवि (1983) ने जेि को समििी सूची के अांिर्गिि ि ने की वसफ ररि की है ।
कथन 2 सही है: मॉडि क र र्ग र अवधवनयम 2023 में प्र िध न है वक उपयुि सरक र कैवदयोां के विए वजिनी आिश्यकि हो
उिनी खुिी और अधि -खुिी जेिोां (सुध र त्मक सांथथ न) की थथ पन और रखरख ि कर सकिी है । मॉडि क र र्ग र अवधवनयम
2023' वब्रवटि क ि के क नून (1894 क क र र्ग र अवधवनयम) की जर्गह िेर्ग । जेि अवधवनयम 1894 मुि रूप से अपर वधयोां
को वहर सि में रखने और जेिोां में अनुि सन और व्यिथथ ि र्गू करने पर केंवद्रि है । आर्दशि कारागार अवधवनयम 2023
कैवर्दय ं के सुधार और पुनिािस पर केंवद्रत है ।
कथन 3 सही है : चल्ल र मकृष्ण रे ड्डी और अन्य के ऐविह वसक म मिे में सुप्रीम कोटि ने फैसि सुन य वक भ रि में कैदी
जीिन के अवधक र सवहि सभी मौविक अवधक रोां के हकद र हैं , जब िक वक िे क र ि स के क रण अक्षम्य न होां। म मिे में
यह भी कह र्गय वक जीिन क अवधक र म नि वधक रोां में सिोपरर है ।
कथन 4 सही है : न्य यमूविि मुल्ल सवमवि और न्य यमूविि िी. आर. कृष्ण अय्यर सवमवि दोनोां भ रि में जेि सुध रोां से सांबांवधि
हैं ।
• न्य यमूविि मुल्ल सवमवि: इसे जेि सुध र पर अस्खि भ रिीय सवमवि के रूप में भी ज न ज ि है , इसने 1983 में अपनी ररपोटि
प्रस्तुि की। कुछ वसफ ररिें:
o जेि कमििाररय ं के विए अक्तिि भारतीय कैडर और जेि क समिती सूिी के अंतगित िाना; (b) सरकार क जेि ं
पर एक रािरीय नीवत बनानी िावहए; (c) सरकार कारािास के विकल्प ं जैसे सामुर्दावयक सेिा आवर्द का उपय ग करे गी।
• न्य यमूविि िी. आर. कृष्ण अय्यर सवमवि: वििेष रूप से मवहि कैवदयोां से सांबांवधि जेि सुध रोां पर ध्य न केंवद्रि वकय , और
1987 में अपनी ररप टि प्रस्तुत की। कुछ वसफाररशें:
o मवहि अपर वधयोां के विए अकेिे मवहि कमिच ररयोां ि िी अिर्ग सांथथ एाँ ; (b) दोषी प ए ज ने पर भी मवहि ओां की र्गररम
बह ि करने के विए आिश्यक प्र िध न।
Source: https://www.thehindu.com/society/prison-labour-fair-wages-lacking-reform-rehabilitation-india-
jails-tihar-tamil-nadu-kerala-bihar-maharashtra/article67941845.ece
https://forumias.com/blog/issues-related-to-prisons-and-prison-reforms-explained-
pointwise/#:~:text=Without%20enough%20prison%20staff%2C%20overcrowding,and%20lack%20of%20a
dequate%20facilities.
https://pib.gov.in/PressReleseDetailm.aspx?PRID=1923682
https://blog.ipleaders.in/rights-prisoners-major-judgments/#Right_to_Privacy_in_recent_times
https://www.mha.gov.in/MHA1/Par2017/pdfs/par2023-pdfs/LS-08082023/3007.pdf

Q.58)
Ans) c
Exp) विकल्प c सही उत्तर है ।
िन (संरक्षण) संश धन अवधवनयम, 2023, िन (संरक्षण) अवधवनयम, 1980 में संश धन करता है । इसका उद्दे श्य भारत
के िन ं और जैि विविधता का संरक्षण करते हुए सतत विकास क बढािा र्दे ना है ।
कथन 1 सही है : िन (संरक्षण) संश धन अवधवनयम, 2023 िन (संरक्षण) अवधवनयम, 1980 में संश धन करता है
तावक इसे कुछ प्रकार की भूवम पर िागू वकया जा सके। इनमें भ रिीय िन अवधवनयम, 1927 के िहि य 1980 अवधवनयम
ि र्गू होने के ब द सरक री ररकॉडि में जांर्गि के रूप में अवधसूवचि भूवम ि वमि है । यह अवधवनयम 12 वदसांबर, 1996 से पहिे
र्गैर-िन उपयोर्ग में पररिवििि भूवम पर ि र्गू नहीां होर्ग ।
कथन 2 सही है: सांिोधन अवधवनयम िन भूवम में अनुमि र्गविविवधयोां की सूची में और अवधक र्गविविवधय ाँ जोड़ि है , जैसे:
1) िन्यजीि (सांरक्षण) अवधवनयम, 1972 के िहि वचवड़य घर और सफ री

Forum Learning Centre: Delhi - 2nd Floor, IAPL House, 19 Pusa Road, Karol Bagh, New Delhi - 110005 | Patna - 2nd floor, AG Palace, E Boring Canal
Road, Patna, Bihar 800001 | Hyderabad - 1st & 2nd Floor, SM Plaza, RTC X Rd, Indira Park Road, Jawahar Nagar, Hyderabad, Telangana 500020
9311740400, 9311740900 | https://academy.forumias.com | admissions@forumias.academy | helpdesk@forumias.academy

[46]
PTS 2024 | Test Code: 121405 - Solutions |

2) इकोटू ररज्म सुविध एां ,


3) वसल्वीकल्चरि सांच िन और
4) केंद्र सरक र ि र वनवदि ष्ट कोई अन्य उद्दे श्य।
कथन 3 सही है: नए सांिोधन के अनुस र कुछ प्रक र की िन भूवम को अवधवनयम के प्र िध नोां से छूट दी र्गई है , जैसे:
1) आि स य सुविध ओां िक पहां च के विए सरक र ि र सांच विि रे ि ि इनोां य स ििजवनक सड़कोां के वकन रे 0.10 हे िेयर
िक की िन भूवम।
2) रणनीविक र ष्टरीय पररयोजन ओां के विए अां िरर ष्टरीय सीम ओां, वनयांत्रण रे ख य ि स्तविक वनयांत्रण रे ख के 100 वकमी के
भीिर िन भूवम।
3) सुरक्ष से सांबांवधि बुवनय दी ढ ां चे के विए 10 हे िेयर िक िन भूवम।
4) ि मपांथी उग्रि द ि िे क्षेत्रोां में रक्ष , अधिसैवनक विविरोां य स ििजवनक पररयोजन ओां के विए प ांच हे िेयर िक िन भूवम।
Source: https://egazette.gov.in/WriteReadData/2023/247866.pdf
https://prsindia.org/billtrack/the-forest-conservation-amendment-bill-2023
https://forumias.com/blog/forest-conservation-amendment-bill-2023-explained-
pointwise/#:~:text=The%20Forest%20(Conservation)%20Amendment%20Act%2C%202023%2C%20amen
ded,Act%2C%201927%20or%20under%20any%20other%20law%2C

Q.59)
Ans) a
Exp) विकल्प a सही उत्तर है ।
पय ि िरण, िन और जिि यु पररिििन मांत्र िय ने प्लाक्तस्ट्क अपवशि प्रबंधन वनयम, 2016 में संश धन करने के विए
प्लाक्तस्ट्क अपवशि प्रबंधन (संश धन) वनयम, 2024 क अवधसूवित वकया।
विकल्प a सही है : प्लाक्तस्ट्क अपवशि प्रबंधन (संश धन) वनयम 2024 बाय वडग्रेडेबि प्लाक्तस्ट्क क पररभावषत करता है
जो न केिि वमट्टी, िैंडवफि जैसे विविष्ट ि ि िरण में जैविक प्रविय ओां ि र वर्गर िट में सक्षम है , बस्ि ऐसी स मग्री भी है जो
कोई म इिोप्ल स्स्ट्क नही ां छोड़िी है ।
विकल्प b गलत है : सांिोधन वनयम यह वनवदि ष्ट नहीां करिे हैं वक कौन से र स यवनक परीक्षण म इिोप्ल स्स्ट्ि की अनुपस्थथवि
थथ वपि कर सकिे हैं य उन्मूिन के विए म इिोप्ल स्स्ट्ि को वकस हद िक कम वकय ज न च वहए।
विकल्प c गलत है : नए वनयमोां के िहि, वनम ि ि ओां को कांपोस्ट्े बि य ब योवडग्रेडेबि प्ल स्स्ट्क से कैरी बैर्ग और िस्तुओां क
उत्प दन करने की अनुमवि है , और उन्हें अपने उत्प दोां के विपणन य वबिी से पहिे केंद्रीय प्रदू षण वनयांत्रण बोडि (CPCB) से
प्रम ण पत्र प्र प्त करन होर्ग ।
विकल्प d ग़ित है : सांिोधन में वकसी भी उद्योर्ग में म इिोप्ल स्स्ट्क पर प्रविबांध क उल्लेख नहीां है । नए वनयमोां क मुि वबांदु
जैि अपघटनीय प्ल स्स्ट्क से म इिोप्ल स्स्ट्क को खत्म करने पर है । भ रि में, प्ल स्स्ट्क अपविष्ट प्रबांधन वनयम, 2016 के
अनुस र, 2022 में एकि-उपयोर्ग प्ल स्स्ट्क पर प्रविबांध िर्ग वदय र्गय है ।
Source: https://www.thehindu.com/sci-tech/energy-and-environment/no-microplastics-new-rules-queer-
the-pitch-for-biodegradable-plastics/article67977130.ece
http://www.indiaenvironmentportal.org.in/content/476831/plastic-waste-management-amendment-rules-
2024/

Q.60)
Ans) b
Exp) विकल्प b सही उत्तर है।

Forum Learning Centre: Delhi - 2nd Floor, IAPL House, 19 Pusa Road, Karol Bagh, New Delhi - 110005 | Patna - 2nd floor, AG Palace, E Boring Canal
Road, Patna, Bihar 800001 | Hyderabad - 1st & 2nd Floor, SM Plaza, RTC X Rd, Indira Park Road, Jawahar Nagar, Hyderabad, Telangana 500020
9311740400, 9311740900 | https://academy.forumias.com | admissions@forumias.academy | helpdesk@forumias.academy

[47]
PTS 2024 | Test Code: 121405 - Solutions |

जैसा नक केंद्रीय बजट 2023 में घोषणा की गई थी, सेबी ने हाल ही में 'कॉपोरे ट ऋण बाजार नवकास कोष' (सीडीएमडीएफ) की
स्थापना की शुरुआत की।
कथन 1 सही है : सीडीएमडीएफ एक बैकस्टॉप सुनविा के रूप में कायण करता है और 5 साल तक की शेष पररपक्वता वाली
ननवेश-ग्रेड सूचीबद्ध कॉपोरे ट ऋण प्रनतभूनतयों को खरीदकर तनावग्रस्त बाजार ल्कस्थनतयों के दौरान सहायता प्रदान करता है ।
बैकस्टॉप शेयरों के अनसब्सिाइब्ड नहस्से के नलए सुरक्षा पेशकश में अंनतम उपाय सहायता या सुरक्षा प्रदान करने का एक
कायण है ।
कथन 2 सही है : सीडीएमडीएफ को सेबी द्वारा नवननयनमत नकया जाता है और एक अलग उप-श्रेणी के तहत वैकल्कल्पक ननवेश
कोष (एआईएफ) के रूप में कायण करता है ।
कथन 3 गलत है : कॉपोरे ट ऋण बाजार नवकास कोष (सीडीएमडीएफ) को म्यूचुअल फंड (एमएफ), पररसंपनत्त प्रबंिन कंपननयों
(एएमसी) और केंद्र सरकार द्वारा नवत्त पोनषत नकया जाएगा। एमएफ को अपने फंड की प्रबंिनािीन पररसंपनत्तयों (एयूएम) का
0.25% सीडीएमडीएफ में योगदान करना आवश्यक है , जबनक एएमसी 0.02% योगदान दें गे। सरकार 30,000 करोड रुपये का
योगदान दे गी और शेष 3,000 करोड रुपये एएमसी से आएं गे।
Source: https://economictimes.indiatimes.com/markets/bonds/corporate-debt-market-development-
fund-how-will-it-work/articleshow/102745529.cms?from=mdr
https://pib.gov.in/PressReleseDetailm.aspx?PRID=1943686

Q.61)
Ans) a
Exp) विकल्प a सही उत्तर है ।
वमिेट्स (मोट अन ज), ग्र स फैवमिी से सांबांवधि छोटे अन ज, कठोर ि ि िरण में अपनी िन्यिीिि के विए प्रवसद्ध हैं , वजससे
िे कई एविय ई और अफ्ीकी दे िोां में एक प्रमुख फसि बन ज िे हैं । िैविक वमिेट्स कृवष क 26.6% और एवशया के
वमिेट्स फसि क्षेि का 83% के साथ, भारत िैविक वमिेट्स उत्पार्दन में पहिे स्थान पर है ।
विकल्प a सही है : बाजरा, रागी, सांिा और क र्द ये सभी वमिेट्स के प्रकार हैं । बाजरा और रागी प्रमुि वमिेट्स हैं ,
जबवक सानिा और क र्द माइनर वमिेट्स हैं ।
• बाजरा (पिि वमिेट):
• ब जर एक प्रमुख अन ज की फसि है और मुि रूप से भ रि और अफ्ीक में उर्ग य ज ने ि ि प्रमुख वमिेट है ।
• यह कम िष ि ि िे िुष्क और अधि -िुष्क क्षेत्रोां में पनपि है ।
• भ रि में इसकी खेिी मुि रूप से र जथथ न, मह र ष्टर, र्गुजर ि र ज्योां में की ज िी है ।
• द ने छोटे , र्गोि और मोिी जैसे होिे हैं , इसविए इसक न म "पिि वमिेट" है ।
• ब जर प्रोटीन और आह र फ इबर से भरपूर है और इसमें सभी अन जोां की िुिन में वनय वसन की म त्र सबसे अवधक है ।
• च िि और र्गेहां के ब द यह भ रि में सबसे अवधक उर्ग य ज ने ि ि अन ज है ।
• रागी (वफंगर वमिेट):
• र र्गी एक अन्य महत्वपूणि प्रमुख वमिेट्स है , वजसकी खेिी प्रमुख रूप से कन ि टक, आां ध्र प्रदे ि, िवमिन डु और र जथथ न
र ज्योां में की ज िी है । भ रि में िर्गभर्ग 60% र र्गी क उत्प दन अकेिे कन ि टक ि र वकय ज ि है ।
• इसे कई प्रक र की पररस्थथवियोां में उर्ग य ज सकि है िेवकन यह अच्छी जि वनक सी ि िी, दोमट वमट्टी में सबसे अच्छ
पनपि है ।
• यह अपनी उच्च एां टीऑिीडें ट र्गविविवध और कैस्ल्ियम, आयरन और प्रोटीन सवहि पोषण स मग्री के विए ज न ज ि है ।
• र र्गी ग्लूटेन-मुि है और इसक स्व द अखरोट जैस है । इसक उपयोर्ग र र्गी बॉल्स (मुड्डे), डोस और दविय बन ने के विए
वकय ज ि है ।
• सानिा (बानियाडि वमिेट):

Forum Learning Centre: Delhi - 2nd Floor, IAPL House, 19 Pusa Road, Karol Bagh, New Delhi - 110005 | Patna - 2nd floor, AG Palace, E Boring Canal
Road, Patna, Bihar 800001 | Hyderabad - 1st & 2nd Floor, SM Plaza, RTC X Rd, Indira Park Road, Jawahar Nagar, Hyderabad, Telangana 500020
9311740400, 9311740900 | https://academy.forumias.com | admissions@forumias.academy | helpdesk@forumias.academy

[48]
PTS 2024 | Test Code: 121405 - Solutions |

• स नि को ब निय डि वमिेट के न म से भी ज न ज ि है , यह एक म इनर वमिेट है वजसकी खेिी मुि रूप से भ रि, चीन
और ज प न में की ज िी है । भ रि में इसकी खेिी मुि रूप से उत्तर खांड, िवमिन डु , आां ध्र प्रदे ि और कन ि टक र ज्योां में की
ज िी है ।
• िे अवधक ऊांच ई पर अच्छी िरह विकवसि होिे हैं , िेवकन पररपक्वि के विए िीन से च र महीने की आिश्यकि होिी है ।
• यह कच्चे फ इबर और आयरन क अच्छ स्रोि है
• क र्द (क र्द वमिेट):
• कोदो वमिेट एक म इनर वमिेट है , वजसकी खेिी मुि रूप से ओवडि , मध्य प्रदे ि, कन िटक और िवमिन डु र ज्योां में की
ज िी है ।
• यह सूख -सवहष्णु है और पथरीिी वमट्टी पर उर्गि है जो अन्य फसिोां के विए उपयुि नहीां है
• कोदो वमिेट में सभी वमिेट्स के बीच सबसे अवधक आह र फ इबर होि है , और यह वनय वसन, फोविक एवसड से भी समृद्ध
होि है ।
विकल्प b गित है : ज्व र, ब जर और क कुम वमिेट्स हैं , जबवक मक्क कॉनि है , वमिेट नही ां है ।
• मक्का (मकई): मक्क (मकई) ब जर नहीां है । मक्क बड़े , पीिे द नोां ि ि एक अन ज है । इसक उपयोर्ग म नि उपभोर्ग,
पिु च र और औद्योवर्गक उद्दे श्योां के विए वकय ज ि है ।
विकल्प c गित है : र र्गी, प रसो और कुटकी वमिेट हैं , जबवक न इजरसीड एक वििहन है , वमिेट नहीां।
• नाइजरसीड (नाइजर): न इजरसीड एक वििहनी फसि है , वमिेट नहीां। इसे मुि रूप से इसके िेि-समृद्ध बीजोां के विए
उर्ग य ज ि है , वजनक उपयोर्ग ख न पक ने और औद्योवर्गक अनुप्रयोर्गोां में वकय ज ि है ।
विकल्प d गित है : ज्व र और क कुम वमिेट्स हैं , जबवक सरसोां और िोररय वििहन हैं ।
• सरस :ं सरसोां वमिेट नही ां है ; यह एक वििहनी फसि है । सरसोां के बीज क उपयोर्ग सरसोां क िेि बन ने के विए वकय
ज ि है और इसक उपयोर्ग मस िे के रूप में भी वकय ज ि है ।
• त ररया (रे पसीड): िोररय एक अन्य वििहनी फसि है । इसके बीज िेि और प्रोटीन से भरपूर होिे हैं । िोररय िेि क
उपयोर्ग ख न पक ने और औद्योवर्गक उद्दे श्योां के विए वकय ज ि है ।
ज्ञानक ष: ब जर के अन्य प्रक र:
• प्र स (प्र स वमिेट):
• प्रोसो एक छोटे द ने ि ि वमिेट है जो मुि रूप से भ रि के उत्तरी र ज्योां में उर्ग य ज ि है ।
• यह अत्यवधक सूख प्रविरोधी और कम मौसम ि िी फसि है जो कम िष ि ि िे क्षेत्रोां में उर्गिी है ।
• इसमें उच्च कैस्ल्ियम के स थ प्रोटीन की म त्र सबसे अवधक होिी है । यह कोिेस्ट्रॉि के स्तर को कम करके हृदय रोर्गोां के
खिरे को कम करि है ।
• कुटकी (विवटि वमिेट):
• यह एक म इनर वमिेट है , जो पूरे भ रि में, मुि रूप से मध्य प्रदे ि, उड़ीस और झ रखांड में उर्ग य ज ि है ।
• यह अवधकिर अन्य वमिेट, द िोां और वििहनोां के स थ वमि य ज ि है ।
• यह जल्दी पररपक्व हो ज ि है और सूखे और जि जम ि दोनोां को सहन करि है ।
• यह एां टीऑिीडें ट, आह रीय फ इबर और िौह ित्व से भरपूर है ।
• ज्वार (ज्वार):
• ज्व र एक बहमुखी प्रमुख वमिेट फसि है , वजसकी खेिी मुि रूप से मह र ष्टर (सबसे बड़ उत्प दक), कन ि टक और
िवमिन डु में की ज िी है ।
• दु वनय भर में पैद होने ि िे अन जोां में इसक थथ न प ां चि ां और भ रि में चौथ है ।
• यह फोविक एवसड, प्रोि वमन और र इबोफ्लेविन से भरपूर है । इसमें एां टी-इां फ्लेमेटरी और एां टी-कैंसर र्गुण होिे हैं ।
• काकुम (फॉक्सटे ि वमिेट):
• क कुम एक सूख प्रविरोधी वमिेट है जो मुि रूप से मेघ िय, कन ि टक, र जथथ न में उर्ग य ज ि है ।
• यह सबसे पुर ने कृवषि वमिेट्स में से एक है ।

Forum Learning Centre: Delhi - 2nd Floor, IAPL House, 19 Pusa Road, Karol Bagh, New Delhi - 110005 | Patna - 2nd floor, AG Palace, E Boring Canal
Road, Patna, Bihar 800001 | Hyderabad - 1st & 2nd Floor, SM Plaza, RTC X Rd, Indira Park Road, Jawahar Nagar, Hyderabad, Telangana 500020
9311740400, 9311740900 | https://academy.forumias.com | admissions@forumias.academy | helpdesk@forumias.academy

[49]
PTS 2024 | Test Code: 121405 - Solutions |

• यह िौह और ि ां बे से भरपूर है । यह प्रविरक्ष में सुध र करि है और हृदय स्व स्थ्य के विए अच्छ है ।
Source: https://eatrightindia.gov.in/millets2023/assets/documents/Millets%20-
%20Nutritious%20Cereal%20of%20India.pdf

Q.62)
Ans) b
Exp) विकल्प b सही उत्तर है।
प्रीपेड पेमेंट इां स्ट्ुमेंट्स (PPI) भुर्गि न विवधय ां हैं वजनक उपयोर्ग स म न और सेि ओां को खरीदने और िॉिेट में सांग्रहीि मूल्य
क उपयोर्ग करके पैसे भेजने/प्र प्त करने के विए वकय ज सकि है ।
कथन 1 गित है : PPI भारतीय ररजिि बैंक (RBI) से अनुम र्दन प्राप्त करने के बार्द बैंक ं और गैर-बैंक ं र्द न ं द्वारा जारी
वकया जा सकता है । बैंकोां को RBI की मांजूरी की आिश्यकि है , जबवक र्गैर-बैंक ज रीकि ि कांपनी अवधवनयम, 1956/2013
के िहि पांजीकृि और RBI ि र अवधकृि भ रिीय कांपवनय ां होनी च वहए।
कथन 2 सही है: PPI में, उपय गकतािओ ं क अपनी िांवछत रावश िॉिेट में पहिे से ि ड करनी ह गी। िे इसे नकद, बैंक
ख िे से डे वबट, िेवडट/डे वबट क डि , यूपीआई, य भ रि में अन्य अनुमोवदि भुर्गि न विवधयोां क उपयोर्ग करके कर सकिे हैं ।
PPI क उपयोर्ग केिि भ रिीय रुपये में वकय ज सकि है ।
कथन 3 सही है : हािााँवक म बाइि िॉिेट व्यापक हैं , PPI भौवतक स्माटि काडि , सुरवक्षत ट कन, िाउिर, या प्रीपेड फंड
तक पहुं ि की अनुमवत र्दे ने िािी वकसी अन्य विवध के रूप में भी आ सकते हैं ।
Source: https://www.rbi.org.in/commonperson/english/scripts/FAQs.aspx?Id=2812
https://www.npci.org.in/what-we-do/upi/faqs#:~:text=merchants%20that%20accepted-,Paytm,-
or%20Mobikwik%20QR

Q.63)
Ans) b
Exp) विकल्प b सही उत्तर है।
भ रिीय ररज़िि बैंक क स ििजवनक ऋण क य ििय (Public Debt Office; PDO) सरक री प्रविभूवियोां की रवजस्ट्री/वडपॉवजटरी
के रूप में क यि करि है और पररपक्वि पर मुद्दे, ब्य ज भुर्गि न और मूिधन के पुनभुिर्गि न से सांबांवधि है । अवधक ां ि वदन ां वकि
प्रविभूविय ाँ वनविि कूपन प्रविभूविय ाँ होिी हैं ।
कथन 1 सही है : वर्दनांवकत G-Sec ऐसी प्रवतभूवतयााँ हैं वजनमें एक वनवित या फ्ल वटं ग कूपन (ब्याज र्दर) ह ता है
वजसका भुगतान अधि-िावषिक आधार पर अंवकत मूल्य पर वकया जाता है ।
कथन 2 सही है : वदन ां वकि सरक री प्रविभूविय ाँ (G-Sec) सरक र ि र वनविि य फ्लोवटां र्ग ब्य ज दर (य कूपन) के स थ ज री
वकए र्गए दीघिक विक वित्तीय उपकरण हैं , वजनक भुर्गि न सुरक्ष के अांवकि मूल्य के आध र पर िषि में दो ब र वकय ज ि है ।
उद हरण के विए, यवद सरक र 1000 रुपये के अांवकि मूल्य और 10% की ि वषिक कूपन दर के स थ वदन ां वकि G-Sec ज री
करिी है , िो इसक मििब है वक सुरक्ष ध रक को ब्य ज आय के रूप में हर छह महीने में 100 रुपये क भुर्गि न वकय
ज एर्ग । वफक्स्ड कूपन G-Sec अपने जीिनक ि में एक स्थथर ब्य ज दर बन ए रखिे हैं , जबवक फ्लोवटां र्ग कूपन G-Sec ब ज र
स्थथवियोां के आध र पर समय-समय पर अपनी दरोां को सम योवजि कर सकिे हैं । आम िौर पर, वदन ां वकि प्रविभूवियोां की
अिवध 5 िषि से 40 िषि िक होिी है ।
कथन 3 ग़ित है : ि वणस्ज्यक बैंकोां के प स बक य वदन ां वकि प्रविभूवियोां क सबसे बड़ प्रवििि है , 2023-2024 में उनक
स्व वमत्व वतणमान में िर्गभर्ग 36% है । ररज़िि बैंक के पास 2023-2024 के र्दौरान वतणमान में 15-17% है ।
Source:
https://www.rbi.org.in/commonperson/english/scripts/FAQs.aspx?Id=711#1:~:text=fixed%20or%20floatin
g-,coupon,-(interest%20rate)%20which

Forum Learning Centre: Delhi - 2nd Floor, IAPL House, 19 Pusa Road, Karol Bagh, New Delhi - 110005 | Patna - 2nd floor, AG Palace, E Boring Canal
Road, Patna, Bihar 800001 | Hyderabad - 1st & 2nd Floor, SM Plaza, RTC X Rd, Indira Park Road, Jawahar Nagar, Hyderabad, Telangana 500020
9311740400, 9311740900 | https://academy.forumias.com | admissions@forumias.academy | helpdesk@forumias.academy

[50]
PTS 2024 | Test Code: 121405 - Solutions |

https://dea.gov.in/sites/default/files/Quarterly%20Report%20on%20Public%20Debt%20Management%20f
or%20the%20quarter%20October%20-%20December%202023.pdf (page 18)

Q.64)
Ans) b
Exp) विकल्प b सही उत्तर है।
ह ि ही में, केंद्रीय स म वजक न्य य और अवधक ररि मांत्र िय ने 2022 में SMILE (आजीविक और उद्यम के विये सीम ां ि
व्यस्ियोां हे िु समथिन) योजन िुरू की और 2021-22 से 2025-26 िक इस योजन के विए 365 करोड़ रुपये आिांवटि वकए।
विकल्प b सही है : टर ां सजेंडर समुद य और भीख म ां र्गने के क यि में िर्गे िोर्गोां को व्य पक कल्य ण और पुनि ि स उप य प्रद न
करने के विए स्म इि योजन । SMILE योजन की वििेषि एां वनम्नविस्खि हैं :
● यह कक्ष IX से िेकर स्न िकोत्तर िक टर ां सजेंडर छ त्रोां को उनकी विक्ष में सह यि के विए छ त्रिृवत्त प्रद न करि है ।
● इसमें प्रध नमांत्री दक्षि और कुििि सांपन्न वहिग्र ही (PM-DAKSH) योजन के िहि कौिि विक स और आजीविक के
प्र िध न ि वमि हैं ।
● यह प्रधान मंिी जन आर ग्य य जना (PM-JAY) के सहयोर्ग से एक व्य पक स्व स्थ्य दे खभ ि पैकेज प्रद न करि है ,
वजसमें चयवनि अस्पि िोां के म ध्यम से विांर्ग-पुवष्ट सजिरी ि वमि है ।
● यह 'गररमा गृह' के म ध्यम से आि स सुविध एां प्रद न करि है , टर ां सजेंडर व्यस्ियोां और भीख म ां र्गने में िर्गे िोर्गोां को
भोजन, कपड़े , मनोरां जक र्गविविवधय ां , कौिि विक स के अिसर और वचवकत्स सह यि जैसी आिश्यक चीजें प्रद न करि है ।
● अपर ध के म मिोां की वनर्गर नी और त्वररि पांजीकरण, ज ां च और अवभयोजन सुवनविि करने के विए प्रत्येक र ज्य में एक
टर ां सजेंडर सांरक्षण सेि होर्ग ।
● आिश्यकि पड़ने पर टर ां सजेंडर व्यस्ियोां और भीख म ां र्गने में िर्गे िोर्गोां को आिश्यक ज नक री और सम ध न प्रद न करने
के विए एक र ष्टरीय पोटि ि और हे ल्पि इन उपिब्ध होर्गी।
Source: https://vikaspedia.in/schemesall/schemes-for-unemployed-and-poor/smile-scheme-for-support-
for-marginalised-individuals-for-livelihood-and-
enterprise#:~:text=Ministry%20of%20Social%20Justice%20%26%20Empowerment,%2D22%20to%202025
%2D26

Q.65)
Ans) d
Exp) विकल्प d सही उत्तर है।
कथन 1 सही है: भिे ही ऋण माफी क बजटीय प्रािधान ं के भीतर समाय वजत वकया जाता है , यह साििजवनक व्यय में
अन्य मर्द ं में की जाने िािी कटौती क प्रेररत करे गा। अनुभि से पि चि है वक सबसे कमजोर श्रेणी पूांजीर्गि व्यय है ।
बदिे में, इससे व्यय की र्गुणित्त में वर्गर िट आएर्गी और अन्य ब िोां के स थ-स थ पररसांपवत्त वनम ि ण वनिेि के रूप में
उत्प दकि पर प्रविकूि प्रभ ि पड़े र्ग , वजसमें कृवष क्षेत्र भी ि वमि है - उद हरण के विए, वसांच ई क यि, कोल्ड स्ट्ोरे ज चेन
इत्य वद - को छोड़ वदय र्गय है
कथन 2 सही है : कृवष ऋण माफी का आम तौर पर मतिब यह है वक कृवष ऋण क बैंक ं की बैिेंस शीट के पररसंपवत्त
पक्ष से सरकार की पुस्तक ं के र्दे नर्दारी पक्ष में स्थानांतररत कर वर्दया जाएगा। अल्प िवध में बैंकोां को ि भ हो सकि है
क्ोांवक उनकी ऋण पुस्स्तक हिी हो ज एर्गी और उन्हें कुछ र्गैर-वनष्प वदि पररसांपवत्तयोां से छु टक र वमि ज एर्ग । इससे
अथिव्यिथथ में बैंकोां के म ध्यम से समग्र ऋण उपिब्धि बढ ज िी है । िेवकन ऐसी छूट और भविष्य में उनकी प्रत्य ि िांबे
समय में िेवडट सांस्कृवि को नुकस न पहां च एर्गी। यह आियि की ब ि नहीां है वक 2008 में कृवष ऋण म फी के ब द, बैंकोां के
कृवष खर ब ऋण य NPA में वर्गर िट बमुस्िि एक स ि िक चिी और एक ब र वफर िेजी से बढने िर्गी।

Forum Learning Centre: Delhi - 2nd Floor, IAPL House, 19 Pusa Road, Karol Bagh, New Delhi - 110005 | Patna - 2nd floor, AG Palace, E Boring Canal
Road, Patna, Bihar 800001 | Hyderabad - 1st & 2nd Floor, SM Plaza, RTC X Rd, Indira Park Road, Jawahar Nagar, Hyderabad, Telangana 500020
9311740400, 9311740900 | https://academy.forumias.com | admissions@forumias.academy | helpdesk@forumias.academy

[51]
PTS 2024 | Test Code: 121405 - Solutions |

िेवकन चीजोां को पररप्रेक्ष्य में रखने के विए, आज NPA की कुि टोकरी में कृवष ऋण की वहस्सेद री कम है । ि स्ति में, अवधक
एनपीए ि िे बैंकोां की कुि NPA में कृवष ऋण की वहस्सेद री कम होिी है । इसक मििब यह है वक िन िग्रस्त बैंकोां के विए
अथथ यी र हि भी क फी म मूिी होर्गी।
कथन 3 सही है : कृवष ऋण माफी बजटीय राजस्व व्यय से अवधक के रूप में साििजवनक वित्त की क्तस्थवत क प्रभावित
करती है । बदिे में, इसे अविररि ब ज र उध रोां ि र वित्तपोवषि वकय ज न च वहए, वजससे अांििः स ििजवनक ऋण में िृस्द्ध
होर्गी और स ििजवनक ऋण को सकि घरे िू उत्प द की 60% की सीम से नीचे रखने की सांभ िन कम हो ज एर्गी।
कथन 4 सही है : संपावििक क्षवत (Collateral Damage) यह है वक उधार िेने की िागत बढने के कारण वनजी
उधारकताि वनिेश य ग्य संसाधन ं के सीवमत पूि से बाहर ह जाते हैं ।
उद हरण के विए, कृवष ऋण म फी = सरक र की उध री में िृस्द्ध = वनजी क्षेत्र में उध रकि ि ओां को ऋण वमिने में कवठन ई
(Crowding Out)।
Source: https://rbi.org.in/scripts/BS_ViewBulletin.aspx?Id=17091
https://www.nabard.org/auth/writereaddata/tender/2304223730farm-loan-waivers-in-india-assessing-
impact-and-looking-ahead_compressed.pdf
https://www.livemint.com/Politics/HcS9M9n9rus7h5JiLQC2ZM/How-will-farm-loan-waivers-impact-the-
Indian-economy.html
https://www.financialexpress.com/policy/economy-final-burden-of-farm-loan-waiver-on-farmers-nabard-
deputy-md-3416092/
https://blogs.worldbank.org/en/allaboutfinance/economic-effects-india-s-farm-loan-bailout-business-
usual

Q.66)
Ans) c
Exp) विकल्प c सही उत्तर है ।
जब क ई अथिव्यिस्था अपने संभावित सकि घरे िू उत्पार्द (GDP) पर काम कर रही है , िो इसक मििब है वक
अथिव्यिथथ मुद्र स्फीवि के दब ि य प्र कृविक दर से नीचे बेरोजर्ग री पैद वकए वबन अपने अवधकिम वटक ऊ स्तर पर
िस्तुओां और सेि ओां क उत्प दन कर रही है । इस पररदृश्य में, अथिव्यिथथ को सांिुिन में म न ज ि है ।
अब, यवद ऐसी अथिव्यिथथ में धन की आपूविि बढ ज िी है , िो इससे कुि म ांर्ग में विस्त र होर्ग । अथिव्यिथथ में अवधक पैस
प्रस ररि होने से, उपभोि ओां और व्यिस योां की िय िस्ि में िृस्द्ध होर्गी, वजससे उपभोर्ग और वनिेि व्यय क स्तर ऊांच
होर्ग ।
ह ि ाँ वक, िूंवक अथिव्यिस्था पहिे से ही अपनी संभावित GDP पर काम कर रही है , इसविए कंपवनयां बढी हुई मांग क
पूरा करने के विए अपने उत्पार्दन का और विस्तार करने में सक्षम नही ं ह सकती हैं । यह स्थथवि अथिव्यिथथ में अविररि
म ां र्ग को जन्म दे िी है , वजसे म ां र्ग पक्ष मुद्र स्फीवि के रूप में भी ज न ज ि है । उपभोि सीवमि उपिब्ध िस्तुओां और सेि ओां
के विए प्रविस्पध ि करिे हैं , वजससे कीमिें बढिी हैं , वजससे मुद्र स्फीवि क दब ि पैद होि है ।
ज्ञानक ष:
• आपूवति-पक्ष मुद्रास्फीवत: यह िब होिी है जब म ि की म ां र्ग के स पेक्ष उत्प दन की ि र्गि बढ ज िी है । इस प्रक र की
मुद्र स्फीवि आपूविि-पक्ष क रकोां जैसे उच्च मजदू री और िेि की कीमिोां के क रण होिी है ।
• स्ट्ै गफ्लेशन: यह एक िब्द है वजसक उपयोर्ग उच्च मुद्र स्फीवि, कम आवथिक विक स और उच्च बेरोजर्ग री ि िी
अथिव्यिथथ क िणिन करने के विए वकय ज ि है । यह "स्थथर (स्ट्ै ग्नेंट)" और "मुद्र स्फीवि" िब्दोां क सांयोजन है ।
• हाइपरइन्फ्फ्लेशन: इसक उपयोर्ग उन स्थथवियोां क िणिन करने के विए वकय ज ि है जह ां सभी िस्तुओां और सेि ओां की
कीमिें एक वनध ि ररि समय में अवनयांवत्रि रूप से बढिी हैं । दू सरे िब्दोां में, अवि मुद्र स्फीवि अत्यांि िीव्र मुद्र स्फीवि है । आम
िौर पर, मुद्र स्फीवि को ह इपरइन्फ्फ्लेिन कह ज ि है जब मुद्र स्फीवि की दर प्रवि म ह 50% से अवधक बढ ज िी है ।

Forum Learning Centre: Delhi - 2nd Floor, IAPL House, 19 Pusa Road, Karol Bagh, New Delhi - 110005 | Patna - 2nd floor, AG Palace, E Boring Canal
Road, Patna, Bihar 800001 | Hyderabad - 1st & 2nd Floor, SM Plaza, RTC X Rd, Indira Park Road, Jawahar Nagar, Hyderabad, Telangana 500020
9311740400, 9311740900 | https://academy.forumias.com | admissions@forumias.academy | helpdesk@forumias.academy

[52]
PTS 2024 | Test Code: 121405 - Solutions |

Source: Indian Economy by Shankar Ganesh and Indian Economy by Vivek Singh
https://www.imf.org/en/Publications/fandd/issues/Series/Back-to-Basics/Monetary-Policy

Q.67)
Ans) c
Exp) विकल्प c सही उत्तर है ।
यवर्द अथिव्यिस्था में सभी व्यक्ति अपनी बित य ग्य आय का अनुपात बढाते हैं (अथाित, यवर्द अथिव्यिस्था की सीमांत
बित प्रिृवत्त बढती है ), त अथिव्यिस्था में बित का कुि मूल्य आिश्यक रूप बढे यह ज़रूरी नही ं है - यह य िो घट
सकि है य स्थथर रह सकि है । इस घटन को "वमिव्यवयि क विरोध भ स (Paradox of Thrift)" कह ज ि है , जो बि ि है
वक बढी हई वमिव्ययि के पररण मस्वरूप य िो बचि कम हो सकिी है य अपररिवििि हो सकिी है । बचि में स मूवहक िृस्द्ध
से अथिव्यिथथ के भीिर व्यय कम हो सकि है , वजससे उत्प दन और आय कम हो सकिी है , जो बचि में िुरुआिी
उत्प्ल िकि को सांिुविि कर सकिी है । इसविए अल्प िवध में, बचि क कुि मूल्य आिश्यक रूप से नही ां बढे र्ग ।
Source: https://ncert.nic.in/textbook/pdf/leec104.pdf

Q.68)
Ans) a
Exp) विकल्प a सही उत्तर है ।
भ रि में सांरक्षण ररजिि न वमि सांरवक्षि क्षेत्र हैं जो थथ वपि र ष्टरीय उद्य नोां, िन्यजीि अभय रण्ोां और आरवक्षि िनोां के बीच
महत्वपूणि सांबांधक के रूप में क यि करिे हैं । िे िन्यजीि सांरक्षण में महत्वपूणि भूवमक वनभ िे हैं ।
कथन 1 गित है : भ रि में सांरक्षण ररजिि घोवषि करने के प्र िध न िन्यजीि (सांरक्षण) सांिोधन अवधवनयम 2002 के िहि आिे
हैं , न वक पय ि िरण सांरक्षण अवधवनयम 1986 के िहि।
पयाििरण संरक्षण अवधवनयम 1986 के तहत पाररक्तस्थवतक तंि के वहस् ं क पाररक्तस्थवतक रूप से संिेर्दनशीि क्षेि
(ESA) घ वषत करने का प्रािधान बनाया गया है ।
कथन 2 ग़ित है : र ज्य सरक रोां के प स सांरवक्षि क्षेत्रोां (र ष्टरीय उद्य नोां और अभय रण्ोां से सटे क्षेत्र और िे क्षेत्र जो एक सांरवक्षि
क्षेत्र को दू सरे से जोड़िे हैं ) के ब हर वनजी भूवम के अविररि वकसी भी सरक री स्व वमत्व ि िी भूवम को आसप स के समुद योां
के स थ पर मिि के ब द सांरक्षण ररजिि के रूप में न वमि करने क अवधक र है ।
कथन 3 सही है : र ज्य सरक र मुि िन्यजीि ि डि न को सांरक्षण ररजिि के सांरक्षण, प्रबांधन और रखरख ि के विए सि ह दे ने
के विए एक सांरक्षण ररजिि प्रबांधन सवमवि क र्गठन करिी है ।
सवमवि में िन य िन्यजीि विभ र्ग क एक प्रविवनवध ि वमि होर्ग , जो सवमवि क सदस्य-सवचि होर्ग , प्रत्येक ग्र म पांच यि क
एक प्रविवनवध वजसके अवधक र क्षेत्र में ररजिि स्थथि है , स थ ही िन्यजीि सांरक्षण के क्षेत्र में क म करने ि िे र्गैर-सरक री सांर्गठनोां
के िीन प्रविवनवध और कृवष और पिुप िन विभ र्ग से एक-एक प्रविवनवध। इस प्रक र, इन सांरक्षण भांड रोां के प्रबांधन के विए
थथ नीय भ र्गीद री के स थ सांरक्षण ररजिि प्रबांधन सवमवि क र्गठन वकय ज ि है ।
Source: https://indiankanoon.org/doc/1781078/
http://www.wildbengal.com/conservation-reserve.php
https://www.downtoearth.org.in/news/forests/community-reserves-are-they-forest-department-s-
backdoor-entry-into-north-east-india-85242
https://www.thehindu.com/news/national/tamil-nadu/india-gets-its-first-dugong-conservation-reserve-
in-tamil-nadu/article65918239.ece

Forum Learning Centre: Delhi - 2nd Floor, IAPL House, 19 Pusa Road, Karol Bagh, New Delhi - 110005 | Patna - 2nd floor, AG Palace, E Boring Canal
Road, Patna, Bihar 800001 | Hyderabad - 1st & 2nd Floor, SM Plaza, RTC X Rd, Indira Park Road, Jawahar Nagar, Hyderabad, Telangana 500020
9311740400, 9311740900 | https://academy.forumias.com | admissions@forumias.academy | helpdesk@forumias.academy

[53]
PTS 2024 | Test Code: 121405 - Solutions |

Q.69)
Ans) a
Exp) विकल्प a सही उत्तर है ।
हाि ही में, तवमिनाडु के राज्यपाि ने राज्य सरकार के छह नामांवकत व्यक्तिय ं में से पांि क तवमिनाडु ि क सेिा
आय ग के सर्दस् के रूप में वनयुि वकया है। ह ि ाँ वक, उन्होांने अपने वपछिे क यिथथि से कवथि अनुि सन त्मक मुद्दोां क
हि ि दे िे हए एक अवधक री की वनयुस्ि पर आपवत्त जि ई।
कथन 1 सही है : भ रि के सांविध न के अनुच्छेद 316 के िहि, र ज्य िोक सेि आयोर्ग (SPSC) में र ज्य के र ज्यप ि ि र
वनयुि एक अध्यक्ष और अन्य सदस्य होिे हैं । सांविध न आयोर्ग की िस्ि वनवदि ष्ट नहीां करि है िेवकन इस म मिे को र ज्यप ि
के वििेक पर छोड़ वदय र्गय है । अनुच्छेर्द 318 के तहत संविधान राज्यपाि क आय ग के अध्यक्ष और सर्दस् ं की सेिा
की शतें वनधािररत करने के विए अवधकृत करता है ।
कथन 2 गित है : SPSC के अध्यक्ष और सर्दस् ं की वनयुक्ति राज्यपाि द्वारा की जाती है । ह ि ाँ वक, भ रि के सांविध न के
अनुच्छेद 317 के िहि, उन्हें केिि र ष्टरपवि ि र (र ज्यप ि ि र नहीां) हट य ज सकि है । र ष्टरपवि उन्हें उसी आध र पर और
उसी िरीके से हट सकिे हैं जैसे िह यूपीएससी के अध्यक्ष य सदस्य को हट सकिे हैं । र ष्टरपवि दु व्यििह र के विए SPSC के
अध्यक्ष य वकसी अन्य सदस्य को भी हट सकिे हैं ।
Source: Laxmikanth- Chapter 44 State Public Service Commission
https://www.ndtv.com/india-news/tamil-nadu-governor-rn-ravi-appoints-5-state-service-commission-
members-5070105

Q.70)
Ans) a
Exp) विकल्प a सही उत्तर है ।
अिमूल्यन वकसी दे ि की मुद्र के मूल्य को वकसी अन्य मुद्र य म नक के स पेक्ष ज नबूझकर नीचे की ओर सम योवजि करन
है । यह एक मौवद्रक नीवि उपकरण है वजसक उपयोर्ग वनविि विवनमय दर य अधि -वनविि विवनमय दर ि िे दे िोां ि र वकय
ज ि है ।
कथन I सही है : अिमूल्यन घरे िू िस्तुओ ं के सापेक्ष विर्दे शी िस्तुओ ं की कीमत बढाकर र्दे श के भुगतान संतुिन की
क्तस्थवत में सुधार कर सकता है ।
अथ ि ि्, अिमूल्यन से घरे िू िस्तुओां के स पेक्ष आय ि की ि र्गि बढ ज िी है , वजससे उपभोि ओां को थथ नीय रूप से उत्प वदि
िस्तुओां को खरीदने के विए प्रोत्स वहि वकय ज ि है । इससे आय ि कम हो ज ि है और वनय ि ि बढ ज ि है क्ोांवक विदे िी
िस्तुएाँ अपेक्ष कृि अवधक महां र्गी हो ज िी हैं । निीजिन, यह व्य प र घ टे को कम करके और अांिरर ष्टरीय ब ज रोां में
प्रविस्पध ि त्मकि बढ कर दे ि के भुर्गि न सांिुिन को बेहिर बन ने में मदद कर सकि है ।
कथन II सही है और कथन I का सही स्पिीकरण है : जब क ई र्दे श अपनी मुद्रा का अिमूल्यन करता है , त विर्दे शी मुद्रा
की तुिना में घरे िू मुद्रा का मूल्य घट जाता है। पररण मस्वरूप, घरे िू उपभोि ओां के विए आय ि अवधक महां र्ग हो ज ि है
क्ोांवक उन्हें अब उिनी ही म त्र में विदे िी स म न खरीदने के विए अवधक घरे िू मुद्र की आिश्यकि होिी है ।
दू सरी ओर, अिमूल्यन से घरे िू स म न विदे िी उपभोि ओां के विए सस्त हो ज ि है । विदे िी खरीद रोां को उिनी ही म त्र में
घरे िू स म न खरीदने के विए अपनी मुद्र कम खचि करनी पड़िी है ।
जैसे-जैसे वनय ि ि बढि है और आय ि घटि है , व्य प र घ ट कम होने से आम िौर पर भुर्गि न सांिुिन बेहिर होि है । एक
दे ि जो अपनी मुद्र क अिमूल्यन करि है िह अपने कम महां र्गे वनय ि ि की अवधक म ां र्ग के क रण अपन घ ट कम कर
सकि है ।
Source: Effects of a Devaluation on a Trade Balance in: IMF Staff Papers Volume 1952 Issue 001 (1952)
leec106.pdf (ncert.nic.in)

Forum Learning Centre: Delhi - 2nd Floor, IAPL House, 19 Pusa Road, Karol Bagh, New Delhi - 110005 | Patna - 2nd floor, AG Palace, E Boring Canal
Road, Patna, Bihar 800001 | Hyderabad - 1st & 2nd Floor, SM Plaza, RTC X Rd, Indira Park Road, Jawahar Nagar, Hyderabad, Telangana 500020
9311740400, 9311740900 | https://academy.forumias.com | admissions@forumias.academy | helpdesk@forumias.academy

[54]
PTS 2024 | Test Code: 121405 - Solutions |

Q.71)
Ans) a
Exp) विकल्प a सही उत्तर है ।
कॉस्िहें वसि एां ड प्रोग्रेवसि एग्रीमेंट फॉर टर ां स-पैवसवफक प टि नरविप के विए व्य पक और प्रर्गवििीि समझौि (CPTPP) एक
महत्वपूणि व्य प र समझौि है वजसमें प्रि ां ि मह स र्गर की सीम से िर्गे कई दे ि ि वमि हैं ।अमेररक की ि पसी के एक स ि
से भी कम समय के ब द, िेष 11 सदस्य 23 जनिरी 2018 को एक सांिोवधि व्य प र समझौिे पर सहमि हए, जो अमेररक के
वबन आर्गे बढे र्ग । समझौिे को CPTPP बनने के विए पुनजीविि वकय र्गय थ ।
कथन 1 सही है : CPTPP अपने सर्दस् र्दे श ं के बीि एक मुि व्यापार समझौते के रूप में कायि करता है । इस समझौिे
क उद्दे श्य टै ररफ और अन्य व्य प र ब ध ओां को कम य सम प्त करके व्य प र को सुविध जनक बन न है , वजससे भ र्ग िेने
ि िे दे िोां के बीच आवथिक विक स और समृस्द्ध को बढ ि वमिेर्ग ।

कथन 2 गित है : यूके के शावमि ह ने के साथ, िैविक सकि घरे िू उत्पार्द में CPTPP सर्दस् र्दे श ं का सामूवहक
य गर्दान िगभग 15% (50% नही ं) है । यह CPTPP विि स्तर पर सबसे बडे व्यापाररक ब्लॉक ं में से एक है । यह
समझौिे के भीिर सदस्य दे िोां के पय ि प्त आवथिक प्रभ ि और प्रभ ि को रे ख ां वकि करि है ।
कथन 3 गित है : न त संयुि राज्य अमेररका और न ही रूस CPTPP का वहस्ा हैं । जबवक संयुि राज्य अमेररका
शुरू में TPP के विए बातिीत में शावमि था, िह टर म्प प्रि सन के िहि समझौिे से हट र्गय । इसी िरह, रूस CPTPP में
ि वमि नहीां हआ है ।
Source: https://www.weforum.org/agenda/2023/04/cptpp-trade-tariff-brexit-uk/
https://www.thehindu.com/news/international/uk-formally-signs-up-to-trans-pacific-trading-
bloc/article67087710.ece

Q.72)
Ans) a
Exp) विकल्प a सही उत्तर है ।
वडवजटि सहमवत अवधग्रहण (वडवजटि कंसेंट एक्तिवजशन; DCA) प्रविया टे िीकॉम िावणक्तज्यक संिार ग्राहक िरीयता
विवनयम, 2018 (TCCCPR 2018) में उक्तिक्तित वर्दशावनर्दे श ं के अनुसार ग्राहक ं की सहमवत के आग्रह, प्रवतधारण
और िापसी की सुविधा प्रर्दान करती है । इस प्रविय के म ध्यम से प्र प्त सहमवि डे ट को वडस्ट्रीब्यूटेड िेजर टे क्नोिॉजी

Forum Learning Centre: Delhi - 2nd Floor, IAPL House, 19 Pusa Road, Karol Bagh, New Delhi - 110005 | Patna - 2nd floor, AG Palace, E Boring Canal
Road, Patna, Bihar 800001 | Hyderabad - 1st & 2nd Floor, SM Plaza, RTC X Rd, Indira Park Road, Jawahar Nagar, Hyderabad, Telangana 500020
9311740400, 9311740900 | https://academy.forumias.com | admissions@forumias.academy | helpdesk@forumias.academy

[55]
PTS 2024 | Test Code: 121405 - Solutions |

(DLT) प्लेटफॉमि पर बन ए रख ज ि है , वजसे वििेष रूप से ि वणस्ज्यक सांच र के प्रबांधन के विए TCCCPR-2018 के द यरे में
थथ वपि वकय र्गय है ।
कथन 1 गित है : वडवजटि सहमवत अवधग्रहण (DCA) भारतीय र्दूरसंिार वनयामक प्रावधकरण (TRAI) द्वारा स्थावपत
वनयम ं क संर्दवभित करता है , I & B मांत्र िय ि र नहीां। इन विवनयमोां क उद्दे श्य दू रसांच र ि वणस्ज्यक सांच र के विए ग्र हक
सहमवि प्र प्त करने और प्रबांवधि करने की प्रविय को सुव्यिस्थथि करन है (यह मूि रूप से स्पैम के खिरे को रोकने के विए
है ), जैस वक दू रसांच र ि वणस्ज्यक सांच र ग्र हक िरीयि विवनयम, 2018 (TCCCPR-2018) के िहि अवनि यि है ।
कथन 2 सही है : वडवजटि सहमवत अवधग्रहण (DCA) के प्राथवमक उद्दे श्य ं में से एक स्पैम संर्देश ं के मुद्दे क संब वधत
करना है , वजसे आमतौर पर अनिाहे िावणक्तज्यक संिार (UCC) के रूप में जाना जाता है । DCA को ि र्गू करके, टर ई क
इर द ग्र हक की सहमवि प्र प्त करने के विए एक म नकीकृि और विवनयवमि प्रविय थथ वपि करने क है , वजससे अि ां वछि
ि वणस्ज्यक सांच र की म त्र कम हो सके।
सहमवि म ां र्गने ि िे सांदेि भेजने के विए एक स म न्य िॉटि कोड 127xxx क उपयोर्ग वकय ज एर्ग । सांवक्षप्त कोड के म ध्यम
से भेजे र्गए सहमवि म ां र्गने ि िे सांदेि में उद्दे श्य, सहमवि क द यर और प्रमुख इक ई/ब्र ां ड न म क स्पष्ट रूप से उल्लेख वकय
ज एर्ग । सहमवि म ां र्गने ि िे सांदेिोां में केिि िेिसूचीबद्ध यूआरएि/एपीके/ओटीटी विांक/कॉि बैक नांबर आवद क उपयोर्ग
वकय ज एर्ग । ग्र हकोां को सहमवि प्र स्प्त पुवष्टकरण सांदेि में सहमवि रद्द करने से सांबांवधि ज नक री भी होर्गी
कथन 3 गित है : बैंक ं और बीमा कंपवनय ं क टर ाई द्वारा जारी वडवजटि सहमवत अवधग्रहण (DCA) वनयम ं का पािन
करने से छूट नही ं है । बैंकोां और बीम कांपवनयोां सवहि सभी सांथथ ओां को ि वणस्ज्यक सांच र के विए ग्र हक की सहमवि प्र प्त
करने और प्रबांवधि करने के विए इन वनयमोां क प िन करन आिश्यक है ।
Source: https://pib.gov.in/PressReleaseIframePage.aspx?PRID=1975372

Q.73)
Ans) d
Exp) विकल्प d सही उत्तर है।
रािरीय भू-स्थावनक नीवत, 2022 क हाि ही में विज्ञान और प्रौद्य वगकी मंिािय द्वारा अवधसूवित वकया गया था। इससे
भू-स्थावनक डे टा, इसके विवनयमन और अनुप्रय ग पर ध्यान केंवद्रत ह गया।
कथन 1 सही है : भू-स्थावनक डे टा पृथ्वी की सतह पर वकसी विशेष स्थान पर िस्तुओ,ं घटनाओं या घटनाओं से जुडी
समय-आधाररत जानकारी क संर्दवभित करता है । भू-थथ वनक डे ट आमिौर पर भौर्गोविक ज नक री (आमिौर पर पृथ्वी पर
समन्वय करि है ) और एवटर ब्यूट ज नक री (सांबांवधि िस्तु, घटन य घटन की वििेषि एां ) को टें पोरि ज नक री (समय य
जीिन क ि वजस पर थथ न और वििेषि एां मौजूद हैं ) के स थ जोड़िी है ।
कथन 2 सही है : रािरीय भू-स्थावनक नीवत, 2022, ब्लू इकॉन मी क बढािा र्दे ने के विए अंतर्दे शीय जि और समुद्र की
सतह स्थिाकृवत के उच्च-ररज़ॉल्यूशन भू-स्थावनक डे टा विकवसत करने का प्रयास करती है । इसमें समुद्री और िटीय
प ररस्थथविकी प्रण वियोां क सांरक्षण करिे हए आवथिक विक स के विए समुद्री सांस धनोां क सिि उपयोर्ग ि वमि है ।
कथन 3 सही है : भू-स्थावनक सूिना प्रणािी (GIS) में डे टा की दृश्य मानवििण, ररम ट सेंवसंग डे टा, भूवम ररकॉडि और
कृवष जनगणना जैसे विवभन्न स्र त ं से जानकारी क एकीकृत करना शावमि है । GIS भूवम आिरण विश्लेषण, िनस्पवि
वनर्गर नी और वमट्टी की नमी के आकिन की सुविध प्रद न करि है , वजससे सटीक कृवष को बढ ि वमिि है । वप्रवसजन
एग्रीकल्चर (PA) उच्च प्रौद्योवर्गकी सेंसर और विश्लेषण उपकरणोां क उपयोर्ग करके फसि की पैद ि र में सुध र और प्रबांधन
वनणियोां में सह यि करने क विज्ञ न है ।
Source:
https://www.surveyofindia.gov.in/webroot/UserFiles/files/National%20Geospatial%20Policy.pdf
https://www.manage.gov.in/studymaterial/GIS.pdf (Pg no 32)
https://www.ibm.com/topics/geospatial-data

Forum Learning Centre: Delhi - 2nd Floor, IAPL House, 19 Pusa Road, Karol Bagh, New Delhi - 110005 | Patna - 2nd floor, AG Palace, E Boring Canal
Road, Patna, Bihar 800001 | Hyderabad - 1st & 2nd Floor, SM Plaza, RTC X Rd, Indira Park Road, Jawahar Nagar, Hyderabad, Telangana 500020
9311740400, 9311740900 | https://academy.forumias.com | admissions@forumias.academy | helpdesk@forumias.academy

[56]
PTS 2024 | Test Code: 121405 - Solutions |

Q.74)
Ans) a
Exp) विकल्प a सही उत्तर है ।
वििेष वधक र सवमवि सांसदीय और विध यी ढ ांचे में एक महत्वपूणि थथ न रखिी है , वजस पर सदस्योां के वििेष वधक रोां और
उन्मुस्ियोां की सुरक्ष की वजम्मेद री है । यह सुरक्ष र्ग रां टी दे िी है वक क नून वनम ि ि ब हरी हस्तक्षेप य दब ि से मुि होकर
अपनी वज़म्मेद ररय ाँ वनभ सकिे हैं । जब भी इन वििेष वधक रोां के उल्लांघन के द िे होिे हैं , िो सवमवि को ज ां च करने के विए
कह ज ि है , च हे ये सदस्योां ि र वकए र्गए होां य ब हरी दिोां ि र वकसी सदस्य की विध यी कििव्योां को पूर करने की क्षमि
को प्रभ विि करने ि िे होां।
संरिना एिं वनयुक्तियााँ:
• ि कसभा में 15 सर्दस् ं की एक सवमवत ह ती है , जबवक उच्च सर्दन राज्यसभा में 10 सर्दस् शावमि ह ते हैं ।
• सवमवत की संरिना में तटस्थता बनाए रिने िािे विवभन्न राजनीवतक र्दि ं के सर्दस् शावमि हैं । यह वििरण आम िौर
पर विध वयक के भीिर प वटि योां के प्रविवनवधत्व के अनुप ि में होि है ।
• ि कसभा में अध्यक्ष सवमवत के सर्दस् ं क नामांवकत करता है , जबवक र ज्यसभ में यह वजम्मेद री सभ पवि की होिी है ।
सवमवि के अध्यक्ष की वनयुस्ि िोकसभ अध्यक्ष ि र िोकसभ सवमवि के विए और र ज्यसभ के सभ पवि ि र र ज्यसभ
सवमवि के विए सवमवि के सदस्योां में से की ज िी है ।
कथन 1 सही है: 18 म चि 1986 से र ज्य सभ सदस्य (दि-बदि के आध र पर अयोग्यि ) वनयम, 1985 के ि र्गू होने के स थ,
सवमवि को एक अविररि क यि सौांप र्गय है ।
यवद अध्यक्ष म मिे की प्रकृवि और पररस्थथवियोां को ध्य न में रखिे हए सांिुष्ट हैं िो िह ऐस कर सकिे हैं । यवद ऐस करन
आिश्यक य समीचीन है , िो दिबदि के आध र पर वकसी सदस्य की अयोग्यि के सांबांध में एक य वचक सवमवि को प्र रां वभक
ज ां च करने और उसे एक ररपोटि सौांपने के विए भेजें।
कथन 2 ग़ित है : विशेषावधकार सवमवत केिि सज़ा की वसफाररश कर सकती है । सवमवत सर्दन क िेतािनी जारी
करने, फटकार िगाने , वनिंवबत करने या र्दु ििभ मामि ं में वकसी सर्दस् क वनष्कावसत करने की सिाह र्दे सकती है ।
दोनोां सदनोां की सवमवि ि र अपन ई ज ने ि िी परां पर यह है वक यवद वजस स ां सद के स्खि फ वििेष वधक र क म मि
उठ य र्गय है , िह अयोग्य म फी म ां र्गि है , िो मुद्दे को ि ां ि कर वदय ज ि है , और िह आर्गे कोई क रि ि ई नहीां करने की
वसफ ररि करिी है । स थ ही, सज़ दे ने की िस्ि केिि सदन के प स है ।
कथन 3 गित है : सवमवत के अध्यक्ष की वनयुक्ति राज्य सभा के सभापवत द्वारा सवमवत के सर्दस् ं में से की जाती है ।
ह ि ाँ वक 1958 से थथ वपि परां पर के अनुस र, उप ध्यक्ष को हमेि सवमवि के सदस्य के रूप में न वमि वकय ज ि है और
इसविए, उन्हें सवमवि के अध्यक्ष के रूप में वनयुि वकय ज ि है । इस प्रक र, र ज्यसभ क सभ पवि सवमवि क पदे न अध्यक्ष
नहीां होि है ।
ज्ञानक ष:
भ रि में सांसद के वििेष वधक र कई स्रोिोां से प्र प्त होिे हैं , जो अनुवचि ब हरी हस्तक्षेप के वबन विध यी वनक य के कुिि
क मक ज को सुवनविि करिे हैं । इन स्रोिोां में ि वमि हैं :
• भ रि क सांविध न: अनुच्छेद 105 (सांसद के विए) और अनुच्छेद 194 (र ज्य विध नमांडिोां के विए)।
• िैध वनक क नून: सांसद ि र प ररि कुछ अवधवनयम और क नून विविष्ट वििेष वधक र प्रद न करिे हैं । उद हरण के विए,
सांसद (अयोग्यि वनि रण) अवधवनयम, 1959।
• सांसदीय सम्मेिन: वपछिे कुछ िषों में, िेस्ट्वमांस्ट्र वसस्ट्म के आध र पर कुछ सम्मेिन और प्रथ एां विकवसि हई हैं , वजनक
भ रि बड़े पैम ने पर प िन करि है ।
• न्य वयक वनणिय: सांसदीय वििेष वधक रोां से सांबांवधि म मिोां पर सिोच्च न्य य िय और उच्च न्य य ियोां की व्य ि एां और
फैसिे वििेष वधक रोां के स्रोि के रूप में क यि करिे हैं ।
• सदन के वनयम: सांसद के प्रत्येक सदन के अपने वनयम हैं जो इसके क मक ज को वनयांवत्रि करिे हैं । ये वनयम, ह ि ां वक
िैध वनक प्र िध नोां और सांविध न के अधीन हैं , अपने सदस्योां के वििेष वधक रोां को पररभ वषि और विस्तृि कर सकिे हैं ।

Forum Learning Centre: Delhi - 2nd Floor, IAPL House, 19 Pusa Road, Karol Bagh, New Delhi - 110005 | Patna - 2nd floor, AG Palace, E Boring Canal
Road, Patna, Bihar 800001 | Hyderabad - 1st & 2nd Floor, SM Plaza, RTC X Rd, Indira Park Road, Jawahar Nagar, Hyderabad, Telangana 500020
9311740400, 9311740900 | https://academy.forumias.com | admissions@forumias.academy | helpdesk@forumias.academy

[57]
PTS 2024 | Test Code: 121405 - Solutions |

Source: https://eparlib.nic.in/bitstream/123456789/63246/1/14_Privileges_11.pdf
https://indianexpress.com/article/explained/lok-sabha-congress-adhir-ranjan-chowdhury-privileges-
committee-explained-8887311/
https://indianexpress.com/article/cities/delhi/aap-leader-sanjay-singh-oath-mp-privileges-committee-
pending-matter-rajya-sabha-9145161/
https://cms.rajyasabha.nic.in/UploadedFiles/Procedure/RajyaSabhaAtWork/English/834-
967/CHAPTER25.pdf

Q.75)
Ans) b
Exp) विकल्प b सही उत्तर है।
विकल्प 1, 3 और 5 सही हैं और विकल्प 2 और 4 गित हैं : केंद्रीय स ां स्िकी सांर्गठन (CSO) 1950 से औद्योवर्गक उत्प दन
सूचक ां क (IIP) के सांकिन और प्रक िन के विए वजम्मेद र है ।
IIP क िास्पेयसि के फामूििे का उपय ग करके उत्पार्दन सापेक्ष ं के एक सरि भाररत अंकगवणतीय माध्य के रूप में
संकवित वकया गया है । IIP एक क्व ां टम सूचक ां क है , वजसमें िस्तुओां क उत्प दन भौविक रूप में व्यि वकय ज ि है ।
ह ि ाँ वक, मिीनरी, मिीन टू ल्स, जह ज वनम ि ण आवद जैसी कुछ िस्तुओां के सांबांध में ररपोवटिं र्ग की इक ई मूल्य के सांदभि में है ।
ऐसे म मिोां में उत्प दन मूल्य क म वसक आाँ कड़ सबसे पहिे उद्योर्ग मांत्र िय के आवथिक सि हक र के क य ि िय ि र ज री
सांबांवधि श्रेवणयोां के थोक मूल्य सूचक ां क (WPI) ि र वनक ि ज ि है । सांयुि र ष्टर स ां स्िकी क य ि िय (UNSO) ि र
अनुिांवसि IIP के द यरे में खनन, विवनम ि ण, वनम ि ण, वबजिी, र्गैस और प नी की आपूविि ि वमि है । िेवकन डे ट उपिब्धि की
ब ध ओां के क रण, भ रि में सांकविि IIP ने वनम ि ण, र्गैस और जि आपूविि क्षेत्रोां को ब हर रख है ।
Source: https://mospi.gov.in/54-index-industrial-
production#:~:text=The%20scope%20of%20the%20IIP,gas%20and%20water%20supply%20sectors

Q.76)
Ans) d
Exp) विकल्प d सही उत्तर है।
र ज्यप ि र ज्य क मुि क यिक री प्रमुख होि है । िह र ज्य और केंद्र ि वसि प्रदे िोां क औपच ररक प्रमुख होि है । इनकी
वनयुस्ि भ रि के र ष्टरपवि ि र की ज िी है और ये एक महत्वपूणि सांिैध वनक पद पर होिे हैं ।
कथन 1 र्गिि है : जब र ज्यप ि र ज्य वििविद्य ियोां के कुि वधपवि के रूप में क यि करिे हैं , िो िे मांवत्रपररषद से स्विांत्र रूप से
क यि करिे हैं । सुप्रीम कोटि ने ह ि ही में फैसि सुन य वक र ज्य वििविद्य िय के कुि वधपवि के रूप में क यि करने ि िे
र ज्यप ि र ज्य की मांवत्रपररषद की सि ह से ब ध्य नहीां हैं ।
कथन 2 गित है : संविधान का अनुच्छेर्द 200 राज्यपाि ं क राज्य सरकार से क ई विधेयक प्राप्त ह ने पर या त पाररत
विधेयक ं पर सहमवत र्दे ने, अनुमवत र कने या रािरपवत के वििार के विए आरवक्षत करने का अवधकार र्दे ता है । विधेयकोां
पर सहमवि प्रद न करने के विए र ज्यप ि के विए सांविध न ि र कोई विविष्ट समय सीम नहीां िर्ग ई र्गई है ।
Source: https://indianexpress.com/article/india/crime/sc-governors-acting-as-chancellor-not-bound-by-
advice-of-ministers-9049408/
https://thewire.in/law/governors-not-empowered-to-indefinitely-withhold-assent-to-bills
https://www.thehindu.com/news/national/governors-can-pardon-prisoners-including-death-row-ones-
supreme-court/article35711818.ece
https://www.thehindu.com/news/national/other-states/west-bengal-governor-welcomes-supreme-court-
verdict-on-chancellors-role-in-appointment-of-v-cs/article67597233.ece

Forum Learning Centre: Delhi - 2nd Floor, IAPL House, 19 Pusa Road, Karol Bagh, New Delhi - 110005 | Patna - 2nd floor, AG Palace, E Boring Canal
Road, Patna, Bihar 800001 | Hyderabad - 1st & 2nd Floor, SM Plaza, RTC X Rd, Indira Park Road, Jawahar Nagar, Hyderabad, Telangana 500020
9311740400, 9311740900 | https://academy.forumias.com | admissions@forumias.academy | helpdesk@forumias.academy

[58]
PTS 2024 | Test Code: 121405 - Solutions |

Q.77)
Ans) b
Exp) विकल्प b सही उत्तर है।
ख द्य और कृवष सांर्गठन (FAO) ने "ख द्य सुरक्ष और पोषण क एविय प्रि ां ि क्षेत्रीय अििोकन 2023: स ां स्िकी और रुझ न"
िीषिक से एक ररपोटि ज री की है । इसक उद्दे श्य वहिध रकोां को भूख सम प्त करने, ख द्य सुरक्ष प्र प्त करने और पोषण में
सुध र की वदि में प्रर्गवि के ब रे में सूवचि करन है ।
ररपोटि के मुि वनष्कषि:

एवशया-प्रशांत क्षेि: भारत

1) महामारी और "5F" संकट (भ जन, िारा, ईंधन, 1) स्वस्थ आहार का सामर्थ्ि : 2021 में, 74% से अवधक
उििरक और वित्त) के दौर न, एविय प्रि ां ि क्षेत्र ने र्गम्भीर भारतीय (2020 में 76.2%) स्वस्थ आहार का ििि उठाने
रूप से वचांि जनक आाँ कड़ो क अनुभि वकय । में सक्षम नही ं थे।
2) विि के िर्गभर्ग आधे कुपोवषि िोर्ग एविय -प्रि ां ि क्षेत्र 2) अल्पप षण: भ रि की 16.6% आब दी अल्पपोवषि है ,
में रहिे हैं , जह ाँ पुरुषोां की िुिन में मवहि एाँ अवधक ख द्य वजसे आवथिक और स म वजक ि र्गि क स मन करन पड़
असुरवक्षि हैं । रह है ।
3) दवक्षणी एविय में अल्पपोषण की व्य पकि सबसे 3) बाि कुप षण की व्यापकता:
अवधक 15.6 प्रवििि है । a. स्ट्ां वटां र्ग (उम्र के वहस ब से कम ऊांच ई): प ां च
स ि से कम उम्र के 31.7% बच्चे।
b. िेस्स्ट्ां र्ग (ऊांच ई के मुक बिे कम िजन): 18.7%
(क्षेत्र में उच्चिम दर)
c. अवधक िजन: 2.8%।
4) जन्म के समय कम िजन: 27.4% (क्षेि में उच्चतम
क्षेिीय प्रसार)
5) एनीवमया: 2019 में 15-49 आयु िर्गि की 53% मवहि ओां
को एनीवमय थ (क्षेत्र में सबसे अवधक प्रस र)
6) स्तनपान: िैविक औसि (47.7%) की िुिन में भ रि में
वििुओां (0-5 महीने) में स्तनप न क प्रचिन (63.7%)
अवधक है ।
Source: https://forumias.com/blog/food-security-and-nutrition-report-2023-74-indians-could-not-
afford-healthy-diet-in-2021/
https://www.fao.org/documents/card/en?details=cc8228en

Q.78)
Ans) d
Exp) विकल्प d सही उत्तर है।
भ रिीय ररजिि बैंक ("RBI") ने 'वडवजटि ऋण में वडफॉल्ट ह वन र्ग रां टी पर वदि वनदे ि' ("DLG वदि वनदे ि") ज री वकए।
सरि शब् ं में, वडफॉल्ट हावन गारं टी विवनयवमत इकाई (रे गुिेटेड एं वटटी; RE-ज ऋणर्दाता है ) और ऋण सेिा
प्रर्दाताओं के बीि एक िेवडट ज क्तिम साझा करने की व्यिस्था है , वजसके िहि ऋण सेि प्रद ि (LSP) उध रकि ि के
वडफॉल्ट होने पर आरई को मुआिज दे ने की र्ग रां टी दे िे हैं ।
विकल्प 1, 2, 3 और 4 सही हैं : DLG वदि वनदे ि वडवजटि ऋण पररच िन में सांिग्न मुि रूप से सभी ि वणस्ज्यक बैंकोां (िघु
वित्त बैंकोां सवहि), प्र थवमक (िहरी) सहक री बैंकोां, र ज्य सहक री बैंकोां, केंद्रीय सहक री बैंकोां और र्गैर-बैंवकांर्ग वित्तीय
कांपवनयोां (आि स वित्त कांपवनयोां सवहि) पर ि र्गू होिे हैं ।

Forum Learning Centre: Delhi - 2nd Floor, IAPL House, 19 Pusa Road, Karol Bagh, New Delhi - 110005 | Patna - 2nd floor, AG Palace, E Boring Canal
Road, Patna, Bihar 800001 | Hyderabad - 1st & 2nd Floor, SM Plaza, RTC X Rd, Indira Park Road, Jawahar Nagar, Hyderabad, Telangana 500020
9311740400, 9311740900 | https://academy.forumias.com | admissions@forumias.academy | helpdesk@forumias.academy

[59]
PTS 2024 | Test Code: 121405 - Solutions |

Source: https://www.livelaw.in/law-firms/law-firm-articles-/lending-service-providers-digital-lending-
guidelines-rbi-luthra-and-luthra-law-offices-npa-msme-233257
https://www.rbi.org.in/scripts/FS_Notification.aspx?Id=12514
https://www.mondaq.com/india/fin-tech/1347832/guidelines-on-default-loss-guarantee-in-digital-
lending#:~:text=The%20Reserve%20Bank%20of%20India,guarantee%20(%22FLDG%22).

Q.79)
Ans) c
Exp) विकल्प c सही उत्तर है ।
ह ि ही में, र ष्टरीय म नि वधक र आयोर्ग (NHRC) ने NHRC ि र आयोवजि "भ रि में नॉमेवडक टर इब (NTs), सेमी नॉमेवडक
टर इब (SNTs) और डीनॉवटफ इड टर इब (DNTs)की सुरक्ष और आर्गे के प्रक्षेप पथ" पर एक पररचच ि के दौर न इडे ट आयोर्ग
की ररपोटि को ि र्गू करने की आिश्यकि पर जोर वदय ।
इडे ट आयोर्ग, वजसे आवधक ररक िौर पर नेिनि कमीिन फॉर डीनॉवटफ इड, नॉमेवडक एां ड सेमी नॉमेवडक टर इब्स (NCDNT)
कह ज ि है , 2015 में भ रि सरक र ि र र्गवठि एक आयोर्ग थ ।
इसकी थथ पन स म वजक और आवथिक चुनौवियोां क स मन करने ि िी र्गैर-अवधसूवचि, ख न बदोि और अधि -घुमांिू
जनज वियोां (DNT) के विए कल्य णक री उप योां की पहच न करने और वसफ ररि करने के विए की र्गई थी।
वसफाररशें :
• असूचीबद्ध DNT व्यस्ियोां को ओबीसी श्रेणी में ि वमि करें ।
• अत्य च रोां के विरुद्ध क नूनी सुरक्ष उप य बढ एाँ ।
• DNT, घुमांिू जनज वियोां (NT), और अधि -घुमांिू जनज वियोां (SNT) के विए एक थथ यी आयोर्ग की थथ पन करें ।
• महत्वपूणि DNT आब दी ि िे र ज्योां में समवपिि विभ र्ग बन एां ।
• DNT जनसांि और वििरण वनध ि ररि करने के विए व्य पक सिेक्षण आयोवजि करें
Source: https://www.thehindu.com/news/national/protection-of-nomadic-tribes-nhrc-stress-on-need-
for-implementation-of-the-idate-commission-report/article67757054.ece
https://sansad.in/getFile/loksabhaquestions/annex/179/AU1397.pdf?source=pqals

Q.80)
Ans) a
Exp) विकल्प a सही उत्तर है ।
सांयुि र ष्टर जनसांि कोष (UNFPA) ने इां टरनेिनि इां स्ट्ीट्यूट फॉर पॉपुिेिन स इां सेज (IIPS) के सहयोर्ग से "इां वडय एवजांर्ग
ररपोटि 2023" ज री की है । "इां वडय एवजांर्ग ररपोटि 2023" भ रि में िृद्ध व्यस्ियोां की रहने की स्थथवि और कल्य ण क र्गहन
विश्लेषण प्रस्तुि करिी है ।
सांयुि र ष्टर जनसांि कोष (UNFPA) के प्रमुख वनष्कषि:
• एवशया और यूर प में घटती जनसंख्या: 2100 तक, सबसे तेज़ी से घटने िािी आबार्दी एवशया और यूर प में ह गी।
• जनसंख्या का बुढापा: िैविक स्तर पर, 2022 में कुि जनसंख्या का िगभग 14% वहस्ा 60 िषि और उससे अवधक
आयु के ि ग ं का है । 2050 तक, यह वहस्ा बढकर 22% ह ने की उम्मीर्द है ।
• विक सिीि दे िोां में कम प्रजनन दर की प्र रां वभक िुरुआि: उनकी विक स य त्र में बहि पहिे ही प्रजनन स्तर में महत्वपूणि
वर्गर िट दे खी ज रही है ।
भारत में िृद्ािस्था पररदृश्य:
• जनसंख्या का तीव्रता से िृद्ािस्था की ओर अग्रसर ह ना: 2022 में, भ रि की 10.5% आब दी 60 िषि और उससे अवधक
आयु की थी। 2050 िक यह वहस्सेद री दोर्गुनी होकर 20.8% हो ज एर्गी।

Forum Learning Centre: Delhi - 2nd Floor, IAPL House, 19 Pusa Road, Karol Bagh, New Delhi - 110005 | Patna - 2nd floor, AG Palace, E Boring Canal
Road, Patna, Bihar 800001 | Hyderabad - 1st & 2nd Floor, SM Plaza, RTC X Rd, Indira Park Road, Jawahar Nagar, Hyderabad, Telangana 500020
9311740400, 9311740900 | https://academy.forumias.com | admissions@forumias.academy | helpdesk@forumias.academy

[60]
PTS 2024 | Test Code: 121405 - Solutions |

• िृद् ं की संख्या बच्च ं की संख्या से अवधक ह जाएगी: 2046 िक बुजुर्गों की सांि बच्चोां (15 िषि से कम) की सांि से
अवधक हो ज एर्गी।
ज्ञानक ष:
"ग्रेइांर्ग" घटन क् है ?
• ग्रेइांर्ग होन बढिी उम्र, घटिी प्रजनन क्षमि और जनसांि के बड़े समूहोां के िृद्ध िथथ की ओर बढने के सांचयी पररण म को
सांदवभिि करि है ।
1970 के दिक में, 15 से 24 िषि के युि ओां की सांि िृद्ध िोर्गोां की िुिन में िीन र्गुन अवधक थी। 2050 िक ये आयु िर्गि
बर बर हो ज येंर्गे।
Source: https://forumias.com/blog/on-increasing-global-elderly-population-the-world-is-getting-older-
can-india-
cope/#:~:text=According%20to%20UNFPA's%20India%20Aging,share%20will%20double%20to%2020.8%
25.
https://india.unfpa.org/en/news/india-ageing-report-2023-unveils-insights-elderly-care-challenges-and-
solutions#:~:text=The%20%22India%20Ageing%20Report%202023%22%20serves%20as%20a%20valuabl
e%20resource,older%20citizens%20in%20the%20country.

Q.81)
Ans) c
Exp) विकल्प c सही उत्तर है ।
भ रि में, विवभन्न प्र वधक ररयोां को अपने क य ि िय में प्रिेि करने से पहिे िपथ य प्रविज्ञ न िेन और उस पर हस्त क्षर करन
च वहए। इस सांदभि में, उपर ष्टरपवि, र ज्य के र ज्यप ि, उच्च न्य य िय के मुि न्य य धीि और र ज्य में मांवत्रपररषद को िपथ
वनम्नविस्खि अवधक ररयोां ि र वदि ई ज िी है :
युग्म 1 सही है : भ रि क उपर ष्टरपवि भ रि के र ष्टरपवि के ब द दे ि में दू सर सबसे बड़ पद रखि है । उपर ष्टरपवि को पद की
िपथ भ रि के र ष्टरपवि य र ष्टरपवि ि र उसकी ओर से वनयुि वकसी व्यस्ि ि र वदि ई ज िी है ।
युग्म 2 सही है : राज्यपाि क पर्द की शपथ संबंवधत राज्य उच्च न्यायािय के मुख्य न्यायाधीश द्वारा वर्दिाई जाती है , और
उनकी अनुपस्थथवि में, उस न्य य िय के िररष्ठिम न्य य धीि ि र िपथ वदि ई ज िी है । र ज्यप ि के क यों क वनििहन करने
ि ि प्रत्येक व्यस्ि भी इसी प्रक र की िपथ य प्रविज्ञ न िेि है ।
युग्म 3 गित है : उच्च न्यायािय के न्यायाधीश (उच्च न्यायािय के मुख्य न्यायाधीश सवहत) के रूप में वनयुि व्यक्ति क
अपने कायाििय में प्रिेश करने से पहिे शपथ या प्रवतज्ञान िेना और हस्ताक्षर करना ह गा। उच्च न्य य िय के मुि
न्य य धीि को िपथ र ज्य के र ज्यप ि य उनके ि र इस उद्दे श्य के विए वनयुि वकसी व्यस्ि ि र वदि ई ज िी है , न वक
भ रि के मुि न्य य धीि ि र ।
युग्म 4 सही है : एक राज्य में, राज्यपाि मुख्यमंिी की वनयुक्ति करता है, जबवक अन्य मंविय ं की वनयुक्ति राज्यपाि
द्वारा मुख्यमंिी की सिाह के आधार पर की जाती है । पद ग्रहण करने से पहिे, मांत्री को सांबांवधि र ज्य के र ज्यप ि ि र
पद और र्गोपनीयि की िपथ वदि यी ज िी है ।
Source: Indian Polity by Laxmikanth

Q.82)
Ans) d
Exp) विकल्प d सही उत्तर है।

Forum Learning Centre: Delhi - 2nd Floor, IAPL House, 19 Pusa Road, Karol Bagh, New Delhi - 110005 | Patna - 2nd floor, AG Palace, E Boring Canal
Road, Patna, Bihar 800001 | Hyderabad - 1st & 2nd Floor, SM Plaza, RTC X Rd, Indira Park Road, Jawahar Nagar, Hyderabad, Telangana 500020
9311740400, 9311740900 | https://academy.forumias.com | admissions@forumias.academy | helpdesk@forumias.academy

[61]
PTS 2024 | Test Code: 121405 - Solutions |

विवनमय दर िह दर है वजस पर एक मुद्र को दू सरी मुद्र के विए विवनमय वकय ज एर्ग और यह दे िोां के बीच व्य प र और धन
की आि ज ही को प्रभ विि करिी है । न मम त्र प्रभ िी विवनमय दर (NEER) और ि स्तविक प्रभ िी विवनमय दर (REER) र ष्टर
की मुद्र विवनमय दर वनध ि ररि करने के विए वनयोवजि िकनीकें हैं ।
कथन 1 गित है : NEER और REER र्द न ं विर्दे शी मुद्राओं की एक ट करी के मुकाबिे वकसी र्दे श की मुद्रा विवनमय र्दर
के भाररत औसत का प्रवतवनवधत्व करते हैं । मुख्य अंतर REER में है , जो मुद्र स्फीवि दर क क रक है । REER अपने
व्य प ररक स झेद रोां की मुद्र स्फीवि दर के स पेक्ष घरे िू दे ि की मुद्र स्फीवि दर के आध र पर NEER को सम योवजि करि है ।
इस प्रक र, REER घरे िू कीमिोां और विदे िी कीमिोां के अनुप ि ि र सम योवजि NEER क भ ररि औसि है ।
कथन 2 गित है : एक उच्च REER व्यापार प्रवतस्पधाित्मकता में वगरािट का संकेत र्दे ता है । जब REER अवधक ह ता है ,
त वकसी र्दे श का वनयाित महंगा ह जाता है , जबवक विर्दे शी िस्तुओ ं की तुिना में आयात सस्ता ह जाता है । स थ ही,
उच्च NEER घरे िू मुद्र की सर हन को दि िि है , वजसके पररण मस्वरूप विदे िी ब ज रोां में घरे िू िस्तुओां की व्य प र
प्रविस्पध ि त्मकि कम हो ज िी है । इसविए, यह कथन वक NEER में िृस्द्ध घरे िू िस्तुओां की बढी हई व्य प र प्रविस्पध ि त्मकि
क सुझ ि दे िी है , र्गिि है ।
Source:
https://www.indiabudget.gov.in/budget2022-23/economicsurvey/doc/echapter.pdf

Q.83)
Ans) a
Exp) विकल्प a सही उत्तर है ।
र ष्टरीय औद्योवर्गक र्गविय र विक स क यििम (National Industrial Corridor Development Programme, NICDP) भ रि
क सबसे महत्व क ां क्षी बुवनय दी ढ ां च क यििम है वजसक िक्ष्य नए औद्योवर्गक िहरोां को "स्म टि वसटी" के रूप में विकवसि
करन और बुवनय दी ढ ां च क्षेत्रोां में अर्गिी पीढी की प्रौद्योवर्गवकयोां को एकीकृि करन है ।
कथन 1 सही है: भ रि सरक र NICDP के वहस्से के रूप में विवभन्न औद्योवर्गक र्गविय र पररयोजन ओां क विक स कर रही है ,
वजसक उद्दे श्य भ रि में भविष्य के औद्योवर्गक िहरोां क विक स करन है जो दु वनय के सिोत्तम विवनम ि ण और वनिेि थथिोां के
स थ प्रविस्पध ि कर सकें।
कथन 2 गित है : रािरीय औद्य वगक गवियारा विकास और कायािन्वयन टर स्ट् (NICDIT) NICDP क िागू करने के विए
उत्तरर्दायी है । NICDIT ि वणज्य और उद्योर्ग मांत्र िय के उद्योर्ग सांिधिन और आां िररक व्य प र विभ र्ग (DPIIT) के प्रि सवनक
वनयांत्रण में है । इसविए, NICDP को ि वणज्य और उद्योर्ग मांत्र िय (सड़क पररिहन और र जम र्गि मांत्र िय नहीां) के समग्र
प्रि सवनक वनयांत्रण के िहि क य ि स्न्वि वकय ज रह है ।

कथन 3 गित है : ितिमान में, भारत सरकार िरणबद् तरीके से र्दे श भर में रािरीय औद्य वगक गवियारा कायििम के
वहस्े के रूप में ग्यारह (िार नही ं) औद्य वगक गवियारा पररय जनाओं का विकास कर रही है । इन र्गविय रोां में वदल्ली
मुांबई औद्योवर्गक र्गविय र (DMIC), चेन्नई बेंर्गिुरु औद्योवर्गक र्गविय र (CBIC), अमृिसर कोिक ि औद्योवर्गक र्गविय र

Forum Learning Centre: Delhi - 2nd Floor, IAPL House, 19 Pusa Road, Karol Bagh, New Delhi - 110005 | Patna - 2nd floor, AG Palace, E Boring Canal
Road, Patna, Bihar 800001 | Hyderabad - 1st & 2nd Floor, SM Plaza, RTC X Rd, Indira Park Road, Jawahar Nagar, Hyderabad, Telangana 500020
9311740400, 9311740900 | https://academy.forumias.com | admissions@forumias.academy | helpdesk@forumias.academy

[62]
PTS 2024 | Test Code: 121405 - Solutions |

(AKIC), पूिी िट औद्योवर्गक र्गविय र (ECIC) के स थ विज र्ग चेन्नई औद्योवर्गक र्गविय र (VCIC), बेंर्गिुरु मुांबई औद्योवर्गक
र्गविय र (BMIC) और कोयम्बटू र के म ध्यम से कोस्च्च िक CBIC क विस्त र ि वमि हैं । इसके अविररि, है दर ब द न र्गपुर
औद्योवर्गक र्गविय र (HNIC), है दर ब द ि रां र्गि औद्योवर्गक र्गविय र (HWIC), है दर ब द बेंर्गिुरु औद्योवर्गक र्गविय र (HBIC),
ओवडि आवथिक र्गविय र (OEC), और वदल्ली न र्गपुर औद्योवर्गक र्गविय र (DNIC) भी इस महत्व क ां क्षी पहि क वहस्स हैं ।
कथन 4 गित है : रािरीय औद्य वगक गवियारा विकास वनगम (NICDC) एक पररय जना विकास कंपनी है िेवकन
NICDC के ितिमान इक्तिटी शेयरधारक भारत सरकार (49%) हैं , वजनक प्रविवनवधत्व उद्योर्ग और आां िररक व्य प र सांिधिन
विभ र्ग (DPIIT), ज प न बैंक फॉर इां टरनेिनि कोऑपरे िन (JBIC) (26%) और ह उवसांर्ग एां ड अबिन डे ििपमेंट कॉरपोरे िन
(HUDCO) (19.9%), इां वडय इां फ् स्ट्रक्चर फ इनेंस कांपनी विवमटे ड (IIFCL) (4.1%) और जीिन बीम वनर्गम (LIC) (1.0%)।
जैसे वित्तीय सांथथ नोां के म ध्यम से वकय ज ि है ।
Source: https://www.nicdc.in/index.php/about/overview.html
https://pib.gov.in/PressReleaseIframePage.aspx?PRID=1947855

Q.84)
Ans) a
Exp) विकल्प a सही उत्तर है ।
वपछिे प ां च िषों में, भ रि ि र वनय ि ि की ज ने ि िी कृवष िस्तुओां (ब समिी च िि, कच्च कप स, अरां डी क िेि और र्गेहां) में,
ब समिी च िि क वनय ि ि मूल्य िर्ग ि र सबसे अवधक रह है । इस अिवध के र्दौरान बासमती िािि वनयाित से कमाई
26871 कर ड (2017-18), 32804 कर ड (2018-19), 31026 कर ड (2019-20), 29848 कर ड (2020-21) और
26390 कर ड (2021-22) रही।
भ रि और प वकस्त न अपनी सुर्गांध के विए प्रवसद्ध च िि की प्रीवमयम िांबे द ने ि िी वकस्म के ईर न, इर क, यमन, सऊदी
अरब, सांयुि अरब अमीर ि और सांयुि र ज्य अमेररक जैसे दे िोां में अग्रणी वनय ि िक हैं ।

Source: https://agriwelfare.gov.in/en/AgricultureTrade#:~:text=terms%20of%20value)-,for,-
the%20year%202017
https://pib.gov.in/PressReleaseIframePage.aspx?PRID=1941490#:~:text=of%20agriculture%20and-
,related,-products.%20This%20helps

Forum Learning Centre: Delhi - 2nd Floor, IAPL House, 19 Pusa Road, Karol Bagh, New Delhi - 110005 | Patna - 2nd floor, AG Palace, E Boring Canal
Road, Patna, Bihar 800001 | Hyderabad - 1st & 2nd Floor, SM Plaza, RTC X Rd, Indira Park Road, Jawahar Nagar, Hyderabad, Telangana 500020
9311740400, 9311740900 | https://academy.forumias.com | admissions@forumias.academy | helpdesk@forumias.academy

[63]
PTS 2024 | Test Code: 121405 - Solutions |

Q.85)
Ans) c
Exp) विकल्प c सही उत्तर है ।
विकल्प c सही है : सतिंत वसंह बनाम सहायक पासप टि अवधकारी, भारत सरकार, 1967 के मामिे में सुप्रीम क टि ने
माना वक विर्दे श यािा का अवधकार भारतीय संविधान के अनुच्छेर्द 21 और व्यक्ति के पासप टि के र्दायरे में है । यवद इसे
विवनयवमि करने ि िे क नून क अभ ि हो िो इसे नक र नहीां ज सकि । और इस फैसिे के ब द सांसद ने प सपोटि ज री
करने और प सपोटि दे ने से इनक र करने की सही प्रविय के विए प सपोटि अवधवनयम, 1967 प ररि वकय ।
अनुच्छेद 19(1)(d) दे ि के भीिर उसकी आि ज ही की स्विांत्रि की रक्ष करि है । विदे ि ज ने क उनक अवधक र अनुच्छेद
21 ि र सांरवक्षि है ।
मेनक र्ग ां धी केस, 1978 में सुप्रीम कोटि ने दोहर य वक 'विदे ि य त्र क अवधक र' भ रिीय सांविध न के अनुच्छेद 21 ि र
सांरवक्षि है । न्य य िय ने यह भी कहिे हए अनुच्छेद 21 के द यरे को विस्तृि वकय वक 'व्यस्िर्गि स्विांत्रि ' को व्य पक और
उद रि पूििक समझ ज न च वहए, न वक सांकीणि और सख्त। न्य य िय ने म न वक विवध ि र थथ वपि प्रविय अनुच्छेद 14, 19
और 21 के बीच अांिसिंबांध प्रदवििि करके मनम नी और िकिहीनि से मुि होनी च वहए।
Source: https://www.legalserviceindia.com/legal/article-11996-maneka-gandhi-vs-union-of-india-a-
landmark-legal-battle-for-individual-rights.html
https://timesofindia.indiatimes.com/india/person-cant-be-deprived-of-fundamental-right-to-travel-
abroad-on-ground-of-property-dispute-hc/articleshow/106385515.cms

Q.86)
Ans) c
Exp) विकल्प c सही उत्तर है ।
कर अनुप िन उस अिध रण को सांदवभिि करि है वजस िक करद ि अपने दे ि के कर वनयमोां क अनुप िन करि है (य
अनुप िन करने में विफि रहि है ), उद हरण के विए आय घोवषि करन , ररटनि द स्खि करन और समय पर दे य कर क
भुर्गि न करन ।
विकल्प a गित है : कैशिेस िेनर्दे न क प्र त्सावहत करने से िेनर्दे न में पारर्दवशिता क बढािा वमिता है , वजससे
व्यक्तिय ं के विए अपने राजस्व क कम करके आं कना कवठन ह जाता है और कर अनुप िन में सुध र होि है । इसविए
भ रि सरक र कर अनुप िन में सुध र के विए यह उप य अपन सकिी है ।
विकल्प b गित है : एक सरिीकृि कर द स्खि प्रविय कर अनुप िन को आस न और कम समय िेने ि िी बन िी है , वजससे
अवधक िोर्गोां को भ र्ग िेने के विए प्रोत्स वहि वकय ज ि है । कर अनुप िन में सुध र के विए िस्तु एिां सेि कर द स्खि करने
के क्षेत्र में 'GST कम्प जीशन स्कीम' शुरू करने के भारत सरकार के प्रयास से इसका उर्दाहरण वमिा।
विकल्प c सही है : भारत सरकार द्वारा कर अनुपािन बढाने के विए कर र्दरें बढाने की संभािना नही ं है , क्य वं क उच्च
र्दरें व्यक्तिय ं और व्यिसाय ं क अपनी आय का पूरी तरह से िुिासा करने से हत त्सावहत कर सकती हैं , वजससे कर
चोरी बढ ज एर्गी। उद हरण के विए, िैफर िि कर दरोां और कर र जस्व स्तर के बीच सांबांध को दि ि ि है , यह सुझ ि दे ि है
वक एक वनविि सीम से अवधक कर बढ ने से कर से बचने के व्यिह र में िृस्द्ध और कम अनुप िन के क रण र जस्व कम हो
सकि है ।
विकल्प d गित है : मनी िॉस््रांर्ग र्गविविवधयोां पर अांकुि िर्ग ने के विए क नूनी प्र िध न पेि करने से कर अनुप िन में सुध र
होर्ग । उद हरण के विए, भ रि सरक र ने मनी िॉस््रांर्ग र्गविविवधयोां पर अांकुि िर्ग ने और कर अनुप िन में सुध र के विए
'मनी िॉस््रांर्ग रोकथ म अवधवनयम, 2002' ि र्गू वकय है ।
जैसा वक 2012 के IMF नीवत प्रपि में वनष्कषि वनकािा गया है , "AML ढांिे के प्रभािी उपय ग से कर ि र ं का पता
िगाने की संभािना में िृक्तद् और वनि रक प्रविबांध िर्ग कर और र जस्व में िृस्द्ध करके कर क नूनोां क अनुप िन बढ न
च वहए"। Source: https://www.elibrary.imf.org/view/journals/001/2023/083/article-A001-en.xml

Forum Learning Centre: Delhi - 2nd Floor, IAPL House, 19 Pusa Road, Karol Bagh, New Delhi - 110005 | Patna - 2nd floor, AG Palace, E Boring Canal
Road, Patna, Bihar 800001 | Hyderabad - 1st & 2nd Floor, SM Plaza, RTC X Rd, Indira Park Road, Jawahar Nagar, Hyderabad, Telangana 500020
9311740400, 9311740900 | https://academy.forumias.com | admissions@forumias.academy | helpdesk@forumias.academy

[64]
PTS 2024 | Test Code: 121405 - Solutions |

Q.87)
Ans) b
Exp) विकल्प b सही उत्तर है।
ह ि के वदनोां में, भ रिीय ररजिि बैंक (RBI) ने एक सम िेिी, िचीि और वटक ऊ वडवजटि भुर्गि न प ररस्थथविकी िांत्र बन ने के
विए भ रिीय र ष्टरीय भुर्गि न वनर्गम (NPCI) ि र वनवमिि नए उत्प दोां की एक श्रृांखि िॉन्च की है ।
कथन 1 सही है : परं परागत रूप से, अल्पकाविक िेवडट िाइन तक पहुं ि उन ि ग ं तक सीवमत थी वजनके पास
िेवडट काडि तक पहुं ि थी, वजसके विए कई ज ां च की आिश्यकि होिी थी। अब, नई िुरू की र्गई ' UPI ऑन िेवडट ि इन'
उपयोर्गकि ि ओां को विवभन्न वित्तीय जरूरिोां के विए िेवडट क डि क उपयोर्ग वकए वबन अपने यूपीआई-विांक्ड बैंक ख िोां के
म ध्यम से पूिि-अनुमोवदि िेवडट िक पहां चने में सक्षम बन िी है । उपिब्ध िेवडट की म त्र उपयोर्गकि ि के िेनदे न इविह स,
स ख योग्यि और अन्य क रकोां पर वनभिर करिी है । उपयोर्गकि ि िब उपयोर्ग वकए र्गए िेवडट को एक ब र में ि पस भुर्गि न
करने क वनणिय िे सकिे हैं य अपने वित्तीय विकल्पोां और क्षमि ओां के आध र पर आस न वकस्त योजन एां चुन सकिे हैं ।
कथन 2 ग़ित है : UPI LITE एक नया भुगतान समाधान है ज कम मूल्य के िेनर्दे न क संसावधत करने के विए
वििसनीय NPCI कॉमन िाइब्रेरी (CL) एक्तप्लकेशन का िाभ उठाता है , वजसे ₹ 500 से कम पर सेट वकय र्गय है ।
यूपीआई ि इट एक ऐसी सुविध है जो उपयोर्गकि ि ओां को यूपीआई वपन की आिश्यकि के वबन कम मूल्य के भुर्गि न िेजी से
करने में सक्षम बन िी है ।
कथन 3 गित है : यूपीआई ि इट X उपयोर्गकि ि ओां को इां टरनेट कनेक्शन के वबन भी िेनदे न करने में सक्षम बन ि है , जैसे
भूवमर्गि स्ट्े िनोां य दू रदर ज के क्षेत्रोां में। यह फीचर वनयर फील्ड कम्युवनकेिन (NFC) सपोटि ि िे वडि इस पर क म करि है ।
इस प्रक र, प्रेषक और प्र प्तकि ि दोनोां को वनकटि में स्थथि होन आिश्यक है ।इस प्रक र 'यूपीआई X ि इट फैवसविटी' क
उद्दे श्य भौर्गोविक रूप से दू र स्थथि व्यस्ियोां के बीच िेनदे न में िेजी ि न और सुव्यिस्थथि करन है , यह सच नहीां है ।
कथन 4 सही है : UPI टै प एां ड पे सुविध स्म टि फोन पर वनयर फील्ड कम्युवनकेिन (NFC) िकनीक क उपयोर्ग करके टचिेस
भुर्गि न को सक्षम बन िी है । स्कैन एां ड पे में, भुर्गि नकि ि अपनी UPI ID/VPA प्र प्त करने के विए प्र प्तकि ि के QR कोड को
स्कैन करने के विए अपने कैमरे क उपयोर्ग करि है । इसके विपरीि, टै प एां ड पे में, भुर्गि नकि ि NFC के म ध्यम से अपनी
UPI आईडी प्र प्त करने के विए ररसीिर के वडि इस पर अपने वडि इस को टै प करि है और इसके विए वकसी इां टरनेट
कनेस्िविटी की आिश्यकि नहीां होिी है ।
Source: https://www.npci.org.in/PDF/npci/press-releases/2023/NPCI-Press-Release-RBI-Governor-
Launches-Key-Digital-Payment-Initiatives-at-Global-Fintech-Fest-2023.pdf
https://bfsi.economictimes.indiatimes.com/news/fintech/explained-what-is-upi-lite-x-and-how-is-it-
different-from-upi-
lite/103577131#:~:text=They%20can%20now%20send%20or,in%20locations%20with%20no%20connectiv
ity
https://razorpay.com/blog/rbi-rolls-out-credit-lines-on-upi/

Q.88)
Ans) c
Exp) विकल्प c सही उत्तर है ।
विकल्प a गित है : ितिमान में, अं तराििरीय उत्तर-र्दवक्षण पररिहन गवियारे (INSTC) के 13 सर्दस् हैं , अथाित्- भारत,
ईरान, रूस, अजरबैजान, आमेवनया, कजावकस्तान, वकवगिस्तान, तावजवकस्तान, तुकी, यूिेन, बेिारूस, ओमान और
सीररया। इस प्रक र, बेिीज़ (दवक्षण अमेररक मह िीप में स्थथि) और क बो िदे (अफ्ीकी मह िीप में स्थथि) INSTC के सदस्य
नहीां हैं ।

Forum Learning Centre: Delhi - 2nd Floor, IAPL House, 19 Pusa Road, Karol Bagh, New Delhi - 110005 | Patna - 2nd floor, AG Palace, E Boring Canal
Road, Patna, Bihar 800001 | Hyderabad - 1st & 2nd Floor, SM Plaza, RTC X Rd, Indira Park Road, Jawahar Nagar, Hyderabad, Telangana 500020
9311740400, 9311740900 | https://academy.forumias.com | admissions@forumias.academy | helpdesk@forumias.academy

[65]
PTS 2024 | Test Code: 121405 - Solutions |

विकल्प b गित है : फ इनेंवसयि एक्शन ट स्क फोसि (FATF) मनी िॉस््रांर्ग और आिांकि दी वित्तपोषण से वनपटने के विए
कमजोर उप योां ि िे क्षेत्र वधक रोां की पहच न करि है और उन्हें िैक और ग्रे सूची में रखि है । बेिीज, क बो िडे , अजरबैज न
और ि वजवकस्त न को एफएटीएफ ि र ग्रे विस्ट् में ि वमि नहीां वकय र्गय है ।
विकल्प c सही है : 2023 में, विि स्व स्थ्य सांर्गठन (WHO) ने अपने क्षेत्रोां में मिेररय उन्मूिन के विए बेिीज, क बो िडे ,
अजरबैज न और ि वजवकस्त न को मिेररय मुि घोवषि वकय है ।
विकल्प d गित है : जबवक भारत के पास वगसर वमविटर ी एयर डर म (GMA) या तावजवकस्तान के गांि अयनी के नाम
पर अयनी हिाई अड्डे के नाम से जाना जाने िािा सैन्य अड्डा है , िेवकन अजरबैज न, बेिीज और क बो िडे जैसे दे िोां में
इसक कोई सैन्य अड्ड नहीां है ।
Source: https://cdn.who.int/media/docs/default-source/malaria/mpac-documentation/mpag-march2024-
session7-elimination-including-zoonotic-
malaria.pdf?sfvrsn=ba84c611_3#:~:text=Note%20that%20the%20true%20impact,)%20were%20declared%
20malaria%2Dfree.
https://www.mea.gov.in/lok-
sabha.htm?dtl/37388/question+no1085+international+northsouth+transport+corridor#:~:text=At%20pre
sent%2C%20there%20are%2013,joined%20as%20an%20Observer%20State.
https://www.fatf-gafi.org/en/countries/black-and-grey-lists.html#:~:text=to%20as%20the-,grey,-list.
https://theprint.in/defence/gissar-military-aerodrome-indias-first-overseas-base-that-came-to-the-
rescue-in-afghan-crisis/720356/

Q.89)
Ans) a
Exp) विकल्प a सही उत्तर है ।
यूरोपीय मुि व्यापार संघ (EFTA) की स्थापना 1960 में स्टॉकहोम कन्वेंशन द्वारा की गई थी। EFTA ने यूरोप के भीतर और
बाहर तीसरे दे शों के साथ सनिय रूप से व्यापार संबंिों को आगे बढ़ाया है । हाल ही में भारत ने यूरोपीय मुि व्यापार संघ के
साथ 100 अरब डॉलर के मुि व्यापार समझौते पर हस्ताक्षर नकये हैं ।
यूरोपीय मुक्त व्यापार सांघ (EFTA) आइसलैंड, वलकर्ें स्टीन, नॉिे और क्तस्वर्ट िरलैंड का अांतरसरकारी सांगठन है ।
इसकी स्थापना 1960 में इसके तत्कालीन सात सदस्य राज्यों द्वारा अपने सदस्यों के बीच मुि व्यापार और आनथणक एकीकरण
को बढ़ावा दे ने के नलए की गई थी।
Source: https://www.efta.int/about-efta/european-free-trade-
association#:~:text=Free%20Trade%20Association%20(-,EFTA,-)%20is%20the%20intergovernmental
https://www.weforum.org/agenda/2024/03/efta-india-free-trade-
deal/#:~:text=Liechtenstein%2C%20Norway%2C%20and-,Switzerland,-have%20made%20the

Q.90)
Ans) c
Exp) विकल्प c सही उत्तर है ।
भारत में, सूक्ष्म, लघु और मध्यम उद्यमोां (MSME) का िगीकरण वानषणक कारोबार मानदं ड (दोनों के नलए) के साथ-साथ
नवननमाण ण संस्थाओं के नलए संयंत् और मशीनरी में ननवेश और सेवा उद्यमों के नलए उपकरणों में ननवेश के आिार पर नकया
जाता है । सरकार बदलते आनथणक पररर्दश्य को प्रनतनबंनबत करने और इन उद्यमों को आवश्यक समथणन और प्रोत्साहन प्रदान
करने के नलए इन मानदं डों को ननयनमत रूप से अपडे ट करती है ।
MSME की पररभाषा का मौजूदा मानदं ड MSMED अवधवनयम, 2006 पर आधाररत है ।

Forum Learning Centre: Delhi - 2nd Floor, IAPL House, 19 Pusa Road, Karol Bagh, New Delhi - 110005 | Patna - 2nd floor, AG Palace, E Boring Canal
Road, Patna, Bihar 800001 | Hyderabad - 1st & 2nd Floor, SM Plaza, RTC X Rd, Indira Park Road, Jawahar Nagar, Hyderabad, Telangana 500020
9311740400, 9311740900 | https://academy.forumias.com | admissions@forumias.academy | helpdesk@forumias.academy

[66]
PTS 2024 | Test Code: 121405 - Solutions |

MSME श्रेणी प्लां ट और मशीनरी में ननवेश की टनणओवर के नलए ऊपरी सीमा
ऊपरी सीमा
सूक्ष्म उद्यम 1 crore 5 crores
लघु उद्यम 10 crores 50 crores
मध्यम उद्यम 50 crores 250 crores
कथन 1 सही है: 5 करोड से अनिक वानषणक कारोबार वाली एक पांिीकृत साझेदारी फमच को 'सूक्ष्म' उद्यम के रूप में वगीकृत
नकया जा सकता है ।
कथन 2 सही है : 50 करोड से अनिक वानषणक कारोबार वाले पंजीकृत टर स्ट को 'लघु' उद्यम के रूप में वगीकृत नकया जा
सकता है ।
कथन 3 सही है : संयंत् और मशीनरी में 50 करोड से अनिक के ननवेश वाले एक स्वयां सहायता समूह (एसएर्िी) को
'मध्यम' उद्यम के रूप में वगीकृत नकया जा सकता है ।
Source: https://msme.gov.in/know-about-msme
https://msme.gov.in/faqs/q1-what-definition-msme

Q.91)
Ans) c
Exp) विकल्प c सही उत्तर है ।
साइबर खतरा नकसी भी दु भाण वनापूणण कायण या गनतनवनि को संदनभणत करता है जो नडनजटल जानकारी या बुननयादी ढां चे की
गोपनीयता, ननष्ठा या उपलब्धता से समझौता करना चाहता है । साइबर खतरे व्यल्कियों, संगठनों या सरकारों को लनक्षत कर
सकते हैं , विनका उद्दे श्य अनवधकृत पहुां र् हावसल करने , सांिेदनशील डे र्ा र्ुराने , सांर्ालन में बाधा डालने या वित्तीय
नुकसान पहुां र्ाने के वलए कांप्यूर्र वसस्टम, नेर्िकच या सॉलर्िेयर में कमिोररयोां का फायदा उठाना है ।
युग्म 1 सही सुमेवलत है : वडक्तस्टर ब्यूर्ेड डे वनअल ऑफ सविचस (DoS) हमले में एक वेबसाइट पर अत्यनिक मात्ा में
धोखाधड़ी िाले र्र ै वफक की बाढ आ िाती है , नजससे यह बहुत िीमी हो जाती है या वैि उपयोगकताण ओं के नलए पूरी तरह से
अनुपलब्ध हो जाती है । हमलावर अनुरोिों की अनिकता करके लनक्षत सवणर या नेटवकण बुननयादी ढां चे पर दबाव डालते हैं ,
उपलब्ध संसािनों को समाप्त कर दे ते हैं और सेवा में व्यविान पैदा करते हैं ।
युग्म 2 सही सुमेवलत है : स्टर क्चडच क्वेरी लैंग्वेि (SQL) कोड इं जेक्शन तब होता है िब एक दु भाचिनापूणच कोड िेब
एक्तप्लकेशन में डाला िाता है , जो डे टाबेस क्वेरी में हे रफेर करने के नलए कमजोररयों का फायदा उठाता है । हमलावर डे टा को
ननकालने, संशोनित करने या नष्ट करने जैसी अननिकृत गनतनवनियों को अंजाम दे सकते हैं , नजससे संभानवत रूप से डे टाबे स
की अखंडता और गोपनीयता से समझौता हो सकता है ।
युग्म 3 सही सुमेवलत है : िॉस-साइर् क्तिविांग (XSS) में हमलािरोां द्वारा अन्य उपयोगकताचओ ां द्वारा दे खे गए िेब पेिोां में
दु भाचिनापूणच कोड, आमतौर पर जावाल्किप्ट, को इं जेक्ट करना शानमल है । िब वबना सोर्े-समझे उपयोगकताच समझौता
वकए गए पृष्ोां पर िाते हैं , तो इां िेक्टेड क्तिि उनके िाउजर में वनष्पावदत होती है , नजससे हमलावरों को जानकारी चुराने ,
सेशन को हाईजैक करने या अन्य दु भाण वनापूणण गनतनवनियां करने की अनुमनत नमलती है ।
युग्म 4 गलत सुमेवलत है : जीरो-डे एक्सप्लॉइर्ट स का तात्पयच सॉलर्िेयर या हाडच िेयर में कमिोररयोां का फायदा उठाना
है िो वििेता या डे िलपसच के वलए अज्ञात हैं। हमलावर पैच या नफक्स उपलब्ध होने से पहले हमले शुरू करने के नलए इन
कमजोररयों का लाभ उठाते हैं , नसस्टम से समझौता करने, डे टा चोरी करने या व्यविान पैदा करने के अवसर का फायदा उठाते
हैं । 'पासिडच अर्ै क' का तात्पयण नवनभन्न लोकनप्रय संयोजनों को बार-बार आजमाकर पासवडण चुराने के नलए िूर बल के हमलों
का उपयोग करना है । इसनलए, नदया गया युग्म गलत सुमेनलत है ।
Source: https://www.ibm.com/blog/types-of-cyberthreats/

Forum Learning Centre: Delhi - 2nd Floor, IAPL House, 19 Pusa Road, Karol Bagh, New Delhi - 110005 | Patna - 2nd floor, AG Palace, E Boring Canal
Road, Patna, Bihar 800001 | Hyderabad - 1st & 2nd Floor, SM Plaza, RTC X Rd, Indira Park Road, Jawahar Nagar, Hyderabad, Telangana 500020
9311740400, 9311740900 | https://academy.forumias.com | admissions@forumias.academy | helpdesk@forumias.academy

[67]
PTS 2024 | Test Code: 121405 - Solutions |

Q.92)
Ans) c
Exp) विकल्प c सही उत्तर है ।
कथन-I सही है : इलेक्तक्टरक प्रीवसवपर्े र्सच, विन्ें इलेक्टरोस्टै वर्क प्रीवसवपर्े र्सच भी कहा िाता है , थमचल पािर प्लाांर्ोां के
भीतर प्रदू िण को कम करने में महत्वपू णच भूवमका वनभाते हैं । ये पौधे िीिाश्म ईांधन को िलाकर वबिली उत्पन्न करते
हैं , एक ऐसी प्रविया िो हावनकारक प्रदू िकोां को िायुमांडल में उत्सविचत करती है । इलेल्कक्टरक प्रीनसनपटे टर इन संयंत्ों में
ननयोनजत उत्सजणन ननयंत्ण प्रणानलयों के अनभन्न अंग हैं , जो दहन के दौरान उत्सनजणत कण पदाथण और अन्य प्रदू षकों को कम
करने में सहायता करते हैं ।
कथन-II गलत है : इलेक्टरोस्टै वर्क प्रीवसवपर्े र्र इलेक्टरोस्टै वर्क आकिचण के वसद्धाांत पर काम करते हैं , विसका अथच है
वक िे गै स धारा से कण पदाथच को इकट्ठा करने और हर्ाने के वलए र्ुांबकीय िेत्र का नही ां बक्ति विदट युत िेत्र उत्पन्न
करने के वलए विदट युत बलोां का उपयोग करते हैं । जैसे ही दहन प्रनिया से ननकलने वाली नग्रप गैसें प्रीनसनपटे टर से होकर
गुजरती हैं , वे आवेनशत प्लेटों या इलेक्टरोडों की एक श्रृंखला का सामना करती हैं । चूाँनक फ़्लू गैसें चुंबकीय क्षेत् के बजाय नवद् युत
क्षेत् से होकर गुजरती हैं , गैस िारा में ननलंनबत िूल के कण और अन्य प्रदू षक आवेनशत हो जाते हैं । तब आवेनशत कण एक
इलेक्टरोस्टै नटक बल का अनुभव करते हैं , नजससे वे अवक्षेपक के भीतर नवपरीत रूप से आवेनशत प्लेटों या इलेक्टरोडों की ओर
आकनषणत होते हैं और नचपक जाते हैं ।
Source: https://www.epa.gov/air-emissions-monitoring-knowledge-base/monitoring-control-technique-
electrostatic-precipitators

Q.93)
Ans) a
Exp) विकल्प a सही उत्तर है ।
एयर-इं नडपेंडेंट प्रोपल्शन (AIP) तकनीक डीजल इं जन के नलए ऑक्सीजन का एक वैकल्कल्पक स्रोत प्रदान करके काम करती है ,
नजससे वायु इन्टे क के नलए सतह की आवश्यकता समाप्त हो जाती है । AIP नसस्टम गैर-परमाणु पनडु ल्कब्बयों को संग्रहीत
अनभकारकों या कम-शल्कि वाले परमाणु बैटरी चाजणर के माध्यम से प्राप्त वायुमंडलीय ऑक्सीजन पर ननभणर नकए नबना कायण
करने में सक्षम बनाता है ।
कथन 1 सही है : ईंिन सेल व्यापक रूप से मां ग वाली प्रौद्योनगकी के रूप में उभर रहे हैं और संभवतः भनवष्य में AIP
प्रौद्योनगनकयों के फ्लैग बेयरर के रूप में उभर सकते हैं । यह प्रणाली प्राथनमक अपनशष्ट उत्पाद के रूप में जल के साथ नवद् युत
ऊजाण का उत्पादन करने के नलए हाइडरोजन और ऑक्सीजन अणुओं के संयोजन के मूल नसद्धां त पर काम करती है ।
स्टनलिंग इं जन डीजल ईंिन तेल के साथ तरल ऑक्सीजन के दहन से नबजली उत्पन्न करते हैं । ऊजाण स्रोत को नसस्टम के नहस्से के
रूप में स्थायी रूप से मौजूद कायणशील तरल पदाथण से ननकाला जाता है । इं जन को कायणशील तरल पदाथण से ननकाली गई ऊष्मा
का उपयोग करके चलाया जाता है । नफर ननकाली गई ऊजाण का उपयोग या तो बैटरी को ररचाजण करने के नलए या पनडु ब्बी के
प्रत्यक्ष प्रणोदक भार के नलए नकया जाता है ।
कथन 2 गलत है : स्टनलिंग इं जन डीजल ईंिन तेल के साथ तरल ऑक्सीजन के दहन से नबजली उत्पन्न करते हैं । ये प्रणानलयााँ
स्वाभानवक रूप से भारी होती हैं और मूक ईंिन सेल AIP प्रणानलयों की तुलना में गोपनीयता में कमी लाती हैं । 50-70% की
दक्षता के साथ ईंिन सेल अन्य AIP प्रणानलयों की तुलना में बहुत आवश्यक लचीलापन, संचालन में आसानी और बढ़ी हुई
गोपनीयता प्रदान करते हैं ।
Source: https://indianexpress.com/article/explained/submarine-tech-that-india-wants-aip-technology-
7900043/
https://www.financialexpress.com/business/defence-drdos-fuel-cell-based-aip-technology-will-
revolutionize-indias-submarine-capabilities-3181366/
https://indianexpress.com/article/india/drdo-and-naval-group-france-sign-pact-to-fit-indigenous-aip-
system-on-kalvari-class-submarines-8400748/

Forum Learning Centre: Delhi - 2nd Floor, IAPL House, 19 Pusa Road, Karol Bagh, New Delhi - 110005 | Patna - 2nd floor, AG Palace, E Boring Canal
Road, Patna, Bihar 800001 | Hyderabad - 1st & 2nd Floor, SM Plaza, RTC X Rd, Indira Park Road, Jawahar Nagar, Hyderabad, Telangana 500020
9311740400, 9311740900 | https://academy.forumias.com | admissions@forumias.academy | helpdesk@forumias.academy

[68]
PTS 2024 | Test Code: 121405 - Solutions |

https://www.drdo.gov.in/drdo/sites/default/files/inline-files/13678-Article%20Text-48810-1-10-
20200210.pdf
https://www.usni.org/magazines/proceedings/2019/june/non-nuclear-submarines-choose-fuel-
cells#:~:text=Fuel%20Cells%20are-,Efficient,-Fuel%20cell%20AIP
https://www.usni.org/magazines/proceedings/1990/august/air-independent-
propulsion#:~:text=diesel%20subma%C2%ADrines.%20This-,could,-be%20possible%20withv

Q.94)
Ans) d
Exp) विकल्प d सही उत्तर है।
117 करोड ग्राहकों के साथ भारत दु ननया में दू सरे सबसे बडे दू रसंचार पाररल्कस्थनतकी तंत् के रूप में उभरा है । संचार के अलावा,
मोबाइल फोन का उपयोग बैंनकंग, मनोरं जन, ई-लननिंग, स्वास्थ्य सेवा, सरकारी सेवाओं का लाभ उठाने आनद के नलए नकया जा
रहा है ।
इसनलए यह महत्वपूणण है नक उपयोगकताण ओं को नवनभन्न िोखािडी जैसे पहचान नछपाना, जाली केवाईसी, मोबाइल उपकरणों
की चोरी, बैंनकंग िोखािडी आनद से बचाया जाए।
उपयोगकताण ओं की सुरक्षा के नलए, दू रसंचार नवभाग ने संचार साथी नाम से एक नागररक केंनद्रत पोटण ल नवकनसत नकया है । यह
नागररकों को इसकी अनुमनत दे ता है :
➢ उनके नाम पर दजण कनेक्शनों की जां च करने में ।
➢ िोखािडी वाले या अनावश्यक कनेक्शन की ररपोटण करने में।
➢ चोरी/गुम हुए मोबाइल फोन को ब्लॉक करने में ।
➢ मोबाइल फोन खरीदने से पहले IMEI की सत्यता जां च करने में ।
पूरी प्रणाली को दू रसंचार नवभाग द्वारा इन-हाउस नडजाइन नकया गया है ।
Source: https://pib.gov.in/PressReleaseIframePage.aspx?PRID=1924552
https://www.india.gov.in/spotlight/sanchar-saathi-portal
https://vikaspedia.in/e-governance/mobile-governance/sanchar-saathi-portal
https://www.cdac.in/index.aspx?id=hpc_nsf_siddhi-AI

Q.95)
Ans) c
Exp) विकल्प c सही उत्तर है
कथन 1 सही है : चेरेनकोव नवनकरण तब होता है जब नवद् युत आवेनशत कण, जैसे इलेक्टरॉन या प्रोटॉन, जल जैसे पारदशी
माध्यम में प्रकाश की गनत से अनिक हो जाते हैं । इस गनत से जल के अणुओं और कणों के बीच परस्पर निया होती है , नजसके
पररणामस्वरूप प्रकाश का उत्सजणन होता है ।
• जल में, प्रकाश ननवाण त की तुलना में अपनी गनत से 75% अनिक गनत से चलता है , नजससे परमाणु ईंिन द्वारा छोडे गए कण
जल के भीतर प्रकाश की तुलना में तेजी से आगे बढ़ सकते हैं ।
• यह उच्च गनत की गनत आवेनशत कणों को उनके मागण में जल के अणुओं को अवरोि का कारण बनती है , नजसके
पररणामस्वरूप प्रकाश कण, या फोटॉन ननकलते हैं । यह अंतः निया प्रकाश की "शॉकवेव" के समान एक र्दश्यमान नीली या
बैंगनी चमक उत्पन्न करती है ।
कथन 2 सही है : नवशेष उपकरणों का उपयोग करके, अथाणत् नेक्स्ट जनरे शन के चेरेनकोव दे खने के उपकरण (XCVD) या
नडनजटल चेरेनकोव दे खने वाले उपकरण (DCVD), जो उत्सनजणत प्रकाश को कैप्चर करते हैं , परमाणु सुरक्षा ननरीक्षक परमाणु
सुनविाओं और अन्य स्थानों पर परमाणु सामग्री का नवश्लेषण कर सकते हैं और उस डे टा की तुलना राज्य द्वारा ररपोटण की गई

Forum Learning Centre: Delhi - 2nd Floor, IAPL House, 19 Pusa Road, Karol Bagh, New Delhi - 110005 | Patna - 2nd floor, AG Palace, E Boring Canal
Road, Patna, Bihar 800001 | Hyderabad - 1st & 2nd Floor, SM Plaza, RTC X Rd, Indira Park Road, Jawahar Nagar, Hyderabad, Telangana 500020
9311740400, 9311740900 | https://academy.forumias.com | admissions@forumias.academy | helpdesk@forumias.academy

[69]
PTS 2024 | Test Code: 121405 - Solutions |

जानकारी से कर सकते हैं । उदाहरण के नलए, वे तालाबों में मौजूद चेरेनकोव नवनकरण को माप सकते हैं िहाां परमाणु
ररएक्टरोां से खर्च वकए गए ईांधन को सांग्रहीत वकया िाता है और यह वनधाचररत वकया िाता है वक खर्च वकए गए परमाणु
ईांधन की ररपोर्च की गई मात्रा सर्ीक है या नही ां। इस तरह, वे पता लगा सकते हैं नक क्ा खचण नकए गए परमाणु ईंिन से
नकसी भी परमाणु सामग्री को उपयोग से हटा नदया गया है ।
नीचे दी गई नचत् जल में मौजू द चे रेनकोव नवनकरण को नदखाती है जो परमाणु ररएक्टरों में ईंिन को घेरती है ।

Source: https://www.iaea.org/newscenter/news/what-is-cherenkov-radiation
https://www.energy.gov/ne/articles/cherenkov-radiation-
explained#:~:text=Cherenkov%20radiation%20happens%20when%20electrically,interact%20to%20give%2
0off%20light.

Q.96)
Ans) c
Exp) विकल्प c सही उत्तर है ।
कथन 1 सही है : सेल-िी डीएनए (cfDNA) डीएनए के छोटे टु कडों को संदनभणत करता है िो विवभन्न भौवतक तरल पदाथों,
मुख्य रूप से रक्त प्लाज्मा में स्वतांत्र रूप से घूमता है । ये टु कडे पूरे शरीर में कोनशकाओं के प्राकृनतक टू टने (मृत्यु) से उत्पन्न
होते हैं । जबनक अनिकां श कोनशकाओं में उनके डीएनए युि एक नानभक होता है , सीएफडीएनए रिप्रवाह में जारी डीएनए
के एक छोटे अंश का प्रनतनननित्व करता है ।
कथन 2 सही है : सेल-फ्ी डीएनए (cfDNA) एक बायोमाकचर है विसका उपयोग कैंसर वनदान, रोग का वनदान और
उपर्ार के वलए वकया िा सकता है । इसका उपयोग रोग की प्रगनत की ननगरानी और उपचार की प्रनतनिया की भनवष्यवाणी
करने के नलए भी नकया जा सकता है ।
बायोमाकणर एक जैनवक अणु है जो शरीर के तरल पदाथण या ऊतकों में पाया जाता है जो एक सामान्य या असामान्य प्रनिया,
ल्कस्थनत या बीमारी को इं नगत करता है । बायोमाकणर को आणनवक माकणर के रूप में भी जाना जाता है ।
कथन 3 सही है : गभचिती मवहला के रक्त (cfDNA िीवनांग या NIPT) से cfDNA विश्लेिण का उपयोग विकासशील
भ्रूण में गुणसूत्र असामान्यताओां के िोक्तखम का आकलन करने के वलए वकया िाता है । गैर-इनवेनसव प्रीनेटल परीक्षण
(NIPT) जोल्कखम को ननिाण ररत करने की एक नवनि है नक एक भ्रूण कुछ आनुवंनशक असामान्यताओं के साथ पैदा होगा। NIPT
मातृ रि में CFDNA के नवश्लेषण पर आिाररत है । परीक्षण गभाण वस्था के 10 सप्ताह की शुरुआत में और प्रसव तक नकया जा
सकता है । यह गुणसूत्ों की असामान्य संख्या के कारण होने वाली कुछ ल्कस्थनतयों के नलए िीन करता है ।
Source: https://www.thehindu.com/sci-tech/science/cell-free-dna-cancer-screening-high-risk-
pregnancy/article67135117.ece
https://www.genome.gov/genetics-glossary/Cell-Free-DNA-Testing
https://www.frontiersin.org/articles/10.3389/fcell.2021.639233/full

Forum Learning Centre: Delhi - 2nd Floor, IAPL House, 19 Pusa Road, Karol Bagh, New Delhi - 110005 | Patna - 2nd floor, AG Palace, E Boring Canal
Road, Patna, Bihar 800001 | Hyderabad - 1st & 2nd Floor, SM Plaza, RTC X Rd, Indira Park Road, Jawahar Nagar, Hyderabad, Telangana 500020
9311740400, 9311740900 | https://academy.forumias.com | admissions@forumias.academy | helpdesk@forumias.academy

[70]
PTS 2024 | Test Code: 121405 - Solutions |

Q.97)
Ans) c
Exp) विकल्प c सही उत्तर है ।
िायुमांडलीय तरां गें प्रयोग (AWE) नासा का एक प्रयोग है िो वनर्ले िायुमांडल में तरां गोां और अांतररि के मौसम के बीर्
सांबांधोां का अध्ययन करता है।
AWE का उद्दे श्य पृथ्वी के आकाश में आकाशीय चमक, नजसे एयरग्लो के रूप में जाना जाता है , पर उनके प्रभाव को दे खकर
वायुमंडलीय गुरुत्वाकषणण तरं गों (AGWs) का अध्ययन करना है । एजीडब्ल्यू बवंडर और तूफान जैसी मौसमी घटनाओं से उत्पन्न
होते हैं , और वे अंतररक्ष तक फैलते हैं , अंतररक्ष के मौसम को प्रभानवत करते हैं और संभानवत रूप से उपग्रह और संचार संकेतों
को बानित करते हैं ।
AWE अपने उन्नत मेसोस्फेररक तापमान मैपर (AMTM) उपकरण का उपयोग करके, पृथ्वी की सतह से लगभग 54 मील (87
नकलोमीटर) ऊपर, मेसोपॉज में AGWs को मापेगा। इन्िारे ड एयरग्लो चमक में नभन्नता की ननगरानी करके, AWE AGWs के
आकार, शल्कि और नवस्तार में अभूतपूवण अंतर्दण नष्ट प्रदान करे गा, नजसमें छोटे पैमाने के तरं ग भी शानमल हैं नजनका पहले पता
नहीं चल पाता था।
यह हमें समझने में मदद करता है
• वायुमंडलीय गुरुत्व तरं गों (एजीडब्ल्यू) के कारण बादल पैटनण को। वायुमंडल में नीचे से गमण , घनी हवा में अनिक जल होता है ,
इसनलए जब हवा और तूफान जैसी मौसमी घटनाएं हवा के उन नहस्सों को अनिक ऊंचाई पर िकेलती हैं , तो वह जल उन तरं गों
के नशखर पर बादल बनाता है ।
• वैज्ञाननकों को ऊपरी वायुमंडल में अंतररक्ष के मौसम में होने वाले पररवतणनों को समझने और अंततः पूवाण नुमानों में सुिार करने
में मदद करने के नलए अंतररक्ष स्टे शन पर एक पचण से हवा की चमक का ननरीक्षण करने में मदद कर सकता है ।
• गुरुत्वाकषणण तरं गें ऊपरी वायुमंडल से कैसे गुजरती हैं । एडब्ल्यूई द्वारा एकत् नकया गया डे टा वैज्ञाननकों को वायुमंडलीय
गुरुत्वाकषणण तरं गों की भौनतकी और नवशेषताओं को ननिाण ररत करने में सक्षम करे गा और स्थलीय मौसम आयनमंडल को कैसे
प्रभानवत करता है , जो उपग्रहों के साथ संचार को प्रभानवत कर सकता है ।
ज्ञानधार:
एयरग्लो:
• एयरग्लो नकसी ग्रह के वायुमंडल से होने वाला हिा प्रकाश उत्सजणन है । यह तब होता है जब ऊपरी वायुमंडल में परमाणु
और अणु सूयण के प्रकाश से उत्तेनजत होते हैं और अपनी अनतररि ऊजाण को मुि करने के नलए प्रकाश उत्सनजणत करते हैं ।
Source: https://science.nasa.gov/science-research/heliophysics/awe-launching-to-space-station-to-study-
atmospheric-waves-via-airglow/
https://indianexpress.com/article/explained/explained-sci-tech/nasa-experiment-study-air-glow-
atmospheric-waves-explained-9035757/
https://www.nasa.gov/solar-system/why-nasa-watches-airglow-the-colors-of-the-upper-atmospheric-
wind/#:~:text=Airglow%20occurs%20when%20atoms%20and,and%20capture%20a%20free%20electron.

Q.98)
Ans) b
Exp) विकल्प b सही उत्तर है
भारतीय खाद्य सुरक्षा और मानक प्रानिकरण (FSSAI) खाद्य सुरक्षा और मानक अनिननयम, 2006 (FSS अनिननयम) के तहत
स्थानपत एक वैिाननक ननकाय है ।
कथन 1 सही है : FSSAI एक स्वायत्त िैधावनक वनकाय है । FSSAI के नलए प्रशासननक मांत्रालय स्वास्थ्य और पररिार
कल्याण मांत्रालय है ।

Forum Learning Centre: Delhi - 2nd Floor, IAPL House, 19 Pusa Road, Karol Bagh, New Delhi - 110005 | Patna - 2nd floor, AG Palace, E Boring Canal
Road, Patna, Bihar 800001 | Hyderabad - 1st & 2nd Floor, SM Plaza, RTC X Rd, Indira Park Road, Jawahar Nagar, Hyderabad, Telangana 500020
9311740400, 9311740900 | https://academy.forumias.com | admissions@forumias.academy | helpdesk@forumias.academy

[71]
PTS 2024 | Test Code: 121405 - Solutions |

कथन 2 गलत है : FSSAI कृनष उत्पादों के नलए AGMARK (कृनष नचह्न) प्रमाणन नचह्न प्रदान नही ं करता है । AGMARK एक
अलग प्रमाणन प्रणाली है नजसका प्रबंिन कृनष और नकसान कल्याण मंत्ालय के तहत नवपणन और ननरीक्षण ननदे शालय (DMI)
द्वारा नकया जाता है ।
कथन 3 सही है : खाद्य सुरक्षा और मानक (खाद्य पदाथों का फोनटण नफकेशन) नवननयम, 2018 के अनुसार, FSSAI के पास भारत
सरकार के वनदे शोां या राज्योां/केंद्रशावसत प्रदे शोां की वसफाररशोां के आधार पर और वहतधारकोां के परामशच से वकसी भी
प्रधान भोिन के अवनिायच फोवर्च वफकेशन को वनवदच ष्ट करने का अवधकार है ।
खाद्य सुरक्षा और मानक (खाद्य पदाथों का सुर्दढ़ीकरण) नवननयम, 2018 के अनुसार, "मुख्य खाद्य पदाथण" का अथण दै ननक आिार
पर बडे पैमाने पर उपभोग के नलए खाद्य पदाथण हैं और इसमें चावल, गेहूं, गेहूं का आटा, मैदा, तेल, नमक, दू ि शानमल हैं । और
भोजन की ऐसी अन्य वस्तुएं नजन्हें इस नवननयमन के तहत मुख्य खाद्य पदाथण नानमत नकया जा सकता है ।
ज्ञानधार:
कोडे क्स एनलमेंटेररयस कमीशन एक अं तरराष्टरीय खाद्य मानक ननकाय है नजसे 1963 में खाद्य और कृवि सांगठन (एफएओ)
और विश्व स्वास्थ्य सांगठन (डब्ल्यूएर्ओ) द्वारा संयुि रूप से उपभोिाओं के स्वास्थ्य की रक्षा करने और दु ननया भर में
खाद्य व्यापार में उनचत प्रथाओं को सुनननित करने के उद्दे श्य से स्थानपत नकया गया था।
Source:
https://fssai.gov.in/cms/about-fssai.php
https://extranet.who.int/nutrition/gina/sites/default/filesstore/IND%202016%20FSSAI%20fortification%20r
egulation_4.pdf
https://forumias.com/blog/what-is-fssai-what-are-its-
functions/#:~:text=Food%20Safety%20and%20Standards%20Authority,FSSAI%20is%20an%20autonomou
s%20body.
https://fssai.gov.in/upload/uploadfiles/files/Compendium_Food_Fortification_Regulations_04_03_2021.pdf

Q.99)
Ans) c
Exp) विकल्प c सही उत्तर है ।
NexCAR19 भारत की पहली घरे लू CAR T-सेल थेरेपी है , जो कैंसर के नलए एक अभूतपूवण उपचार है । यह नवशेष रूप से उन
कैंसर कोनशकाओं को लनक्षत करता है नजनमें CD19 प्रोटीन होता है , जो B नलम्फोसाइटों के नलए एक बायोमाकणर है ।
NexCAR19 का नवकास एक दशक तक चला सहयोगात्मक प्रयास था। इस थेरेपी को बनाने के नलए भारतीय प्रौद्योवगकी
सांथथान (आईआईर्ी) बॉिे और र्ार्ा मेमोररयल सेंर्र (र्ीएमसी) के शोधकताचओ ां ने वमलकर काम वकया। अक्टू बर
2023 में, भारत की ननयामक संस्था, सेंटरल डरग्स स्टैं डडण कंटर ोल ऑगणनाइजेशन ने NexCAR19 को मंजूरी दे दी।
ज्ञानधार:
सीएआर-टी सेल थेरेपी (काइमेररक एं टीजन ररसेप्टर टी-सेल थेरेपी):
• यह इम्यूनोथेरेपी का एक व्यल्किगत रूप है ।
• इसमें मरीज की टी कोनशकाओं (प्रनतरक्षा प्रणाली की कोनशकाओं) को ननकालना, कैंसर कोनशकाओं को पहचानने और उन
पर हमला करने के नलए उन्हें आनुवंनशक रूप से संशोनित करना और नफर उन्हें मरीज में दोबारा डालना शानमल है ।
Source: https://timesofindia.indiatimes.com/business/india-business/iit-b-startup-immunoacts-
affordable-blood-cancer-therapy-gets-regulatory-approval-a-first-for-indias-car-t-cell-
therapy/articleshow/104427825.cms
https://www.downtoearth.org.in/news/health/cancer-treatment-breakthrough-india-s-homegrown-car-t-
cell-therapy-a-form-of-immunotherapy-gets-market-authorisation-92302

Forum Learning Centre: Delhi - 2nd Floor, IAPL House, 19 Pusa Road, Karol Bagh, New Delhi - 110005 | Patna - 2nd floor, AG Palace, E Boring Canal
Road, Patna, Bihar 800001 | Hyderabad - 1st & 2nd Floor, SM Plaza, RTC X Rd, Indira Park Road, Jawahar Nagar, Hyderabad, Telangana 500020
9311740400, 9311740900 | https://academy.forumias.com | admissions@forumias.academy | helpdesk@forumias.academy

[72]
PTS 2024 | Test Code: 121405 - Solutions |

Q.100)
Ans) b
Exp) नवकल्प b सही उत्तर है
रि एक नवशेष भौनतक तरल पदाथण है । इसके चार मुख्य घटक हैं : प्लाज्मा, लाल रक्त कोवशकाएां , सफेद रक्त कोवशकाएां
और प्लेर्लेर्टस। रि के कई अलग-अलग कायण होते हैं , नजसमें फेफडों और ऊतकों तक ऑक्सीजन और पोषक तत्वों का
पररवहन भी शानमल है । अनतररि रि हानन को रोकने के नलए रि के थक्के बनाना।
कथन 1 सही है : रि प्रकार 'ओ' नेगेनटव वाले व्यल्कियों को उनके आरबीसी पर एां र्ीिन की अनुपक्तथथवत के कारण
सािचभौवमक दाता माना िाता है । र्ाइप O नकारात्मक रक्त में कोई एां र्ीिन नही ां होता है (और यह एकमात्र रक्त
प्रकार है नजसमें कोई भी नहीं होता है ), इसनलए यह प्रनतरक्षा प्रनतनिया को नटर गर नहीं करे गा, भले ही प्राप्तकताण का रि प्रकार
अलग हो।
टाइप O पॉनजनटव रि नकसी भी अन्य रि प्रकार की तुलना में रोनगयों को अनिक नदया जाता है , यही कारण है नक इसे सबसे
अनिक आवश्यक रि प्रकार माना जाता है । लगभग 38% आबादी का रक्त O पॉविवर्ि है , िो इसे सबसे आम रक्त
प्रकार भी बनाता है ।
और, जबनक ओ पॉनजनटव लाल रि कोनशकाएं सभी प्रकार के नलए सावणभौनमक रूप से संगत नही ं हैं , वे नकसी भी सकारात्मक
लाल रि कोनशकाओं (A+, B+, O+, AB+) के साथ संगत हैं । 80% से अनिक आबादी का रि प्रकार पॉनजनटव है , नजसका
अथण है नक वे O पॉनजनटव रि प्राप्त कर सकते हैं ।
कथन 2 सही है : रि समूह 'AB' पॉविवर्ि िाले व्यक्तक्तयोां को उनके प्लाज्मा में एां र्ीबॉडी की अनुपक्तथथवत के कारण
सािचभौवमक प्राप्तकताच माना िाता है । AB पॉविवर्ि रक्त प्रकार को "सािचभौवमक प्राप्तकताच" के रूप में िाना िाता है
क्योांवक AB पॉविवर्ि रोगी सभी रक्त प्रकारोां से लाल रक्त कोवशकाएां प्राप्त कर सकते हैं । AB नेगेनटव रि प्रकार वाले
रोगी सभी नेगेनटव रि प्रकारों से लाल रि कोनशकाएं प्राप्त कर सकते हैं ।
कथन 3 गलत है : Rh एं टीजन, नजसका नाम रीसस बंदरों में पाए जाने वाले समान एं टीजन से नमलता जुलता है , लगभग 80%
मनुष्यों में लाल रि कोनशकाओं (आरबीसी) की सतह पर मौजूद होता है । इस एं टीजन वाले व्यल्कियों को Rh पॉनजनटव
(Rh+ve) के रूप में वगीकृत नकया जाता है , जबनक इसकी कमी वाले व्यल्कियों को Rh नेगेनटव (Rh-ve) के रूप में वगीकृत
नकया जाता है ।
Rh-नेगेवर्ि व्यक्तक्तयोां के Rh-पॉविवर्ि रक्त के सांपकच में आने से एां र्ीबॉडी उत्पादन में िृक्तद्ध होती है , विससे सांभावित
िवर्लताएाँ पैदा होती हैं । इसवलए, प्रवतकूल प्रवतवियाओां से बर्ने के वलए Rh-नेगेवर्ि व्यक्तक्तयोां को केिल अन्य Rh-
नेगेवर्ि दाताओां (Rh+ve व्यक्तक्तयोां से नही ां) से रक्त प्राप्त करना र्ावहए।
कथन 4 गलत है : प्लेर्लेर्टस की तुलना में रक्तदान के बाद लाल रक्त कोवशकाओां (आरबीसी) को पुनः भरने में अवधक
समय लगता है। आमतौर पर शरीर को प्लाज्मा को वफर से भरने में 48 घांर्े, प्लेर्लेर्टस को वफर से भरने में 7 वदन और
लाल रक्त कोवशकाओां को बदलने में 8 सप्ताह तक का समय लगता है ।
Source: https://ncert.nic.in/ncerts/l/kebo118.pdf
https://my.clevelandclinic.org/health/body/22865-plasma#:~:text=your%20blood%20that-,contributes,-
to%2055%25%20of
https://www.redcrossblood.org/faq.html#:~:text=times%20a%20year.-,Platelet,-
apheresis%20donors%20may
https://www.medicalnewstoday.com/articles/blood-donation-facts#:~:text=hours%20to%20replenish-
,plasma,-and%20up%20to

Forum Learning Centre: Delhi - 2nd Floor, IAPL House, 19 Pusa Road, Karol Bagh, New Delhi - 110005 | Patna - 2nd floor, AG Palace, E Boring Canal
Road, Patna, Bihar 800001 | Hyderabad - 1st & 2nd Floor, SM Plaza, RTC X Rd, Indira Park Road, Jawahar Nagar, Hyderabad, Telangana 500020
9311740400, 9311740900 | https://academy.forumias.com | admissions@forumias.academy | helpdesk@forumias.academy

[73]

You might also like